2ちゃんねる ■掲示板に戻る■ 全部 1- 最新50    

■ このスレッドは過去ログ倉庫に格納されています

■ちょっとした物理の質問はここに書いてね280■

1 :ご冗談でしょう?名無しさん:2023/07/09(日) 17:07:10.70 ID:jVjQtupd.net
★荒らし厳禁、煽りは黙殺
★書き込む前にの注意事項を読んでね
★数式の書き方(参考)はこちら   >>2-3

===質問者へ===

重要 【 丸 投 げ 禁 止 】

・質問する前に
1. 教科書や参考書をよく読む
2. http://www.google.com/ などの検索サイトを利用し、各自で調べる
3. 学生は自分の学年、物理科目の履修具合を書く
4. 宿題を聞くときは、どこまでやってみてどこが分からないのかを書く
・質問に対する回答には返答してね、感謝だけでなく「分からん」とかダメ出しでもOK
・質問するときはage&ID表示推奨
・高度すぎる質問には住人は回答できないかもしれないけれど、了承の上での質問なら大歓迎

===回答者へ===

・丸投げは専用スレに誘導
・不快な質問は無視、構った方が負け
・質問者の理解度に応じた適切な回答をよろしく
・単発質問スレを発見したらこのスレッドへの誘導をよろしくね
・逆に議論が深まりそうなら新スレ立てて移動するのもあり
・板違いの質問は適切な板に誘導を
・不適切な回答は適宜訂正、名回答は素直に賞賛

※前スレ
■ちょっとした物理の質問はここに書いてね275■
https://rio2016.5ch.net/test/read.cgi/sci/1651322579/
■ちょっとした物理の質問はここに書いてね276■
https://rio2016.5ch.net/test/read.cgi/sci/1680947631/
■ちょっとした物理の質問はここに書いてね277■
https://rio2016.5ch.net/test/read.cgi/sci/1681468872/
ちょっとした物理の質問はここに書いてね278
https://rio2016.5ch.net/test/read.cgi/sci/1684123736/
■ちょっとした物理の質問はここに書いてね279■
https://rio2016.5ch.net/test/read.cgi/sci/1684973712/

2 :ご冗談でしょう?名無しさん:2023/07/09(日) 17:07:33.02 ID:jVjQtupd.net
●括弧: (), [], {}を適切に入れ子にして分かりやすく書く
●スカラー: a,b,...,z, A,...,Z, α,β,...,ω, Α,Β,...,Ω,...(「ぎりしゃ」「あるふぁ〜おめが」で変換)
●ベクトル: V=(v1,v2,...), |V>,V↑, (混同しないならスカラーの記号でいい。通常は縦ベクトル)
●テンソル: T^[i,j,k...]_[p,q,r,...], T[i,j,k,...; p,q,r,...]  (上下付き1成分表示)
●行列: M[i,j], I[i,j]=δ_[i,j] M = [[M[1,1],M[2,1],...], [M[1,2],M[2,2],...],...], I = [[1,0,0,...],[0,1,0,...],...]
 (右は全成分表示。行または列ごとに表示する。例:M=[[1,-1],[3,2]])
●対角行列: diag(a,b) = [[a,0],[0,b]]
●転置行列・随伴行列:M^T, M†("†"は「だがー」で変換可) ●行列式・トレース:|A|=det(A), tr(A)
●複号: a±b("±"は「きごう」で変換可)
●内積・外積: a・b, a×b
●関数・汎関数・数列: f(x), F[x(t)] {a_n}
●平方根: √(a+b) = (a+b)^(1/2) = sqrt(a+b) ("√"は「るーと」で変換可)
●指数関数・対数関数: exp(x+y)=e^(x+y) ln(x)=log_e(x) (底を省略して単にlogと書いたとき多くは自然対数)
 括弧を省略しても意味が容易に分かるときは省略可: sin(x) = sin x
●三角関数、逆三角関数、双曲線関数: sin(a), cos(x+y), tan(x/2), asin(x)=sin^[-1](x), cosh(x)=[e^x+e^(-x)]/2
●絶対値:|x| ●ノルム:||x|| ●共役複素数:z^* = conj(z)

3 :ご冗談でしょう?名無しさん:2023/07/09(日) 17:07:50.89 ID:jVjQtupd.net
a:加速度、昇降演算子 A:振幅、ベクトルポテンシャル B:磁束密度 c:光速 C:定数、熱・電気容量
d:次元、深さ D:領域、電束密度 e:自然対数の底、素電荷 E:エネルギー、電場
f:周波数 f,F:力 F:Helmholtzエネルギー g:重力加速度、伝導度
G:万有引力定数、Gibbsエネルギー、重心 h:高さ、プランク定数 H:エンタルピー、Hamiltonian、磁場
i:虚数単位 i,j,k,l,m:整数のインデックス I:電流、慣性モーメント j:電流密度・流束密度
J:グランドポテンシャル、一般の角運動量 k:バネ定数、波数、Boltzmann定数 K:運動エネルギー
l,L:長さ L:Lagrangian、角運動量、インダクタンス m,M:質量 n:物質量 N:個数、トルク
M:磁化 O:原点 p:双極子モーメント p,P:運動量、圧力 P:分極 q:波数
q,Q:一般化座標、電荷 Q:熱 r:距離 R:抵抗、気体定数 S:エントロピー、面積 t:時間 T:温度
U:ポテンシャル、内部エネルギー v:速度 V:体積、ポテンシャル、電位
W:仕事、状態数 x,y,z:変数、位置 z:複素変数 Z:分配関数

4 :神の僕 ◆P10fR708T6 :2023/07/09(日) 20:03:37.75 ID:96rfWqlM.net
>>3
>C:定数、熱・電気容量
電気容量なんて言わないぞ。
静電容量だろうがよ?

5 :神の僕 ◆P10fR708T6 :2023/07/09(日) 20:15:57.68 ID:96rfWqlM.net
>L:Lagrangian、角運動量、インダクタンス
正確には自己インダクタンス
相互インダクタンスの量記号はM

6 :ご冗談でしょう?名無しさん:2023/07/10(月) 12:59:49.40 ID:???.net
可能な限りの精度で円周率を宇宙空間や何処かの重力圏内で実測値算出した場合、所謂πとのズレは予測出来ますか?

7 :poem:2023/07/10(月) 13:38:19.83 ID:4SzuvXZl.net
外から見て紙の座標が歪んでても中に居る人の定規がそれに合わせてるからπは歪まない
けど中から見て座標が生成され進行形すると一瞬前の座標と座標同士の繋ぎ方が変わるから斥力引力みたいな場になると思う妄想πは変わらない斥力引力は定規で測るとき人間は排除するから。円運動などの軌道は変わるが
生成され進行形じゃなく生成された後は繋ぎ方変わった後だから場は発生しない。多分m/s速度のような1乗物理が保存量なのと、m/s^2時間加速のような2乗物理が失う量、(√m)/sの距離加速のような1/2乗物理が増える量なのの理由が指数物理は座標生成を行ってる説。もしかしたらね。生成する進行形じゃないと場が生じないなら生成する進行形物理は場を生じてるとしたら、時間加速の後ろに押しつけられるのは場の一種となり、場が生じるから失う量、座標生成を行い続ける物は失う量、逆に座標消滅を続ける物は増える量の可能性。妄想

8 :poem:2023/07/10(月) 13:45:53.90 ID:4SzuvXZl.net
ちょっと皆に聞きたいからこちらに質問
磁石の異極の引力の話
なんかペンとか鉛筆とか、それを角度を変えないまま何かに近づける力学より、それを動かさず角度を変えて近づける力学の方が磁石の引力に近い気がした気のせい?
もしかしたら磁石の引力は
・位置的に近づける力…×
・角度を直角に変える力…○
なんじゃ?と
全ての磁荷(磁荷の実在は観測されてない)1つ1つがその先にある異極の磁荷1つ1つを直角に配置しようとする力が近づける引力にならないか?

電流のローレンツ力とか外積だから電磁気は外積物理=回転物理
なら磁力の引力も回転物理的に角度を直角に変える力の可能性ってあるんじゃ?
ない?

9 :poem:2023/07/10(月) 13:47:10.11 ID:4SzuvXZl.net
まあまともな返信は無いよね

10 :poem:2023/07/10(月) 13:48:16.28 ID:4SzuvXZl.net
偽物スレも消費してお願い
👾🐙🦑

11 :ご冗談でしょう?名無しさん:2023/07/10(月) 15:45:04.59 ID:???.net
››7-8
日本語でOK

12 :ご冗談でしょう?名無しさん:2023/07/10(月) 16:15:48.23 ID:6QuJlE23.net
スマホのボディって様々な色がありますが、最も熱の吸収、放射に優れた色って何になるのでしょうか?
自分の予想だと、プラスチックなら、樹脂自体とSoC側を熱の吸収率がいい黒にして、外気側は熱を反射するように白塗装にしたら一番冷えると思います
金属はメッキ層が薄いので大差ないと思いますが、プラスチックと同様に中は黒、外は白が良いのでは

13 :poem:2023/07/10(月) 17:01:48.07 ID:HsQOYG50.net
マジレスだから荒らしじゃないからね
・ガラスの方がプラスチックより放熱いいだろう
・ガラスより金属の方が放熱いいだろう
・後ろにメタルプレート貼るだけでプラカバーでも放熱上がるだろう。二枚重ねは実はさらに上がるだろう。それとメタルプレートをスマホカバーで覆わず手で持つ手が熱くなるまで放熱効果大。手が濡れてたらなお良し
・黒色だろうと赤外線を吸収しないと意味ない。ようは黒色だろうと日焼け止めの紫外線じゃなく赤外線バージョン塗ってたら意味ない。要するにスマホ発熱の赤外線波長の吸収色がよい。下手に黒色だとスマホカバーが可視光出さないのに周りの可視光を吸収すると思う。赤外線波長じゃないからどれくらい熱上がるかは知らない。いや黒色なら赤外線も吸収するのか?なら黒色だと炎天下は良くない。もしスマホの温度帯の赤外線吸収黒色なら金属と合わせて放熱力上がるかもだけど放熱できる材質じゃないプラスチックなどでは意味ない。世の中にはシリコン放熱シートなる物があるから、シリコンで放熱カバー作ることできそうだけど商品はないんじゃ?。環境に接する方を赤外線反射、スマホ側を赤外線吸収にすればいいんじゃ?可視光は反射してもいいから色んな色大丈夫

14 :poem:2023/07/10(月) 17:04:56.27 ID:HsQOYG50.net
そもそもスマホ温度帯(室温)の赤外線吸収反射剤が実在するのか次第

15 :ご冗談でしょう?名無しさん:2023/07/10(月) 17:29:50.70 ID:???.net
ピカー

16 :ご冗談でしょう?名無しさん:2023/07/10(月) 17:44:32.23 ID:???.net
統計によれば基地外の寿命は平均寿命より20年短いらしい

17 :poem:2023/07/10(月) 18:20:15.55 ID:QliUmlNw.net
>>16
テンクス

18 :poem:2023/07/10(月) 18:20:57.74 ID:QliUmlNw.net
>>16
ちなみに自分も聞いたことあるそれ

19 :ご冗談でしょう?名無しさん:2023/07/10(月) 18:42:13.42 ID:???.net
専ブラ規制でとうとう終了するのかな

20 :ご冗談でしょう?名無しさん:2023/07/10(月) 18:59:27.61 ID:???.net
荒らされまくってほとんど死んでたようなもんだから仕方ないか

21 :poem:2023/07/10(月) 19:49:33.95 ID:F1nYZN01.net
>>19
あ、やっぱり専ブラ規制だったの
5chトップに書き込みトップ板に実況なくてvipだけだったからもしかして専ブラ規制なのかと思ったけど実況が反映されないように壊れただけかと高括ってた
自分専ブラじゃないから規制巻き込まれてないから
前はクロームだったから古いの規制に巻き込まれて違うブラウザにしてる

22 :poem:2023/07/10(月) 19:56:37.82 ID:F1nYZN01.net
ちょっとpoemさん少しオカ板とかその他板とか覗いてこようかな
古いのクローム規制なら異常があるのかないのか異常がないのかもしれないし異常があるとしても今まで見つけられなかったからクローム規制の方は多分見つけられないけど
専ブラ規制はそれより上の異常だと思うから
こういう規制系は、どこかのスレに規制される原因書き込みがあるから、クローム規制よりもしかしたら運良かったり分かり易かったら見つけられるかも知れないから。まあ見つけられないだろうけど、覗いてこようかなと。皆も覗ける人は探してみたら、運いい人は見つけられてその人が何かその何個も後にフォローレスして規制溶かすことできるかもだから、なんとなく探したい人は探してみて。

23 :poem:2023/07/10(月) 20:01:25.61 ID:F1nYZN01.net
関連物は見つけたけど
https://itest.5ch.net/asahi/test/read.cgi/newsplus/1688982186
これが根本じゃないと思う
これの背景があるはず

24 :ご冗談でしょう?名無しさん:2023/07/10(月) 20:12:44.33 ID:Is3gFdBR.net
ChatGPTの性格分析:
臆病で、慇懃無礼で、自己保身が強く、紋切り型で、
知らないとはなかなか言わない、
すぐ謝る、何度も謝る、
前言撤回を頻繁にする

25 :poem:2023/07/10(月) 20:18:01.91 ID:F1nYZN01.net
>>24
自分のレス抽出から性格分析有難いしもっと詳しく分析して欲しいアンド聞きたいんだけど、今の謎な事象優先してるから

wikipedia
Talk掲示板は2023年7月1日に開設された日本の電子掲示板(匿名掲示板)。 Talk Technologies Inc.のMasaによって開設された。 有料サービスであるTalkプレミアムでは規制緩和や専用ブラウザの広告除去ができる。以下、Talkプレミアム機能一覧。

特定板のスレッド作成

スレッド作成時や連続投稿の規制緩和

専用ブラウザの広告除去

特定プロバイダーの規制回避

海外ホストからの書き込み


Talk言語日本語タイプ電子掲示板
(匿名掲示板)運営者Talk Technologies Inc.


専用ブラウザであるJaneStyle、twinkle、2chGearが、Talk対応され閲覧、投稿が可能となっている。


最終編集見たとき10分前
と、一番上に箱、削除がいどらいんに触れるからまもなく削除されますだって

26 :poem:2023/07/10(月) 20:20:58.45 ID:F1nYZN01.net
この新しい掲示板作ったのが今の専ブラ規制の背景だと思う
だとすると専ブラ規制の原因はかなり前に遡る。そしてこのタイミング(7月1日にリリースされて)規制が7月10日なのは切っ掛けが、この今日の日付の直前直後どちらかの過去未来にあるはず

27 :poem:2023/07/10(月) 20:25:12.74 ID:F1nYZN01.net
切っ掛けは近いはずだけど、過去か未来かまでは場合による

原因はかなり前に遡る

直接の原因
直接の結果
真の原因
真の結果

直接の原因は過去未来直近…いや未来直近は過去直近が潜在で顕在化が未来なだけか。直近過去
直接の結果は直接の原因の後
真の原因は遠い未来
真の結果は遠い過去…作る動機

28 :poem:2023/07/10(月) 20:27:38.92 ID:F1nYZN01.net
専ブラ規制、talkに流れろ、なら直近の未来でtalkに何か重要なことが書かれる?7月1日付近から重要なこと書く者がスタンバってるからこのタイミングで規制か?

29 :poem:2023/07/10(月) 20:29:30.46 ID:F1nYZN01.net
>>28
スタンバってる者のムーブメントが直接の原因か、スタンバってる者のムーブメントに合わせて未来に別の真の原因を作り、このタイミングで第三者的に規制の直接の原因を作った?

30 :poem:2023/07/10(月) 20:33:08.93 ID:F1nYZN01.net
とりあえず専ブラ規制そのものの対策は分からないけど、
今見てる皆がやるべきことはわかった。多くの者がtalkに流れなきゃいけない。
見た情報により専ブラ規制そのものの原因レスをフォローするレスが産まれるかも知れない
まず意味もわからずtalkをより多くの者が見ることが今必要なんじゃないかと仮説

31 :ご冗談でしょう?名無しさん:2023/07/10(月) 20:50:22.16 ID:???.net
普通のブラウザだとこういう荒らしのクソをフィルタリングできないから極めて不快

32 :ご冗談でしょう?名無しさん:2023/07/11(火) 00:29:12.88 ID:???.net
ピカー

33 :ご冗談でしょう?名無しさん:2023/07/11(火) 02:31:39.77 ID:???.net
ピカーって、20年くらい前のキャラだろ?

34 :神の僕 ◆P10fR708T6 :2023/07/11(火) 03:27:38.97 ID:EKzWBuBp.net
ということは、おまいも二十年越しか。

35 :poem:2023/07/11(火) 07:42:14.22 ID:3L/WxUuz.net
>>24
今わかったけど、自分って他人を反射する鏡のような人格属性だから、
これについて、3人称の根源(潜在)は2人称、2人称相手が頭悪いとかの根源(潜在)はそれが2人称ならば1人称自分が相手より頭悪いからキレるとか。顕在化には潜在があり0人称特定の者より潜在は無い。ようするに自分は他人にお前馬鹿と思われるけど他人の自分へのお前馬鹿は特定の者として自分でなく相手として自分だから、言う他人が馬鹿。すなわち他人の鏡なのが自分。
だからこのGPTの自分への性格分析もAIを自分の人格がぜったい反射してる、と今さっき気づいた

臆病で、慇懃無礼で、自己保身が強く、紋切り型で、
知らないとはなかなか言わない、
すぐ謝る、何度も謝る、
前言撤回を頻繁にする

これ自分の性格分析じゃなくAIのそのGPT自体の性格分析そのままだよ。ぜったい

36 :poem:2023/07/11(火) 07:43:52.77 ID:3L/WxUuz.net
>>35
これGPTに言ってみたらどうなるかな

37 :poem:2023/07/11(火) 07:48:00.51 ID:3L/WxUuz.net
少なくとも物理とかの少なくとも一部以上の専門板の使い勝手は5chの方が上。NG昨日とか使ってる人多いみたいだしね
大衆板は逆にtalkでも見易かった。ああ、1日30レスまでだから、専門板のレスバはやっぱり無理だから、レスバしない大衆板だけなのは当然か

38 :poem:2023/07/11(火) 07:52:43.72 ID:3L/WxUuz.net
今openの物理見たけど使われてないね
避難所またはサブ利用はopenの方がtalkよりいい気がする

39 :poem:2023/07/11(火) 07:56:35.92 ID:3L/WxUuz.net
ひろゆきクローンの方は使い難すぎるし、というか5chよりひろゆきクローンの方ってアンダーグラウンドと言える雰囲気と使い勝手。あとキッズ多いと言うかキッズは5chはまず規制されるから数回で書き込めなくなりひろゆきクローンはキッズを規制しないからキッズはあそこから入る気がする。だからキッズも多くこれも加えてアンダーグラウンド感増加な感じ

40 :poem:2023/07/11(火) 07:59:34.75 ID:3L/WxUuz.net
物理板に関しては
メイン…5ch
サブ…open(未使用)サブならここ
ひろゆき…アンダーグラウンド放置
talk…利用方法ありそう
という使い勝手では

41 :ご冗談でしょう?名無しさん:2023/07/11(火) 10:26:44.08 ID:???.net
test

42 :ご冗談でしょう?名無しさん:2023/07/11(火) 10:27:26.32 ID:???.net
account.cfgを削除するとjanestyle(4.23)で読み書きできる

43 :神の僕 ◆P10fR708T6 :2023/07/11(火) 21:58:37.90 ID:EKzWBuBp.net
この世のすべてが停止したら時間はなくなりますか?

44 :ご冗談でしょう?名無しさん:2023/07/11(火) 23:20:19.66 ID:???.net
物理的に停止できない(零点振動)から物質系の時間は止まらない。

45 :ご冗談でしょう?名無しさん:2023/07/11(火) 23:22:50.63 ID:???.net
その2人は相手にするだけ無駄

46 :ご冗談でしょう?名無しさん:2023/07/11(火) 23:35:10.49 ID:???.net
>>44
ゼノンのパラドックスによれば、飛んでいる矢は或る時刻で止まっている。

これは物理学的に正しいのか? 間違い
飛んでいる矢の或る時刻の速度は0でないから止まっていない!
つまり、或る時刻に矢が特定の位置に有ると言っているだけだ。

47 :ご冗談でしょう?名無しさん:2023/07/12(水) 03:07:18.99 ID:???.net
ピカー

48 :神の僕 ◆P10fR708T6 :2023/07/12(水) 05:26:17.99 ID:YiteIR7n.net
>>44
宇宙は最終的に消滅しますが、それでも時間は存在しますか?
宇宙の始まりの前は時間は存在しますか?
Youtubeで全ての物体が停止すると時間は存在しなくなる
とか言ってました。

>>46
分かりやすく言えば、どんなにシャッターの速さを早くしても
飛んでる矢の写真にはブレがあるということ。

>>45
つまりね、存在と実在の二人は人智を超えてるんだよ。
俺らがどこからきてどこへ行くかは神のみぞ知る。
アインシュタインは言った。
海辺で歩いてめずらしい貝殻を見つけて楽しんでいる子供のような
事をやってるのが科学者。

49 :ご冗談でしょう?名無しさん:2023/07/12(水) 05:39:06.62 ID:???.net
カルト信者は巣に池

50 :ご冗談でしょう?名無しさん:2023/07/12(水) 05:43:46.26 ID:???.net
絶対零度でも運動は止まらない、不確定性原理

51 :ご冗談でしょう?名無しさん:2023/07/12(水) 05:53:26.70 ID:???.net
宇宙始まりの前とか宇宙の消滅とかは物理学の範疇ではない
「もしも〜の世界」も物理学ではない

52 :ご冗談でしょう?名無しさん:2023/07/12(水) 06:18:16.06 ID:???.net
山口人生とビルゲイツはどっちのほうがすごいですか?

53 :神の僕 ◆P10fR708T6 :2023/07/12(水) 17:43:02.72 ID:YiteIR7n.net
山口県の美しい詩を残した金子みすゞの地を訪ねたことがある。
命ある者に対する切なくもやさしいまなざしで語られる美しい小さな詩。
これほど愛に満ちた詩があろうか。
山口で短い人生を過ごした彼女の詩はいつまでも生きている。
ビルゲイツは世界的に有名な実業家で成功者だがイエスは彼女が偉大であると
言うであろう。

54 :ご冗談でしょう?名無しさん:2023/07/12(水) 18:00:17.49 ID:???.net
その2人は相手にするだけ無駄

55 :神の僕 ◆P10fR708T6 :2023/07/12(水) 18:23:24.29 ID:YiteIR7n.net
金子みすゞが短い生涯のあいだに残した詩に大漁というのがある。
彼女の住んだ地にイワシの大群のイルミネーションと詩が内部の壁にある
小さな建物がある。
イワシは高級魚ではない。どちらかといえば大衆魚だ。
俺はイワシよりもサンマがうまいと思う。昔はサンマも大衆魚だった。
人間の目線とイワシの目線。大漁の日はイワシにはホロコーストの日。
この場合はみんな違ってみんないいなんていえないや。
創世記には生き物を食べていいが、血抜きしてからにせよ、血は命だから。
と神がノアに命令したとある。
金子みすゞとビルゲイツみんなちがってみんないい。
だが、二人ともイワシぐらい食べただろう。

56 :神の僕 ◆P10fR708T6 :2023/07/12(水) 23:30:25.76 ID:YiteIR7n.net
慣性の法則はなぜ成り立つでしょうか?

57 :ご冗談でしょう?名無しさん:2023/07/13(木) 00:28:56.08 ID:???.net
その2人は相手にするだけ無駄

58 :神の僕#sakanakun:2023/07/13(木) 00:59:07.45 ID:f5THZNvO.net
無駄とは何か?
ここは効率とかコスパとか、何かの目的があるのか?
その目的とは?
目的があるにしては無駄がありすぎるし、すぐ崩れる。
まーろくなのいない板。
チャットgptあれのほうがましだが、持ってねえ。
アレクサとか使ったこともねえしなぁ。
慣性がなぜさようするのか説明を求めているのだが、答えられない
のかなぁ?
@運動を起こす原因は力。
A同じ運動に対して物質の量に比例した力がいる。
これだけだろ、説明に必要なことは。

59 :ご冗談でしょう?名無しさん:2023/07/13(木) 05:13:49.12 ID:???.net
ピカー

60 :ご冗談でしょう?名無しさん:2023/07/13(木) 11:37:42.36 ID:???.net
>>56
慣性の法則はニュートン力学(と特殊相対論)の基本仮説だ
始めから成り立つ理由も証明も不可能。

基本仮説から演繹した結果に論理矛盾がなく、現実の実験結果とも一致すれば
慣性の法則が正しいと帰納的に認められる。

物理学の理論は、カルト宗教とは違うのだよカルト宗教とは!

それが分かったらザクのカルト信者は巣のオカ板に池

61 :ご冗談でしょう?名無しさん:2023/07/13(木) 13:35:17.65 ID:???.net
時空の一様性から出るけどな

62 :ご冗談でしょう?名無しさん:2023/07/13(木) 14:18:52.81 ID:???.net
>>61
慣性の法則の正しい解釈は慣性座標系、つまり時空の一様性の仮定だ。

それが理解できない人が第一法則(慣性の法則)は第二法則(運動方程式)
から導出されるから必要ないなどと騒ぐ。

63 :ご冗談でしょう?名無しさん:2023/07/13(木) 14:45:37.78 ID:???.net
>>62
19世紀に非ユークリッド幾何学が発見されるまで
ユークリッド空間で構成された時空間は自明の真理だと信じられていた。

64 :ご冗談でしょう?名無しさん:2023/07/13(木) 17:33:27.02 ID:???.net
非ユークリッド幾何学が発見されると、時空の一様性から慣性の法則が導かれることが否定されるとでも?
対称性と保存則の関係の発見は20世紀であって非ユークリッド幾何学より後なんだが

65 :ご冗談でしょう?名無しさん:2023/07/13(木) 21:29:15.42 ID:???.net
非ユークリッドでも一様だよな

66 :ご冗談でしょう?名無しさん:2023/07/13(木) 22:31:43.34 ID:???.net
>>64
>非ユークリッド幾何学が発見されると、時空の一様性から慣性の法則が導かれることが否定されるとでも?

 >>62-63 の日本語が全く理解できないようだな
ユークリッド空間が唯一無二の実在では無いということだ。
時空の一様性とユークリッド空間は同様の意味で、慣性の法則はその仮定(仮説)だ
非ユークリッド幾何学のような曲がった時空間では通常の慣性の法則は成り立たない。

>>65
馬鹿

67 :ご冗談でしょう?名無しさん:2023/07/13(木) 22:46:11.37 ID:???.net
>対称性と保存則の関係
ネーターの定理のことなら、基本的にユークリッド空間だ。
非ユークリッド幾何学のような曲がった時空間ならそのままでは成り立たない。

68 :ご冗談でしょう?名無しさん:2023/07/13(木) 23:02:43.89 ID:???.net
現代から見れば(謎の古代ギリシャ人)ユークリッドの偉大な業績は
平行線公理が他の公理から導出できない事実を最初に発見したことである。
つまり、
ユークリッド幾何学(ユークリッド空間)は平行線公理の仮定(仮説)
から創られたということだ。

69 :ご冗談でしょう?名無しさん:2023/07/14(金) 00:14:26.49 ID:???.net
馬鹿って人を馬鹿と言いたがるよね

70 :ご冗談でしょう?名無しさん:2023/07/14(金) 00:15:49.37 ID:???.net
>>67
曲がった時空でもキリングベクトルがあれば良い

71 :ご冗談でしょう?名無しさん:2023/07/14(金) 06:59:21.44 ID:???.net
>>68
現代でも
殆どの人は3角形の内角の和が2直角(180度)であり、論理矛盾が無い唯一の答え
だと学習し信じ込んでる。
2直角(180度)以外でも論理矛盾が無い幾何学理論が構築できるなど理解不能だ
球体表面の三角形とか言われれも球体表面が曲がっているだけであり空間が曲がっ
ているのではない。
ユークリッド空間の慣性系(ニュートン力学の第一法則)が仮定(仮説)だと
理解するのは難しい。

72 :ご冗談でしょう?名無しさん:2023/07/14(金) 07:02:44.95 ID:???.net
ピカー

73 :ご冗談でしょう?名無しさん:2023/07/14(金) 07:38:21.50 ID:dShLqPJ71
物理学は全てが仮定(仮説)なのはあたりまえ
一つの例外もない

74 :神の僕 ◆P10fR708T6 :2023/07/14(金) 08:08:42.12 ID:fxUtT6Gd.net
>>71
>2直角(180度)以外でも論理矛盾が無い幾何学理論が構築できる
東洋館出版社 理論・演習 数学新辞典 235〜236ページ引用

公理V 直線外の1点を通ってこの直線に平行な直線はただ一つある。

ある公理系を基本として数学を組み立てるとき、この公理系の公理の間には
たがいに矛盾がないことが必要である。これを公理系の無矛盾性という。
またこの公理系の内容に重複がないということが望ましい。これを公理系の
独立性という。
たとえば、・・・公理Vのみ変えて”直線外の一点を通って、この直線に平行
な直線は無数にある”という公理を採用すると1つの幾何学が組み立てられそ
の間になんら矛盾が起らないことが分かっている。これは公理Vが他の諸公
理と矛盾のないことを示すものである。

この記述を解釈すれば、公理Vが他の公理と矛盾しないのは、公理Vが独立
であるからだということになろうが、三角形の内角の和が180度であるこ
とは公理ではなく、証明可能な定理なので当てはまらないと考えられるのだが。

75 :神の僕 ◆P10fR708T6 :2023/07/14(金) 08:08:49.06 ID:fxUtT6Gd.net
>>71
>2直角(180度)以外でも論理矛盾が無い幾何学理論が構築できる
東洋館出版社 理論・演習 数学新辞典 235〜236ページ引用

公理V 直線外の1点を通ってこの直線に平行な直線はただ一つある。

ある公理系を基本として数学を組み立てるとき、この公理系の公理の間には
たがいに矛盾がないことが必要である。これを公理系の無矛盾性という。
またこの公理系の内容に重複がないということが望ましい。これを公理系の
独立性という。
たとえば、・・・公理Vのみ変えて”直線外の一点を通って、この直線に平行
な直線は無数にある”という公理を採用すると1つの幾何学が組み立てられそ
の間になんら矛盾が起らないことが分かっている。これは公理Vが他の諸公
理と矛盾のないことを示すものである。

この記述を解釈すれば、公理Vが他の公理と矛盾しないのは、公理Vが独立
であるからだということになろうが、三角形の内角の和が180度であるこ
とは公理ではなく、証明可能な定理なので当てはまらないと考えられるのだが。

76 :神の僕 ◆P10fR708T6 :2023/07/14(金) 08:10:33.21 ID:fxUtT6Gd.net
ダブルこりっくすたらこんなことも稀にある。

77 :ご冗談でしょう?名無しさん:2023/07/14(金) 09:04:19.61 ID:???.net
カルト信者がググり貼りしたところで何も理解できる訳がないオカ板に池

78 :神の僕 ◆P10fR708T6 :2023/07/14(金) 09:22:52.92 ID:fxUtT6Gd.net
そういやオカ板で書き込みしたことあったなぁ。
あれは妖怪事典を編纂しようというものだった。
スレタイは忘れた。
サザエさんの飼ってる猫は何十年もたってるから
猫又になってるはずだとか書いてた。
つうか、オカ板は俺の信仰となじまないと思うんだよな。
俺の信仰つうのはもっとリアルな性格のものだから。
まーそれより物理学のほうがなじむと思われる。
ちなみに、ググりではなく、右横に積んでた本をちらと開いて
書いたんだよ、理解できねえなんて福岡のおばちゃんがよく使う
「〇○しきらんと」と同じでいかんなぁ、だから福岡はいつまで
たっても人口が多いだけの街だと言われれるんだよね。
伸びる芽を摘んでしまう、そういうところがあるからなぁ。

79 :ご冗談でしょう?名無しさん:2023/07/14(金) 09:50:11.01 ID:???.net
>>75
>三角形の内角の和が180度であることは公理ではなく、証明可能な定理なので

馬鹿か、平行線公理と2直角(180度)は同値だ
カルト信者がググり貼りしたところで何も理解できる訳がないオカ板に池

80 :ご冗談でしょう?名無しさん:2023/07/14(金) 10:10:52.02 ID:???.net
つまり、ニュートン力学の第一法則(慣性の法則)は仮定(仮説)でしかない
同じく、第二法則(運動方程式)、第三法則(作用反作用の法則)も仮定(仮説)。

基本三法則は互いに論理矛盾が無い、ニュートン力学が物理理論として公認され
日常的な経験・実験事実と一致、義務教育の物理で教えられてる理由だ。

cf. 5ちゃん物理荒らし殆どの俺様説は始めから矛盾だらけの妄想でしかなく即ゴミ箱行き。

81 :神の僕 ◆P10fR708T6 :2023/07/14(金) 10:30:52.11 ID:fxUtT6Gd.net
>>79みたいにすぐ過激な反応するやついるんだぁ、だから板が荒れる。
あー、こういういうことか。
三角形の内角の和が180度であることの証明には一辺に平行な線を頂点から
引くということをやる。その平行線は唯一の直線でなければ錯角とか同位角と
かが使えなくてうまくいかない。証明自体にこのことを使ってるってことだろ。
馬鹿とかオカ板へ池とか、気軽につかうもんじゃあないよ、あんた。
解析幾何だとどうなの?

82 :ご冗談でしょう?名無しさん:2023/07/14(金) 11:11:57.47 ID:???.net
>>81
お前が馬鹿でないと言い張るなら、お前が他スレでまき散らしてる
バイブルモドキのコピペ貼り内容がタダの仮説で真理でも何でもないと
お前が自分から認めることだ。

83 :ご冗談でしょう?名無しさん:2023/07/14(金) 11:32:46.86 ID:???.net
>>66
時空が曲がってる状況ならそもそも慣性の法則が成立してない
慣性の法則はなぜ成り立つ?の質問の前提とは話が変わってる。
話の前提を変えて反論して来られても、だから何?としか言いようがない

84 :ご冗談でしょう?名無しさん:2023/07/14(金) 11:39:42.64 ID:???.net
ただの仮説に成り立つ理由などないと長々と説明してるのが解らんのか

85 :ご冗談でしょう?名無しさん:2023/07/14(金) 11:48:20.42 ID:???.net
カルト宗教のたぐいがタダの仮説にすぎない事を屁理屈であたかも真理のごとく騙すのとは
違うのだよ。

86 :ご冗談でしょう?名無しさん:2023/07/14(金) 12:48:01.28 ID:???.net
宗教バカを相手にするのは無駄

87 :ご冗談でしょう?名無しさん:2023/07/14(金) 13:24:31.60 ID:P/fNUAbz.net
かなり初歩的な流体力学の質問です。
液体が拡大管を通るときの運動量保存則が理解できません。
単位時間あたりの力積が、単位時間あたりの運動量変化に等しいことを用いる時、
瞬間的に速度が変わるわけではなく
拡大管の中を一定の時間をかけて通って、その過程で速度が変化するはずです。
その時間がまったく無視されているように思って悩んでいます。
幼稚な質問でごめんなさい。
よろしくお願いします。

88 :神の僕 ◆P10fR708T6 :2023/07/14(金) 13:52:55.91 ID:fxUtT6Gd.net
>>82
>バイブルモドキのコピペ貼り内容がタダの仮説
例えば?

89 :ご冗談でしょう?名無しさん:2023/07/14(金) 15:20:33.42 ID:P/fNUAbz.net
>>87
抽象的な質問になってしまったので補足させてください。
F が力、p が密度、Q が単位時間当たりの体積流量
拡大菅前後の速度を v、v' とすると
F=pQ(v'-v)
という運動量保存の式があり、
もともとこの式は
F*s=pQ*s(v'-v)の両辺の力積と運動量変化を時間 s で割ったものとして考えると
左辺に、拡大菅を通るのにかかる時間、t が不足しているのではないかと思っています。
F*t*s=pQ*s(v'-v) から
F*t=pQ(v'-v)となるべきだと思うのです。
私が持つ工学系のテキストやネットで調べてもこの t について特に言及していなかったので質問しました。
何度もすみません。よろしくお願いします。

90 :ご冗談でしょう?名無しさん:2023/07/14(金) 16:40:53.02 ID:???.net
>>89
左辺だけにtをかけてしまって、次元は合うのか?

91 :ご冗談でしょう?名無しさん:2023/07/14(金) 17:12:49.24 ID:???.net
素粒子は点(ひも)だから空間時間とは不連続に存在すことになる
なんとなく理不尽、不連続面が発生しないのだろうか

92 :神の僕 ◆P10fR708T6 :2023/07/14(金) 17:26:33.64 ID:fxUtT6Gd.net
>>85
タダの仮説とは公理のことだろう。
ならば、おなじじゃんよ宗教と。

93 :ご冗談でしょう?名無しさん:2023/07/14(金) 18:32:42.10 ID:???.net
反証可能性のある仮説=科学
反証可能性のない仮説=宗教

94 :ご冗談でしょう?名無しさん:2023/07/14(金) 18:37:47.18 ID:???.net
物理学の基本原理(仮説)を発見できるのはごく僅かの天才物理学者だけだ

95 :ご冗談でしょう?名無しさん:2023/07/14(金) 18:47:29.76 ID:???.net
生き残る仮説が少ないから
そう見えるだけ

96 :神の僕 ◆P10fR708T6 :2023/07/14(金) 18:56:07.55 ID:fxUtT6Gd.net
>>93
具体例を!

97 :ご冗談でしょう?名無しさん:2023/07/14(金) 19:00:28.20 ID:???.net
>>96
あらゆる科学、あらゆる宗教が具体例になっている

98 :神の僕 ◆P10fR708T6 :2023/07/14(金) 19:13:29.00 ID:fxUtT6Gd.net
へたなごまかしだよな。
まんどくさいだけだろ?

99 :ご冗談でしょう?名無しさん:2023/07/14(金) 19:27:36.71 ID:???.net
うん、めんどくさいので反証可能性でぐぐってくれ

100 :神の僕 ◆P10fR708T6 :2023/07/14(金) 20:39:24.59 ID:fxUtT6Gd.net
反証可能性が証明になるのか?

101 :ご冗談でしょう?名無しさん:2023/07/14(金) 22:04:10.23 ID:7mQ5p8+d.net
>>89
Qが単位時間当たりの体積流量なのをよく見て

102 :ご冗談でしょう?名無しさん:2023/07/14(金) 22:06:58.62 ID:UJZZee/X.net
反証可能性は好ましいものだが、科学の本質ではない。
健全な科学は反証可能であって欲しいが、別段、科学が健全である必要もないし。

103 :ご冗談でしょう?名無しさん:2023/07/14(金) 22:17:59.83 ID:???.net
流体の運動量保存則は
https://youtube.com/watch?v=f1qXVf2YciU&feature=share7
が分かりやすかった

104 :ご冗談でしょう?名無しさん:2023/07/14(金) 23:24:04.96 ID:???.net
ピカー

105 :神の僕 ◆P10fR708T6 :2023/07/15(土) 05:43:54.98 ID:XV/9xJX/.net
>>102
反証可能なのは意志のない存在に対してのみ言えること。
だから心理学は科学ではなく、疑似科学に留まる。
神は石ではなくて意志のある存在なのであり、人間を超えた
存在であるので反証可能性は適さない。
神が石なら偶像で矛盾する。

106 :神の僕 ◆P10fR708T6 :2023/07/15(土) 05:47:26.33 ID:XV/9xJX/.net
たとえば人間の行動に物理現象ほどの固定した反応を期待できるのか?

107 :ご冗談でしょう?名無しさん:2023/07/15(土) 11:12:21.20 ID:Syk6miY2.net
>>105 >>106
結局のところ、数学も物理も、複雑系を十分に捉えられていないからだ。
人間の心理も行動も複雑系だ。
数学と物理にブレイクスルーがあれば、的確に捉えられることになろう。

神を物理学が考察するなら、まずもって観測できなければならない。

108 :ご冗談でしょう?名無しさん:2023/07/15(土) 16:35:14.02 ID:???.net
レプトンやクォークの質量はヒッグス場のせいだとして、グルーオンや中間子の
異様な重さは何なんだよ?

109 :神の僕 ◆P10fR708T6 :2023/07/15(土) 18:59:07.48 ID:XV/9xJX/.net
>>107
>神を物理学が考察するなら、まずもって観測できなければならない。
己(自我)が観測できないのにもかからわず、観測において己の存在があらゆる存在に先駆けて
存在することを要求されるという事実から、観測ができないということがその存在を否
定することにはならない。あらゆる物理的存在の創造者としての神はむしろ物理的観測
が可能だとしたら、神ではない。それを神とするのは偶像崇拝になる。
だから、デカルトの有名なセリフの延長には神がいるのである。

110 :ご冗談でしょう?名無しさん:2023/07/15(土) 20:01:10.85 ID:???.net
カルト・オカ板からも追い出されたタダの荒らし

111 :ご冗談でしょう?名無しさん:2023/07/15(土) 20:29:57.54 ID:???.net
>>108
グルーオンの重さって初めて聞いたな
中間子がカイラル対称性の破れてのは聞いてる

112 :神の僕#sakanakun:2023/07/15(土) 20:38:17.70 ID:???.net
タダとはなんだ!
とか、怒ったフリをしてみせる。
石川啄木の詩を思い出すぁ・・・・
はたらけど
はたらけど猶わが生活楽にならざり
ぢっと手を見る

物理板で働けど働けど猶わが書き込み
布教にならざり
じっと我慢の子であった
ちゃーーーーん!
大五郎よチキソラーメソは三分間待たんと出来上がらんのじゃ、
じゃによって武士の子ならば石の上にも三千年、母を訪ねて
三千里あのガキの名前はなんつーたのかなー?
とか書けば荒らしなのだよ。

113 :神の僕 ◆P10fR708T6 :2023/07/15(土) 21:01:17.43 ID:XV/9xJX/.net
0006神も仏も名無しさん
2023/07/14(金) 07:59:21.11ID:7JdxMTZJ
おまけ

※新興宗教系の教会で使用する聖書にはない※

旧約聖書・続編「シラ書」第3章より:

17:子よ、何事をなすにも柔和であれ。そうすれば、施しをする人にもまして愛される。
18:偉くなればなるほど、自らへりくだれ。そうすれば、主は喜んで受け入れてくださる。
19:〔身分の高い人や著名な人は多い。しかし、神の奥義は柔和な人に現される。〕
20:主の威光は壮大。主はへりくだる人によってあがめられる。
21:お前の力に余ることを理解しようとするな。また、手に負えないことを探究しようとするな。
22:お前のために定められていること、/それを熟慮せよ。お前に示されていないことを知る必要はない。
23:できないことに手を出すな。お前に示されたことは、/既に人間の理解を超えたものなのだから。
24:多くの者が早合点して道を誤り、/誤った推測で判断をゆがめてしまった。
25:〔目がなければ、光を見ることはできない。知識がないのに、知ったかぶりをするな。〕

聖書は真実だから、真実に従い、俺はカイワレ対称性のことは触れずにおいて
犬とねんねしてから散歩して寝る。

114 :ご冗談でしょう?名無しさん:2023/07/16(日) 00:11:05.72 ID:???.net
馬鹿は無視しとけ

115 :ご冗談でしょう?名無しさん:2023/07/16(日) 00:16:08.06 ID:CNMg54uU.net
聖書は真実ではないよ。
神なんていないのにいると当の最初から嘘をついている。
嘘に嘘を重ねて捏ねたくった大嘘が聖書。

116 :ご冗談でしょう?名無しさん:2023/07/16(日) 00:17:16.99 ID:???.net
「フェイクニュースは数千年前からありました。聖書がいい例です」
                         ユヴァル・ノア・ハラリ

117 :ご冗談でしょう?名無しさん:2023/07/16(日) 14:51:51.38 ID:???.net
聖書はかみくず

118 :ご冗談でしょう?名無しさん:2023/07/16(日) 17:58:45.33 ID:???.net
ピカー

119 :ご冗談でしょう?名無しさん:2023/07/16(日) 18:06:45.79 ID:???.net

聖書
をノイズワードとしてフィルタするだけで一気にゴミが消える

120 :神の僕 ◆P10fR708T6 :2023/07/16(日) 19:00:27.62 ID:dupQYVZ9.net
>>116
>ユヴァル・ノア・ハラリ
この外人のおっさんの話はおまいとおんなじで具体性がまるでない。
だから、よく知らないやつは雰囲気に流されて何となくこのおっさん
に同調してしまう。馬鹿相手の騙しのテクには長けてるな、このおっさーーん。
いるんだよな、こういう偽善者。
他人を貶めることで自分を良く見せようとするやつ。

121 :ご冗談でしょう?名無しさん:2023/07/16(日) 20:41:00.48 ID:AHMz7pS5.net
A君は、角速度1[rad/s]で回転する円板の中心Oにおいた椅子に座っています。
円板の中心から1[m]離れた地点Bで、円板の回転する方向とは逆向きかつ線分OBとは垂直な方向に質量1[kg]の球を円板に対して、1/2[m/s]で転がしたとき、
A君には球は静止して見えました。

これは正しいですか?

122 :ご冗談でしょう?名無しさん:2023/07/16(日) 21:41:36.21 ID:s2q2KH7b0
>>120
おまえがここでちょっとでも具体性のあること言ったことあんの?

123 :ご冗談でしょう?名無しさん:2023/07/16(日) 22:45:31.30 ID:???.net
正しくない
A君にとって円板は静止している

124 :ご冗談でしょう?名無しさん:2023/07/17(月) 01:43:34.63 ID:???.net
>>120
他人を貶めようが持ち上げようが嘘は嘘。

125 :神の僕 ◆P10fR708T6 :2023/07/17(月) 02:15:45.88 ID:JUAJqYq+.net
>>124
だから、ハラリのおっさんは聖書がホラ吹いてるとか言ってるが、
何をどうホラ吹いてるかについて具体的に何一つ述べてないから
駄目だと俺が>>120に書いてるだろ。
このおっさんには聖なる神にハラキリして詫び入れろと言いたい。

126 :ご冗談でしょう?名無しさん:2023/07/17(月) 02:28:36.14 ID:???.net
自発的対称性の破れがよく出るが、根拠はあるのか、妄想なのかよくわからない

127 :ご冗談でしょう?名無しさん:2023/07/17(月) 02:54:24.03 ID:???.net
反証不能な仮説の極みが聖書
聖書はホラ吹き以前に間違ってさえいない

128 :神の僕 ◆P10fR708T6 :2023/07/17(月) 03:13:56.20 ID:JUAJqYq+.net
十戒に「神を試してはならない」があるからね。
だから信仰(信頼と読み替えてもいい)による。
人に置き換えて考えてみると、恋人を試すことは、恋人を信用していない
か、本当に信じるべき人物なら自分と恋人を裏切っていることになる。
聖書は意思のない数学的真理以上に神への愛を要求する。
イエスは神を父と呼んだ。

ヨハネによる福音 3章11節〜
きわめて真実にあなたに言いますが、わたしたちは自分の知っている
ことを話し、自分の見たことについて証しするのですが、あなた方は
わたしたちのする証を受け入れません。わたしが地上の事柄を話して
もあなた方が信じないのであれば、天の事柄を話しても、どうして信
じるでしょうか。

129 :ご冗談でしょう?名無しさん:2023/07/17(月) 07:19:19.09 ID:???.net
アニメ、カルト宗教の基地外は現実世界でも犯罪者になる可能性がある

物理だけの基地外は犯罪者になり難い、自然現象に逆恨みしても無意味

130 :ご冗談でしょう?名無しさん:2023/07/17(月) 10:40:02.04 ID:6m64hTff.net
どなたか物理に詳しい方にお伺いします。
この動画で言われていることは正しいのでしょうか?
https://www.youtube.com/watch?v=QEwDz90lEjg

131 :ご冗談でしょう?名無しさん:2023/07/17(月) 11:39:08.44 ID:hjRdh3tR.net
人間的に劣ったカスが目立つだけでもほかのキリスト者までもが見下げられる原因になる。

132 :ご冗談でしょう?名無しさん:2023/07/17(月) 13:23:34.01 ID:???.net
>>128

>恋人を試すことは、恋人を信用していない か

誰にでも思い違いや間違いはあるんだから、試してみないとわからん。
誰かの無謬性を100%信じるのはキチガイのやることだよ 。

盲目的な信頼など誰のためにもならない。
恋人から試されてもまったく気にかけない寛容さがなければ人として下の下だよ。

133 :ご冗談でしょう?名無しさん:2023/07/17(月) 13:25:02.86 ID:???.net
ここは物理板です
宗教の話は宗教板でやって下さい

134 :ご冗談でしょう?名無しさん:2023/07/17(月) 14:33:08.79 ID:???.net
本場で相手にされないから
無関係なとこで荒らしてるんよ
哀れだね

135 :ご冗談でしょう?名無しさん:2023/07/17(月) 15:23:18.06 ID:???.net
光の速度から電子の電荷量を導けないのでしょうか、または逆でも

136 :ご冗談でしょう?名無しさん:2023/07/17(月) 15:31:23.56 ID:???.net
ピカー

137 :ご冗談でしょう?名無しさん:2023/07/17(月) 17:40:28.39 ID:???.net
万有引力で地球と太陽の距離がかすかに近づいて地球温暖化が進んでいるんじゃないんですか

138 :神の僕 ◆P10fR708T6 :2023/07/17(月) 19:53:49.05 ID:JUAJqYq+.net
>>132
じゃあ、おまいは何について、どう試したんだよ?

>>134
あそこはなぁ、信仰のあるやつは少数派なんだよ、信仰を馬鹿にするための
板だから相手にされないとすれば、俺にはかなわないのでそうしてるだけ。
だが、おめえら知らねえだろうが、本当に霊感のあるやつがいた。
ロゼというコテハンなんだが、ぜったいに他人には知られることのない事柄
を言い当てていた。俺のコテハンはこてはんだった。

139 :神の僕 ◆P10fR708T6 :2023/07/17(月) 20:01:43.13 ID:JUAJqYq+.net
>>133
超物理が宗教なんだよ。
物理がブレークスルーすると宗教に進化するというわけだ。
とどこかに書いてあるから探せ、そうすれば見いだすであろう。

140 :神の僕 ◆P10fR708T6 :2023/07/17(月) 20:24:23.84 ID:JUAJqYq+.net
>>0133ご冗談でしょう?名無しさん2023/07/17(月) 15:23:18.06ID:???
>光の速度から電子の電荷量を導けないのでしょうか、または逆でも
電気学会 電気磁気学 p12
クーロンの法則の定数Kは

K=C。^2×10^(-7)
ただし、C。=真空中の電磁波(だから光)の速度の数値≒3×10^8
したがって K≒9×10^9

とか、電気学会 電気磁気学 p320
C。=1/√(ε。μ。)

あたりを使えばいいんじゃね?

141 :ご冗談でしょう?名無しさん:2023/07/17(月) 22:32:51.79 ID:p7EHc/K7.net
>>90, 100, 102さん
>>89です
ありがとうございます!!理解できました。
次元が合わないことで、おかしいと気づいて
Ft=pQs(v'-v)
でなんでt=sになるんだという疑問から
いくつかyoutubeの動画を視聴して見つけた、
検査体積のうち共通部分と運動量が変わる部分があって
運動量が変わる部分のみの差を出せばよい。
そのときは微小時間とする、という
基本的な部分を理解できました!!!
ありがとうございます。

142 :ご冗談でしょう?名無しさん:2023/07/18(火) 12:26:15.86 ID:???.net
ピカー

143 :ご冗談でしょう?名無しさん:2023/07/18(火) 19:12:05.92 ID:AWqJ+bK3.net
Berry位相に詳しい人おらん?

144 :ご冗談でしょう?名無しさん:2023/07/18(火) 20:37:50.52 ID:???.net
色が見えるのは反射した光を見てるからいうやん
なら人間は常に反射した光を四方から浴びてるはずなのに何でカラフルにならないんだ?
反射率が低いから?
それだと遠くにある林檎の赤とかが認識できるのおかしくない?
100m先の林檎が赤だとわかるのは100m赤が反射してるってこと?
なのに途中で誰かが遮ってもそいつが赤くなったりしない?
おかしくないか?

145 :ご冗談でしょう?名無しさん:2023/07/18(火) 22:47:48.44 ID:???.net
目の中ですべての光が一点に集まってないからだろうw

146 :ご冗談でしょう?名無しさん:2023/07/18(火) 23:14:12.44 ID:???.net
ごちゃ混ぜにする奴に説明は無駄

147 :ご冗談でしょう?名無しさん:2023/07/19(水) 01:01:35.07 ID:???.net
>>146
説明できないんだろ?
視覚の仕組みは実は解明されていない

148 :ご冗談でしょう?名無しさん:2023/07/19(水) 02:15:18.62 ID:cX5EgiVZ.net
>>144
人間の体にそこら中に目があると仮定してみる
それぞれの目に映る景色で赤ばっかりだったり緑ばかりだったりしたら人間の体もカラフルになるかもしれない
でもそれぞれの目に映る景色は実は元の目に映る景色と大差なくいろんな色が含まれた景色である
そうすると人間の体もいろんな色光が混ざった色があたった時に反射する色だいたい白色光が当たった色になる

149 :ご冗談でしょう?名無しさん:2023/07/19(水) 09:33:41.88 ID:???.net
自分が理解できてないだけなのを、世間一般で解明されていない、と飛躍しちゃうやつっているよね

150 :ご冗談でしょう?名無しさん:2023/07/19(水) 11:36:20.33 ID:???.net
基地外の思考・行動が科学的に予測できるようになれば重犯罪を未然に防ぐ効果がある

悪用すると中国のように共産党に従順でない住民を常時AIが監視する社会となるだろう

151 :ご冗談でしょう?名無しさん:2023/07/19(水) 13:24:48.41 ID:???.net
>>147
落ちこぼれは「解明されていない」と言いたがる

152 :ご冗談でしょう?名無しさん:2023/07/19(水) 16:37:31.04 ID:???.net
ピカー

153 :ご冗談でしょう?名無しさん:2023/07/19(水) 22:39:16.18 ID:???.net
ぬるぽ

154 :神の僕 ◆P10fR708T6 :2023/07/19(水) 23:20:44.92 ID:sW9OKJGq.net
ぬるぽと聞いて、何故だかスペースインベーダ−という単語が浮かんだ。
両者には何ら関係は無いにも関わらずにだ。
で、しばしその、ぬるぽという響きがヌルっとすべってつるつるした感じ
を与えるので、UFOの曲面を連想させたのではないかと、またはアタマ
がやたらでかいつるつる肌の宇宙人を連想させたのだろうと思考してみた。
0.3秒後の自覚とよく似た現象を体験したのだな。
落ちこぼれは「解明されてない」と言いたがるという説についてだが、
それは根拠が欠けているので、独断と偏見と言われても仕方ない。
想像するに、無知なために解明されている事実を知らないからだ、とか、
考えることを他人に任せて自分からは解明しようとしない、できないとか
だろう。まあ、りょうほうなのか、どっちかなのか、どちらでもないのか
分からんが、5ちゃんの住人は書き方が乱暴でいかんな。

155 :ご冗談でしょう?名無しさん:2023/07/20(木) 05:05:06.15 ID:???.net
野球のバッティングで、体重が違ってもバッティング速度が同じなら打球速度も同じになるのはなんで?
剛体じゃないから?

156 :ご冗談でしょう?名無しさん:2023/07/20(木) 05:15:55.20 ID:7dnYARhF.net
大量の人間が生活するための電気製品にはどのようなものがありますか?
物理的な危険はどのように排除されていきますか

157 :ご冗談でしょう?名無しさん:2023/07/20(木) 05:52:31.04 ID:???.net
ピカー

158 :ご冗談でしょう?名無しさん:2023/07/20(木) 10:09:23.16 ID:???.net
>>155
ボールからの反力が最大静止摩擦を超えるほどにならないから。
摩擦のない地面でボールを打てば打者の体重が効いてくるだろうな

159 :ご冗談でしょう?名無しさん:2023/07/20(木) 10:14:40.01 ID:Lw/JjOOM.net
物理向けのベクトル解析の本で、曲線上の積分、曲面上の積分はやるのに、3次元曲面上の積分をやらないのはなぜですか?

160 :ご冗談でしょう?名無しさん:2023/07/20(木) 10:33:46.97 ID:dcD+d/ie.net
自分でやれってことじゃない?

161 :ご冗談でしょう?名無しさん:2023/07/20(木) 11:05:00.03 ID:???.net
>>158
腕の重さとかは反発に関係ないの?
腕が重いほうが運動エネルギー多そうだから飛んでいきそうだけど

162 :ご冗談でしょう?名無しさん:2023/07/20(木) 12:00:45.84 ID:Lw/JjOOM.net
>>159

4次元空間内にある3次元曲面がイメージできないからですか?

163 :ご冗談でしょう?名無しさん:2023/07/20(木) 12:07:08.15 ID:???.net
>物理向けのベクトル解析

3次元のユークリッド空間が前提だから

164 :ご冗談でしょう?名無しさん:2023/07/20(木) 12:14:46.33 ID:???.net
地球のスケールで曲がった時空間は殆ど観測不能、一般相対論の複雑な数学は必要ない。

165 :ご冗談でしょう?名無しさん:2023/07/20(木) 13:01:09.39 ID:???.net
>>159
微分形式まで扱ってるベクトル解析だと普通に一般次元での議論もやってるのでは

166 :ご冗談でしょう?名無しさん:2023/07/20(木) 13:29:34.37 ID:???.net
>>159
そもそも3次元曲面て何?
3次元超曲面か?

167 :ご冗談でしょう?名無しさん:2023/07/20(木) 14:11:50.62 ID:???.net
3次元曲面は3次元曲面だろw

168 :ご冗談でしょう?名無しさん:2023/07/20(木) 15:32:27.09 ID:???.net
ムーニーちゃんしんぷだい

169 :ご冗談でしょう?名無しさん:2023/07/20(木) 18:16:15.14 ID:Lw/JjOOM.net
>>166
3次元超曲面です。

2変数のテイラーの定理について質問です。
f(x + h, y + k) = f(x, y) + (∂/∂x) f(x, y) ✕ h + (∂/∂y) f(x, y) ✕ k + (1/2) ✕ (∂^2/∂x^2) f(x, y) ✕ h^2 + (∂^2/∂x∂y) f(x, y) ✕ h ✕ k + (1/2) ✕ (∂^2/∂y^2) f(x, y) ✕ k^2 + O(|h|^3 + |k|^3)
が成り立つと書いてあります。
誤差項がO(|h|^3 + |k|^3)と書けることはどう証明しますか?
h^2 ✕ kやh ✕ k^2はどうやって評価しますか?

170 :ご冗談でしょう?名無しさん:2023/07/20(木) 19:02:41.62 ID:new+JpuO.net
ピカーって、ッついてなかった?

171 :ご冗談でしょう?名無しさん:2023/07/20(木) 19:04:26.23 ID:new+JpuO.net
4次元超球面

172 :ご冗談でしょう?名無しさん:2023/07/20(木) 19:27:45.63 ID:Lw/JjOOM.net
i + j ≦ nのときに、|h|^i ✕ |k|^j = O(|h|^n + |k|^n)であることの証明。
|h| ≦ |k| のとき、|h|^i ✕ |k|^j ≦ |k|^{i+j} ≦ 1 ✕ (|h|^n + |k|^n)
|k| ≦ |h| のとき、|h|^i ✕ |k|^j ≦ |h|^{i+j} ≦ 1 ✕ (|h|^n + |k|^n)
よって、|h|^i ✕ |k|^j ≦ 1 ✕ (|h|^n + |k|^n)
よって、|h|^i ✕ |k|^j = O(|h|^n + |k|^n)

173 :ご冗談でしょう?名無しさん:2023/07/20(木) 20:58:14.13 ID:???.net
>>169
1変数のテイラーの定理を各変数毎に適用する

174 :ご冗談でしょう?名無しさん:2023/07/21(金) 14:05:56.44 ID:GvvJiWaX.net
流体の運動量保存則のみを使った圧力差の式で
p1-p2=d(v1^2 - v1v2) (dは密度 pは圧力 vは速度
この式は一般性の高い式であるはずなのに、広がり管の圧力差に用いることができないことが腑に落ちません。
これだけ見れば、速度によってのみ、圧力差が決まるということになるはずです。
数学的な導出として、エネルギーの式から
p1-p2=d(v1^2 - v1v2)+d/2 (k-1)(v2 - v1)^2 (kは広がり管の損失係数)
を導出することはできるのですが、物理的な理解?ができていません。
とりあえず数式的には損失係数が1以外(というか未満)なら、
運動量のみによる圧力差の式は使えないという認識でよろしいでしょうか?
他に損失係数に関わらない場面でも、単純に運動量保存のみを用いると不適切な場面はあるでしょうか?
抽象的な質問で申し訳ありません。
よろしくお願いします。

175 :ご冗談でしょう?名無しさん:2023/07/21(金) 19:37:07.54 ID:???.net
>>174
キミの説より、物理学ではエネルギーの式が優先するだけ

前提が非圧縮流体(d一定)で完全流体(k=0)のエネルギー保存法則から
ベルヌーイの定理が導出される。(基本)

176 :ご冗談でしょう?名無しさん:2023/07/22(土) 06:58:13.33 ID:???.net
219 それでも動く名無し 2023/07/17(月) 20:26:51.53 ID:pA5+SQtP0
すっげぇかわいいのにおっぱいも綺麗で大きいし、尻もエロい
https://i.imgur.com/62eA5KE.jpg
https://i.imgur.com/St2P7wG.jpg
https://is.gd/xkUfeT

620 名無しさん@ピンキー sage 2023/07/17(月) 17:36:57.85 ID:AS4vmq4R0
不朽の名作が復活していたので
https://i.imgur.com/C239AFO.jpg
https://i.imgur.com/l35c3gZ.jpg
https://is.gd/Q5ZYzk

177 :ご冗談でしょう?名無しさん:2023/07/22(土) 06:58:16.86 ID:???.net
すみません誤爆しました

178 :ご冗談でしょう?名無しさん:2023/07/22(土) 13:18:37.45 ID:???.net
しんぷさんもムーニーちゃん プー

179 :ご冗談でしょう?名無しさん:2023/07/22(土) 14:29:11.09 ID:l/mZdOrY.net
水圧が重力方向に働くのは分かりますが、なぜ水平方向にも働くのでしょうか?

180 :ご冗談でしょう?名無しさん:2023/07/22(土) 14:33:15.74 ID:???.net
流体だから

181 :ご冗談でしょう?名無しさん:2023/07/22(土) 14:37:50.63 ID:???.net
>>174
これでも読めば?
https://vis-tech.site/運動量保存の法則/

182 :ご冗談でしょう?名無しさん:2023/07/22(土) 14:47:41.48 ID:l/mZdOrY.net
>>180
説明になっていませんね。

183 :ご冗談でしょう?名無しさん:2023/07/22(土) 18:01:44.55 ID:Cfm0VLK6.net
カルトを丁寧にしてオカルト
で思い出したんだが
マソコにオを付けてオマソコ
なのにチソコにオをつけてオチソコ
と言わずに、オチンチンというのは
何でですか?

184 :poem:2023/07/22(土) 19:19:35.15 ID:zHBCjXuR.net
活用(性的)(語尾)がちがうんじゃない?

185 :poem:2023/07/22(土) 19:21:48.61 ID:zHBCjXuR.net
性器の活用方法が違うから
ちんこ→から→ちんちん
になる
同じ活用方法すれば
おまんこ→おまんまん
になる

次の質問
ちんこ、ちんぽ、ちんちん
まんこ、まんぽ、まんまん
の性器使用的活用方法の違いはなんですか

186 :poem:2023/07/22(土) 19:23:26.18 ID:zHBCjXuR.net
たまたま
も活用が違うと
たまこ(ろがし)
になるし

187 :ご冗談でしょう?名無しさん:2023/07/22(土) 19:28:26.23 ID:???.net
山口人生とビルゲイツはどっちのほうがすごいですか?

188 :poem:2023/07/22(土) 19:38:21.30 ID:zHBCjXuR.net
>>155
体重違っても
ボールの変形が飛ぶ距離に比例
体重違かろうが
体重重い人が体重移動せずに打つより
体重軽い人が体重移動物凄くして打った方が
ボールの変形が大きいから飛ぶと思う

189 :ご冗談でしょう?名無しさん:2023/07/22(土) 19:59:36.57 ID:l/mZdOrY.net
電気力線を考えるのはなぜですか?
電場で必要十分じゃないんですか?

190 :ご冗談でしょう?名無しさん:2023/07/22(土) 21:22:28.18 ID:ltqcWMlw.net
この時空での時間の進み方は、連続なのか、ものすごく微小なところでは離散的なのか、どちらでしょうか?

191 :poem:2023/07/22(土) 21:31:36.74 ID:zHBCjXuR.net
>>190
もし時間が連続で0+0+0+0…=0
で時間0がいくら足されても時間が経過しないなら
現象素や変化素のような時間最小単位は実在するとなるけど
この現象素や変化素の大きさは多分大きさマチマチで色んな大きさがあるんじゃない?
ようは宇宙の仕組み的に時間の最小単位は無く連続で時間0は実在するけど
時間経過する場合マチマチな時間大きさがあるって
時間経過は最小単位あり
時間物理は最小単位なし

192 :poem:2023/07/22(土) 21:33:50.66 ID:zHBCjXuR.net
空間も同じ説

193 :poem:2023/07/22(土) 21:35:02.87 ID:zHBCjXuR.net
ようは長さ=空間経過とする
空間0=経過無しも実在し
空間も連続

194 :ご冗談でしょう?名無しさん:2023/07/22(土) 22:12:44.84 ID:l/mZdOrY.net
電磁気学にガウスの法則というものがあります。
積分に関する数学的な定理であるにもかかわらず、「法則」と呼ばれています。
なぜでしょうか?

195 :神の僕 ◆P10fR708T6 :2023/07/23(日) 01:08:47.68 ID:nrvdBh+2.net
電気学会の大学講座電気磁気学では32ページに「ガウスの定理」と書いてある。
逆2乗法則が成立するのは公理でそこから導かれたのがガウスの定理。
数学事典から引用すると

公理または既にその正しいことの分かった命題をもとにして、新しい命題の真で
ある理由を証明することを証明するといい、証明された命題の内それ自身重要な
ものや、後の証明にしばしば用いられるものを定理という。

196 :ご冗談でしょう?名無しさん:2023/07/23(日) 17:43:45.73 ID:???.net
高校物理の履修率ってどこにデータある?

197 :poem:2023/07/23(日) 17:57:28.22 ID:/Dptmb+j.net
高校大学の選択科目の履修率…
ビッグデータになってるの?
なってるなら自分も知りたい

198 :poem:2023/07/23(日) 21:15:06.04 ID:/Dptmb+j.net
0と極限の無限小1/∞は異なるって自分もこれまで思ってたけど、逆に同じだって仮定した場合
1/2−1/4−1/8−1/16…の無限小1/∞と
1/3−1/9−1/27−1/81…の無限小1/∞は
どちらも空間経過がない空間0で
どちらもこの空間から空間0に見えて
無限小=0だけど
空間経過が0のこの宇宙の下の、この宇宙上で点の宇宙上で二つを見たら、大きさが異なるんじゃない?
ようは
本当の空間0(どの宇宙段階でも空間0)と無限小(どの段階かで大きさを持つ)は大きさを持つ下段階宇宙上で0≠、だけど、この宇宙から見たら0=、なんじゃ?とちょっとふと思った

199 :poem:2023/07/23(日) 21:17:46.08 ID:/Dptmb+j.net
発散の無限大も
この宇宙では全部=∞見た目変わらないけど
上の宇宙では大きさに違いがあるんじゃない?
1+1+…の∞と
1+2+…の∞と

200 :poem:2023/07/23(日) 21:20:27.06 ID:/Dptmb+j.net
ようするに
0.0000…=0
かつ
下のどこかの段階では
真の0≠0.0000…
両方真
ではと

201 :poem:2023/07/23(日) 21:22:47.02 ID:/Dptmb+j.net
ようするに
宇宙空間、宇宙時間に
容れ物のセクターは無い
この宇宙の粒子や現象に
内容物のセクターがある


202 :poem:2023/07/23(日) 21:27:23.08 ID:/Dptmb+j.net
空間は正…セクター外(全)(分子)
時間は負…セクター内(瞬)(分母)

別の場所行けないのは分母
別の場所行けるのは分子


203 :poem:2023/07/23(日) 21:28:40.92 ID:/Dptmb+j.net
次の質問(再)
ちんこ、ちんぽ、ちんちん
まんこ、まんぽ、まんまん
の性器使用的活用方法の違いはなんですか

204 :poem:2023/07/23(日) 21:39:14.73 ID:/Dptmb+j.net
>>156 家電
安全は事故対策製品基準と注意書きを守る
電気は攻撃力あるから危ない
攻撃力を防ぐことが安全
高圧電線に触れたら死ぬ
家電程度の電圧製品なら安全製品対策はできるが
高圧電気を使う家電があれば、全人類が同時に使えば人類は絶滅する
高圧電気家電、安全対策がもしできても、人類の何割かはち守れず死ぬ

205 :poem:2023/07/23(日) 21:39:48.37 ID:/Dptmb+j.net
最後の行

人類の何割かは注意書きを守れず死ぬ

206 :ご冗談でしょう?名無しさん:2023/07/23(日) 21:42:11.89 ID:???.net
またゴミだらけ

207 :ご冗談でしょう?名無しさん:2023/07/23(日) 22:00:08.63 ID:zFbjgqgo.net
無限に関わることは人間の直感を裏切るから、
よほど論理を吟味して考えないと間違いがち。

208 :神の僕 ◆P10fR708T6 :2023/07/24(月) 01:18:18.74 ID:jHfoBma6.net
>>203
まんぽ とは言わない。
性器的活用は主に生殖であるが、風俗営業での活用は全く異なる。
チンポは小さいまだ、ガキのそれを思わせるから、可能性としては
あるが、まだ使われていないだけなのかも知れない。
ただ、女性の場合、成長と共に正面からでは見えなくなり、毛か
つるつるてんになるので意識から遠のく。
古代ギリシャ彫刻においてはつるつるてんである事からわかるように、
美的デリカシーから言って、また児童ポルノ禁止法の制限からも
「何もない」という、一種のカーテンを掛けていたほうがいいのである。

209 :ご冗談でしょう?名無しさん:2023/07/24(月) 03:00:54.00 ID:lH+HI1W7.net
物理というか理系かすら解らんけど
「最初にそれを考え出すのが難しい」みたいな言葉で
コロンブスの卵 じゃないやつ誰か解りません?
何とか の なにか  

コロンブスが有名すぎてググっても全然出てこない
お願いします

210 :ご冗談でしょう?名無しさん:2023/07/24(月) 03:24:42.02 ID:lH+HI1W7.net
思い出した ありがとう

211 :ご冗談でしょう?名無しさん:2023/07/24(月) 12:24:20.52 ID:???.net
ピカー

212 :poem:2023/07/24(月) 12:40:43.39 ID:kx50AdTZ.net
暇だねー
屋外プールで肩とか背中とか治そう!

213 :ご冗談でしょう?名無しさん:2023/07/24(月) 13:15:00.70 ID:???.net
近眼なんだけどなんで鏡に写った景色はメガネをかけないとボケて見えるの?
鏡は目の前20センチにあるのに

214 :ご冗談でしょう?名無しさん:2023/07/24(月) 13:19:34.20 ID:???.net
>>209
棚からぼた餅でなくて?

215 :ご冗談でしょう?名無しさん:2023/07/24(月) 14:02:46.42 ID:???.net
>>213
2次元の写真画像でないから、光学的に反転した3次元を映すのが鏡

216 :poem:2023/07/24(月) 15:22:00.85 ID:kx50AdTZ.net
写真の光は紙の一点から1つの像の1点の色の光が出てるけど
鏡の光は鏡の一定範囲面から1つの像の1点の色の光が出てるから

217 :poem:2023/07/24(月) 15:23:39.10 ID:kx50AdTZ.net
これさ
鏡は映ってる景色の光が、重ね合わせ
量子論なんだよね

218 :poem:2023/07/24(月) 15:29:55.53 ID:kx50AdTZ.net
光反射する原子(銀など)を基盤に配置して
原子にあらゆる情報を重ね合わせに入力して
原子の状態を基盤が読み取れれば
量子コンピュータの新たな方式作れる可能性

219 :poem:2023/07/24(月) 15:32:39.50 ID:kx50AdTZ.net
あ、
銀配置した基盤の銀の状態を基盤が読み取れれば
銀配置基盤面に、光学レンズ無しで、景色光入射して
スクリーンのみでカメラ作れる

220 :poem:2023/07/24(月) 15:33:36.92 ID:kx50AdTZ.net
しかもレンズ依存しないから
3D映像カメラ

221 :ご冗談でしょう?名無しさん:2023/07/24(月) 18:59:56.50 ID:QtaY6DoD.net
>>175 >>181
ありがとうございます。
そもそもエネルギー保存則というか、ベルヌーイの定理に従うというのは当然だったと今は思います。
以前流体力学について疑問に思ったことも含めて
リンクのサイトによく記載されていたので、勉強します。
ありがとうございました。

222 :ご冗談でしょう?名無しさん:2023/07/24(月) 20:51:29.15 ID:???.net
古典力学で、質量m、位置x、速度v、運動エネルギーEである質点Aに関して、
-dE/dx = k / v^2
(kは定数で、初期条件は時刻t=0においてx=0, E=E0 (>0))
という関係式があるとき、-dE/dxをvを含まない形で、xを用いて表すことは可能ですか?

イメージ的にはxが大きくなるに連れて-dE/dxが急激に大きくなると予想しますが、微分方程式が複雑でした。

223 :ご冗談でしょう?名無しさん:2023/07/25(火) 09:41:25.43 ID:???.net
v^2とxの変数分離型で積分も簡単

224 :ご冗談でしょう?名無しさん:2023/07/25(火) 13:49:41.00 ID:???.net
E=(1/2)mv^2としていいのならvを消去できる

225 :ご冗談でしょう?名無しさん:2023/07/26(水) 02:55:08.88 ID:???.net
しんぷさんはムーニーちゃんダンスをおどれ

226 :ご冗談でしょう?名無しさん:2023/07/26(水) 12:56:18.53 ID:???.net
哀れやな

227 :ご冗談でしょう?名無しさん:2023/07/26(水) 16:51:11.11 ID:iM142Hp+.net
地球温暖化

228 :ご冗談でしょう?名無しさん:2023/07/26(水) 17:06:39.02 ID:iM142Hp+.net
地球温暖化二酸化炭素原因説は本当かな?

自分が思うに、地球の温度を決める要素は太陽光と地球の持つ熱しかないと思うけど、地球が発熱するのがそんなに変化するとは思えないから、太陽光しか温度を決める要素ないと思う。

だから、地球の温度は太陽光の地球へのエネルギー収支で決まるとして、太陽光の変動が原因じゃないかと思うんだけどな。

どうやって太陽光を計測できるかわからない。あらゆる波長をもれなくちゃんと吸収して放射まで考慮された計測って可能なの?

229 :ご冗談でしょう?名無しさん:2023/07/26(水) 22:05:53.69 ID:dJR5Gxuu.net
://i.imgur.com/DvOr3zU.jpg
(1)ビオサバールの法則を使う問題だと思うのですが誘導にうまくのれません、角度φを誘導では使って欲しそうな気がするのですが自分は角度θを設定して解く方法しかわからないです。申し訳ないのですがどなたか解き方を教えていただけないでしょうか?
(2)変位電流を求める問題なのですが電束密度を求めようにもコンデンサーでもないので求め方がわからないです、、どなたか解き方を教えていただけないでしょうか?
どうかよろしくお願いしますm(__)m

230 :ご冗談でしょう?名無しさん:2023/07/26(水) 22:06:41.31 ID:dJR5Gxuu.net
://i.imgur.com/DvOr3zU.jpg
(1)ビオサバールの法則を使う問題だと思うのですが誘導にうまくのれません、角度φを誘導では使って欲しそうな気がするのですが自分は角度θを設定して解く方法しかわからないです。申し訳ないのですがどなたか解き方を教えていただけないでしょうか?
(2)変位電流を求める問題なのですが電束密度を求めようにもコンデンサーでもないので求め方がわからないです、、どなたか解き方を教えていただけないでしょうか?
どうかよろしくお願いしますm(__)m

231 :ご冗談でしょう?名無しさん:2023/07/26(水) 22:41:53.38 ID:dJR5Gxuu.net
://i.imgur.com/GGVGK6d.jpg
何度も質問して申し訳ありません、この問題の(2)で回路に流れる電流を求めるのですが回路が移動する速さがあたえられていないのにどうやって回路に生じる起電力を求めればよいのでしょうか、
(1)の途中でdφをもとめてるからdφ/dtを求めるのかと思ったのですが、時間で微分しようにも時間を含む項が無いのにどうやって微分するんだろうと思いました
どなたかご存知の方いらっしゃいましたら教えていただけないでしょうか

232 :ご冗談でしょう?名無しさん:2023/07/26(水) 23:39:14.62 ID:YcPN8/NP.net
月の潮汐力で地球の重力の等位曲面が変わったら、海流も影響受けるよね?ところで潮汐力のする仕事の分は月は何のエネルギー失うのだろうか?

233 :ご冗談でしょう?名無しさん:2023/07/27(木) 09:34:51.42 ID:???.net
地球-月の潮汐作用では、月はエネルギーを失うどころが実は増えている。
減っているのは地球の自転エネルギー

234 :ご冗談でしょう?名無しさん:2023/07/27(木) 11:15:54.41 ID:???.net
ピカー

235 :ご冗談でしょう?名無しさん:2023/07/27(木) 12:29:33.64 ID:???.net
>>228
説明くらい読めよ

236 :ご冗談でしょう?名無しさん:2023/07/27(木) 12:29:34.56 ID:yPKGpJAS.net
>>233
月が作用した結果なのに月がエネルギー失わず地球が自転で運動エネルギー落とすって変だね。

237 :ご冗談でしょう?名無しさん:2023/07/27(木) 12:30:27.09 ID:???.net
それが相互作用

238 :ご冗談でしょう?名無しさん:2023/07/27(木) 12:34:47.60 ID:???.net
>>229
点Pてのがどこにあるかすら分からん
本気で聞いてんの?

239 :ご冗談でしょう?名無しさん:2023/07/27(木) 18:42:51.35 ID:???.net
>>236
地球の自転が徐々に遅くなり、月は徐々に地球から遠ざかっていると聞いたことはないですか?それが潮汐相互作用の結果なんですよ

地球の自転が徐々に遅くなる=地球の自転エネルギーは減少
月が徐々に遠ざかる=月の公転エネルギーは増大

変と言われても観測事実ですから

240 :ご冗談でしょう?名無しさん:2023/07/27(木) 21:53:17.00 ID:j6t1p3GY.net
>>239
角速度変わらず、公転半径が大きくなってるの?信じられないな。

241 :ご冗談でしょう?名無しさん:2023/07/28(金) 00:06:27.86 ID:???.net
角速度変わらず、なんて誰か言ったか?

242 :ご冗談でしょう?名無しさん:2023/07/28(金) 05:54:48.88 ID:H/db5bm5.net
光速のロケットで地球を周回したら地上の時間は進んで未来に行けるらしいけど
地上と交信しながらとか地上にカメラ置いてスカイツリーを建設する工程を
リアルタイムでロケットの中で見てたら早回しで見えるんですか?

243 :ご冗談でしょう?名無しさん:2023/07/28(金) 10:58:36.66 ID:???.net
>>242
細かな事考えなければ、そんな感じ

244 :ご冗談でしょう?名無しさん:2023/07/28(金) 19:30:09.31 ID:???.net
ピカー

245 :ご冗談でしょう?名無しさん:2023/07/28(金) 20:11:52.65 ID:???.net
地球上における自由落下速度について
ざっくり言えば重力で加速していく速度とその二乗で増す空気抵抗が均衡して等速度運動になる、終端速度というのがあるのはわかります

それに関してとある掲示板で
「空気抵抗を受ける物体の自由落下の最高速度は200km/hを大幅に超過することはなく、どのような物体でも一定の速度に収束する」
と主張する人がいたんですがこれは本当なんでしょうか

高密度で空気抵抗にも優れた形状の重量物なら200km/hどころではなくもっとずっと出るのではと思うのですが
落下物の質量や形状にかかわらず終端速度にそのような限界は存在するのですか?

検索してもソースは見当たらないし、スカイダイビングも手足を広げてブレーキをかければ200km/h以下だが
頭や足から真っ直ぐ落ちる姿勢だと300km/hに達すると書いてあったりするし……

ちなみに当のスレッドは消されてもうありません

246 :ご冗談でしょう?名無しさん:2023/07/28(金) 20:18:40.16 ID:???.net
音波というのは地球のどこまで届きますか
電気を帯びた音波はどうですか

247 :ご冗談でしょう?名無しさん:2023/07/28(金) 20:19:00.36 ID:???.net
>>245
ウソだから忘れとけ

248 :ご冗談でしょう?名無しさん:2023/07/28(金) 20:28:38.62 ID:???.net
>>245
>スカイダイビングも手足を広げてブレーキをかければ200km/h以下だが
>頭や足から真っ直ぐ落ちる姿勢だと300km/hに達すると書いてあったりするし……

これは信じずに、誰が書いたかも確認できない掲示板の書き込みは信じる、その頭の構造が不思議だわ

249 :ご冗談でしょう?名無しさん:2023/07/28(金) 20:38:18.72 ID:???.net
>>246
1年半前のトンガの火山噴火の音波(圧力変動)は、少なくとも地球を1周以上したことが確認されている

250 :ご冗談でしょう?名無しさん:2023/07/28(金) 20:54:57.22 ID:???.net
>>247
まあ、そうだよねやっぱり

>>248
俺自身はその(200km/h論)主張に懐疑的でそういう書き方をしたつもりだったけど…

その言い方だとここで説明してもらっても信じちゃダメということにならんか
よく知らないサイトの情報だって裏とる手段がなきゃ似たようなもんだし

251 :ご冗談でしょう?名無しさん:2023/07/28(金) 21:12:50.79 ID:???.net
ChatGPTにでもきけば
素数も理解できないほどバカだと噂だけど

252 :ご冗談でしょう?名無しさん:2023/07/28(金) 21:37:21.67 ID:???.net
>>250
科学は誰かを信じる信じないじゃねーんだよ
運動方程式理解してれば人に聞くまでもなく何が正しいかは分かる

253 :ご冗談でしょう?名無しさん:2023/07/28(金) 23:27:20.32 ID:???.net
>>249
成る程、選挙カーも相当飛びそうですね

254 :ご冗談でしょう?名無しさん:2023/07/29(土) 00:15:41.14 ID:???.net
ムーニーちゃんしんぷのおじさんはすごいんだぞ

255 :ご冗談でしょう?名無しさん:2023/07/29(土) 08:43:38.91 ID:???.net
なかよしおむつムーニーマン

256 :poem:2023/07/29(土) 10:06:29.87 ID:lK4yiZyq.net
神父おじさんみたいだよ

257 :poem:2023/07/29(土) 10:08:48.94 ID:lK4yiZyq.net
デボンシャイヤー

デボンは寺院に機械
シャイヤーは?

258 :poem:2023/07/29(土) 10:09:59.61 ID:lK4yiZyq.net
別人じゃなければ

259 :poem:2023/07/29(土) 10:12:16.99 ID:lK4yiZyq.net
ん?
仲良しおむつ
って誰か書いたから
むに神父おじさんに誹謗かなと思って
フォローしたけど
実は
仲良しおむつ
は本人だった?

260 :poem:2023/07/29(土) 10:13:51.19 ID:lK4yiZyq.net
本人だったならフォローいらなかった
仲良しおむつも副業ワードに加えたなら
副業じゃましてごめんね

261 :poem:2023/07/29(土) 10:16:07.77 ID:lK4yiZyq.net
誹謗かと思って紛らわしかった

262 :ご冗談でしょう?名無しさん:2023/07/29(土) 11:05:08.47 ID:???.net
>>252
ガリレイ、ニュートン、アインシュタインが自由落下の法則を理論的に独立に提唱した。
実際に任意の物体を地上で落下させと同時に地面に落ちないのは誰でも知っている
偉大な物理学者達が間違ってたのだろうか?
もしそうならば、慣性の法則についても同様な間違いになるだろう。

実験と違うから間違っていると単純に判断する人は、「物理学の理論」とは何か
を理解できない人なのである。
つまり
ガリレイ、ニュートン、アインシュタインは雑な実験の裏に隠された宇宙の基本法則を
見抜く天才的な知能を持つ人だといことだ。
さらに
偉大なニュートンは流体中の物体実験から、運動速度に比例する抵抗力を発見した
「重力と抵抗力の理論」から終端速度があることが導出できる。
これで終わりではない、「流体力学の理論」では速度の2乗に比例する抵抗力等により
さらに複雑な現実の流体運動を理論から計算できるのである。

馬鹿キチガイ荒らしにはそれすら全く分からない脳ミソだということ

263 :ご冗談でしょう?名無しさん:2023/07/29(土) 12:49:19.22 ID:???.net
こいつ何を言ってんの?

264 :ご冗談でしょう?名無しさん:2023/07/29(土) 13:18:20.65 ID:???.net
ムーニー(メコス)荒らしに分かるわけがない

265 :poem:2023/07/29(土) 16:14:33.80 ID:V6MALitY.net
>263-264 はよくわからないけど
メコスの人は別の人かと思ってた

266 :poem:2023/07/29(土) 16:16:57.69 ID:V6MALitY.net
>>262
飛行機の飛ぶ原理解明できてないじゃん

流体抵抗が速度の二乗に比例するの初めて知った
だけどそれに加えて飛行機の翼の揚力は翼長の何乗かに比例するでしょ?
これ解明できてないじゃん

267 :poem:2023/07/29(土) 16:19:43.42 ID:V6MALitY.net
さらにテコによる重さの遠のき
ようは祭の山車を2人でロープを引っ張るより
紐に直角に長い金属棒つけて遠い両端をひっぱったほうが少ない力で引ける
これが何乗に比例するか
テコの力学も完全解明できてないじゃん
これが飛行機の秘密の1つ

268 :ご冗談でしょう?名無しさん:2023/07/29(土) 16:22:53.54 ID:???.net
物理学で揚力が解明できない??? なら毎日飛行機が飛ぼ回っているわけねーだろ

キチガイのお前には無理なだけ

269 :poem:2023/07/29(土) 16:24:19.50 ID:V6MALitY.net
まあ、考えたけど自分無理だった
言い当ててるとおり

270 :ご冗談でしょう?名無しさん:2023/07/29(土) 17:47:14.26 ID:???.net
無意味なゴミ

271 :ご冗談でしょう?名無しさん:2023/07/29(土) 22:55:41.92 ID:???.net
荒らしに反応するヤツも荒らし

272 :ご冗談でしょう?名無しさん:2023/07/30(日) 00:36:04.64 ID:???.net
そもそも荒らしだけしかいない状況で誰に言ってんだ

273 :ご冗談でしょう?名無しさん:2023/07/30(日) 08:43:00.60 ID:???.net
ダンシングムーニー

274 :ご冗談でしょう?名無しさん:2023/07/30(日) 13:23:56.49 ID:???.net
哀れやな

275 :ご冗談でしょう?名無しさん:2023/07/30(日) 18:49:20.49 ID:rI0xQDJJ.net
>>268
何でも理論が先行して発展してるわけじゃないぞ。

276 :ご冗談でしょう?名無しさん:2023/07/30(日) 19:09:19.54 ID:rI0xQDJJ.net
>>245
空気が与える圧力が重さに等しくなれば、加速は止まるだろうけど。したがって、密度の高い物質にすればするほど、同じ形状でも加速が止まる速度は大きくなるはず。

ある形状の物体が200kmで加速が止まるなら、その倍の密度ならば400km、3倍なら600kmとなるはず。

人間と同じ形でも鉄で構成されてれば、論理的には7から8倍の速度になるはず。形状による影響は難しいので考えないw

277 :ご冗談でしょう?名無しさん:2023/07/30(日) 20:33:41.52 ID:???.net
>>276
要するに抵抗力の大きさは速度に比例するんだと思い込んでそういう計算してるのきみは?

278 :ご冗談でしょう?名無しさん:2023/07/30(日) 20:49:23.14 ID:rI0xQDJJ.net
>>277
いや、加速が止まるってことに着目して、バネに吊した重りが0になるような状況に置き換えて考えてみた。

風の運動量が全部重りに移るとして、この空気の風速を二倍三倍としていけばどこかで0になるから。下から上向きに風を出してさ。

落下による風の相対速度とそれを同じとみなして。単純化だけど。でも考えてみると、運動量を失った空気は物体と同じ運動するはずだから、圧縮を避けられないという気がしたところ。

279 :ご冗談でしょう?名無しさん:2023/07/30(日) 21:15:31.34 ID:rI0xQDJJ.net
物体の運動方向への空気の総圧力が重さと釣り合うとき、加速が止まる。抵抗というのは本当はこうなんじゃないかな。

だから、物体の形状が対称性を持たないような場合は物体の回転によって加わる総圧力が変化し、落下速度が変化する。それが飛行機の墜落とということかなと。

280 :ご冗談でしょう?名無しさん:2023/07/30(日) 21:40:29.06 ID:rI0xQDJJ.net
だから最初の、運動量の全移転が起こるという仮定は否定されて非弾性衝突はなくて、弾性衝突をして運動量を失わず、圧縮しないといけない。このようになるはず。だから、流体力学的な説明って正しいんだろな。

理論的な考察、質点の力学からそのくらいは言えると思える。形状とか、粘性とか、難しいしw

281 :ご冗談でしょう?名無しさん:2023/07/31(月) 01:26:38.83 ID:vVwXAdIB.net
野球のメジャーリーグの玉とかも、回転しやすいように工夫してあるのかもね。日本のプロ野球では見ないような曲がり方するから。流体力学の専門家が設計してるのかも。

282 :ご冗談でしょう?名無しさん:2023/07/31(月) 12:47:11.37 ID:???.net
ナックルなんて飛んでる最中に
縫い目に当たる風で
回転が変わるそうだ
それで訳分からん変化球になる

283 :ご冗談でしょう?名無しさん:2023/07/31(月) 12:55:48.03 ID:gDii162U.net
電磁気で、電荷は導体表面に分布するという定理?がありますが
電荷同士の反発力があるから、表面に厚みゼロで分布するよりも
ある程度の厚みで分布したほうがエネルギーが下がりませんか?

284 :poem:2023/07/31(月) 14:30:40.66 ID:qQn1kz2t.net
なんか
日本の野球選手の球はぶれないけど
外国の野球選手の球はぶれるから
日本の野球選手は外国に行って
打てないらしい。合わせられる選手はレベチらしい?

285 :poem:2023/07/31(月) 14:36:46.02 ID:qQn1kz2t.net
変化球って縫い目に風が当たる回転だけって思われてるじゃん?
自分の微判明させたオカルト未解明力学も変化球のそもそも1要素
回転変化球で物凄く曲がる鋭く曲がるふわっと曲がるとかそれは未解明力学の如何で変わる
回転させなくても未解明力学で変化球を作れる。変化球はそもそも回転と未解明力学の合成だから
だから外国のぶれる球や、ストレートやスローボールでも打ちやすい球打ちにくい球がある

286 :poem:2023/07/31(月) 14:38:16.98 ID:qQn1kz2t.net
力学が違うってこと
風力学だけでなく古典力学が

287 :poem:2023/07/31(月) 14:41:57.91 ID:qQn1kz2t.net
ほんと自分も
微判明
全く解明できてなく
最近なんか未解明力学解明しようとしてるの進んでない
自分の場合は体に力学発生させたりで何かしてのレスポンスでスポーツ的(ただ街歩きだけど)に解明しようとしてる

288 :poem:2023/07/31(月) 14:44:22.91 ID:qQn1kz2t.net
>>283
プラスチックに銅メッキ
厚み変えて抵抗どう変わるか試せば
表面だけに電気流れてるのが正しいかわかる

289 :poem:2023/07/31(月) 14:46:19.10 ID:qQn1kz2t.net
もし電気が表面だけに流れてる
=導線抵抗は断面積でなく表面積
なら表面を星形にフラクタル加工した導体なら物凄い抵抗低くなることに

290 :poem:2023/07/31(月) 14:50:22.62 ID:qQn1kz2t.net
どうでもいい質問
容量コンデンサ(導線1本だけ袋小路)じゃなく寄生コンデンサ(導線2本、導線の一部をコンデンサにしただけ)で
本を二冊ページを噛ませるようにコンデンサを噛ませて、片方を直列、片方を並列に繋いだら、直流変圧器作れ

今日も暑いね

291 :poem:2023/07/31(月) 14:56:19.51 ID:qQn1kz2t.net
プールでストレッチって背中や腰や肩治せるんだよ

整体の種類
骸骨の曲がりや捻れや潰れ治し整体
関節のズレや関節包治し整体
筋肉の縮みや怪我治し整体
これだけじゃないだろうけど
骸骨を治す
関節を治す
プールでストレッチはかなりのかなり効果高い
整体医院行ったことないけどそれより高いんじゃない?
銭湯は浅いから意味ない。深くないと

292 :poem:2023/07/31(月) 14:58:29.42 ID:qQn1kz2t.net
背骨の潰れもマシになる。プールでストレッチすると

今の時期は公園プール100円とかだから
この時期お得にレッツ治療ストレッチ

293 :poem:2023/07/31(月) 15:02:04.01 ID:qQn1kz2t.net
>>289
抵抗が表面積依存なら星形フラクタルで抵抗低く

導線の進む毎に等間隔で表面積を変えれば、人体の中枢神経みたいに電流がリープするようになって、面白い電気効果が発生するかも

294 :poem:2023/07/31(月) 15:03:48.99 ID:qQn1kz2t.net
電気が導線の表面しか通らないなら
理論上そう起こりえるでしょ?
オカルトじゃなく理論上あり得るでしょ?

295 :ご冗談でしょう?名無しさん:2023/07/31(月) 16:18:10.72 ID:914fst+W.net
電気力線について質問です。

電気力線上の任意の点Pを考えます。点Pでの接線の方向は、点Pにおける電場の方向と一致するとのことです。

空間を満たしているこの電気力線という曲線を数学的に数式であらわすことはできるのでしょうか?

296 :ご冗談でしょう?名無しさん:2023/07/31(月) 16:21:35.49 ID:914fst+W.net
例えば、2次元空間上の点(1, 0)に+1[Q]の電荷が、点(-1, 0)に-1[Q]の電荷があるとします。
このとき、任意の点P(u, v)を与えたときに、Pを通る電気力線の方程式はどうやって求めるのでしょうか?

297 :ご冗談でしょう?名無しさん:2023/07/31(月) 16:35:06.21 ID:914fst+W.net
E(x, y, z)を3次元空間の任意の電場とする。
P = (u, v, w)をE(u, v, w) ≠ 0であるような3次元空間の任意の点とする。
このとき、曲線(x(t), y(t), z(t))で、以下の性質を満たすものが存在することを証明せよ。
(x(t_0), y(t_0), z(t_0)) = (u, v, w)となるようなt_0が存在する。
任意のtに対して、正の実数αで、α・(x'(t), y'(t), z'(t)) = E(x(t), y(t), z(t))を満たすようなものが存在する。

298 :ご冗談でしょう?名無しさん:2023/07/31(月) 16:44:38.63 ID:914fst+W.net
「電気力線の分布密度の大きいところほどEの強さが大きい」と電磁気学の本に書いてあります。
電気力線の分布密度というのは数学的にどう定義するのでしょうか?

299 :ご冗談でしょう?名無しさん:2023/07/31(月) 17:12:57.41 ID:???.net
>>298
現代ではその様な説明のやり方は廃れた

300 :ご冗談でしょう?名無しさん:2023/07/31(月) 17:17:23.39 ID:???.net
現代的な電磁気学教科書なら
「マックスウェル方程式で記述される電磁場が物理的に存在する」

301 :ご冗談でしょう?名無しさん:2023/07/31(月) 17:27:23.90 ID:914fst+W.net
物理学というより純粋に数学的に考えてどうなのかということを問題にしています。

302 :ご冗談でしょう?名無しさん:2023/07/31(月) 17:31:34.10 ID:???.net
義務教育、初等教科書では記述する数学を知らなかったファラデーのように
仮想的な力線による直感的なイメージで学習させるのが目的。

303 :ご冗談でしょう?名無しさん:2023/07/31(月) 17:38:01.89 ID:???.net
電場磁場から(ある点を通る)力線は簡単に導出できるが、逆は困難だと自分で
やってみれば分かる

304 :ご冗談でしょう?名無しさん:2023/07/31(月) 19:38:03.01 ID:???.net
オームがオームの法則を発見したのは1827年です。
電流計はその原理がすでに知られていました。
エルステッドは電線の下に方位磁石を置き、電流を流すと方位磁石が動くことを発見しました。
検電作用につながりますが、電線をコイルにし、方位磁針とばねを組み合わせることで電流計ができました。
しかし電圧という概念が無く、電圧計は存在しませんでした。
オームは初期はボルタの電池で実験を行いましたが、再現性が低かったため、
沸騰したお湯(100度)と、氷水(0度)と熱電対を組み合わせて電源を作りました。
その電源にいろいろな導体を接続して電流を測定しました。
オームは色々な長さや太さの導体に対して実験を行い、導体に流れる電流の大きさは,
断面積に比例し、長さに反比例することを見いだしました。
さらにオームは、装置の異なる金属の接合部に与える温度を色々に変化させ
回路に流れる電流が、熱電対の温度差に比例することを発見しました。
したがって、本来のオームの法則は
「熱電流を用いた回路に流れる電流は、回路をつくる導体の断面積に比例し、導体の長さに反比例し
 2カ所の接合部に与える温度差に比例する」

305 :ご冗談でしょう?名無しさん:2023/07/31(月) 20:18:12.82 ID:/olpDpnO.net
オームの法則のオリジナル論文は今でも簡単に見ることができる。
実に周到である。
「オームの法則なんて、今なら高校生でも簡単に実証できるだろ」
と思ったら大間違い。多分、99%の高校生は失敗すると思う。

306 :ご冗談でしょう?名無しさん:2023/07/31(月) 20:54:02.38 ID:???.net
>>298
マックスウェル方程式で「D」と書く
電束密度の事

307 :poem:2023/07/31(月) 21:05:53.29 ID:qQn1kz2t.net
>>304

その電源にいろいろな導体を接続して電流を測定しました。
オームは色々な長さや太さの導体に対して実験を行い、導体に流れる電流の大きさは,
断面積に比例し、長さに反比例することを見いだしました


断面積に比例か表面積に比例かの特定する実験って現代まで一度も為されてないの?
今の電気理論は皮膜にしか電流流れないみたいじゃん。断面積に比例するを反証しなきゃだめじゃん

308 :ご冗談でしょう?名無しさん:2023/07/31(月) 21:13:13.50 ID:ZcOUv6yg.net
脱出速度

309 :ご冗談でしょう?名無しさん:2023/07/31(月) 21:25:40.66 ID:ZcOUv6yg.net
すみません、306はミスです。

改めて質問なんですが、脱出速度ってありますよね。
第二宇宙速度ともいうらしいですが、地球の場合11.2km/s以上の速度で打ち上げられた物体は地球の重力圏を脱出できるって。
これの意味がいまいち理解できないんです。
重力圏って理論上無限に存在しますよね?
どれだけ地球から離れようが、重力というブレーキは常にかかり続けるのであれば、いつかどこかで必ず速度が0になって、反転して戻ってくるんじゃないかって思うんです。
これが実際はそうでなくて、重力が0になる無限遠まで到達できるから戻ってこないという理屈がうまく理解できないのですが、どなたか感覚的に納得できる説明をいただけないでしょうか?

310 :ご冗談でしょう?名無しさん:2023/07/31(月) 21:27:26.98 ID:???.net
電流が表面近くに集中するのは高周波の場合の話 (表皮効果)
電磁気学の教科書にも乗ってる初歩の話です

311 :ご冗談でしょう?名無しさん:2023/07/31(月) 21:33:55.60 ID:???.net
>>304
>熱電流を用いた回路に流れる電流は、回路をつくる導体の断面積に比例し、
>導体の長さに反比例し2カ所の接合部に与える温度差に比例する

オームの偉いところは安定な起電力源を苦労して選別し、導体のオーム法則を発見したこと
素人には実験要素のどの物理量が安定しているか洞察力が無いからマネできない。

>>305
実験室の与えられた測定装置頼りだ、それでも導体抵抗を精密に測るのは難しいな

312 :ご冗談でしょう?名無しさん:2023/07/31(月) 21:37:39.25 ID:???.net
>>310
キチガイを煽っても無駄

313 :ご冗談でしょう?名無しさん:2023/07/31(月) 21:48:22.83 ID:???.net
>>311
静電状態の電荷が導体表面だけの証明することは入門者必須だが

導体内部の(定常)電流の状態はオームの法則が基本といえる。
i = kE

314 :ご冗談でしょう?名無しさん:2023/07/31(月) 22:17:10.82 ID:???.net
一次元と二次元では電荷は端に行かないようだ

315 :ご冗談でしょう?名無しさん:2023/08/01(火) 10:06:54.74 ID:???.net
>>283
アーンショウの定理で説明できそう

316 :ご冗談でしょう?名無しさん:2023/08/01(火) 11:35:52.09 ID:???.net
>>309
速度が0になるまで無限時間かかるから
戻って来れない

317 :ご冗談でしょう?名無しさん:2023/08/01(火) 11:40:23.13 ID:???.net
>>314
一次元で反発力があれば
両端に行くしかないだろ
中間の事だと明記しなきゃ

318 :ご冗談でしょう?名無しさん:2023/08/01(火) 11:44:47.28 ID:EvkVX5BI.net
???

319 :ご冗談でしょう?名無しさん:2023/08/01(火) 12:00:49.28 ID:???.net
海外の掲示板で、原子が中性というのはどのレベルまで?というのがあった
そもそも陽子と電子の電荷が相殺して中性になる理由は?
本当は厳密には中性ではないのでは?というものだった。

320 :poem:2023/08/01(火) 14:45:56.96 ID:M/43SNDk.net
>>310
え?
直流では被膜に流れるじゃなく、表面積に比例せず断面積に比例するの?まじ?
逆に高周波交流が被膜に流れるなら
高周波交流においてのみ断面積に比例せず表面積に比例するってなるね?

こちらにしても検証実験は必要。高周波交流のみ

321 :poem:2023/08/01(火) 14:48:15.75 ID:M/43SNDk.net
>>319
なら
コイルの外側はどのレベルまで磁界無い
と同じ疑問だね

どうなの?どうなんだろね?

322 :poem:2023/08/01(火) 14:49:55.06 ID:M/43SNDk.net
あとコンデンサの極板面してる反対側の電界

323 :ご冗談でしょう?名無しさん:2023/08/01(火) 14:50:22.97 ID:???.net
磁気モノポールが存在すれば電荷は量子化されていなければならないので
陽子と電子の電荷は厳密に相殺することが示されるが、
磁気モノポールが見つかっていない現状では経験則でしか言えない。
観測事実としては10^(-21)以下の精度で一致が確認されている

324 :poem:2023/08/01(火) 14:52:20.95 ID:M/43SNDk.net
単に金属暗室が原理なだけなら
コンデンサの電界は意味ないけどね
コイルは導線が磁性体じゃないから磁気暗室には…磁気暗室の作り方知らない
電界暗室は金属板か格子で囲えばできるんだよね?

325 :ご冗談でしょう?名無しさん:2023/08/01(火) 15:19:04.48 ID:???.net
>>323
電荷の偏りは対称性の破れほど影響が単純ではなくて
もし少しでも中性が成立しないのであれば宇宙全体の原子の量を考えれば
宇宙全体に影響を及ぼすとか何とか

326 :ご冗談でしょう?名無しさん:2023/08/01(火) 16:31:18.69 ID:???.net
ダイエットしてる人間が発生させる不自然な声や動作音は
電波に乗って日本を覆い尽くしますか

327 :ご冗談でしょう?名無しさん:2023/08/01(火) 16:59:54.91 ID:???.net
小学生にこの問題を解かせたら正答率が2割
https://imgopt.asahi.com/ogp/AS20230728004637_comm.jpg

328 :ご冗談でしょう?名無しさん:2023/08/01(火) 17:00:55.23 ID:???.net
小学生だとおおざっぱだった
小学6年生
学校教育界に衝撃が走っている

329 :ご冗談でしょう?名無しさん:2023/08/01(火) 17:28:26.46 ID:???.net
>>327
これ、3.2cmの辺を底辺とときの高さが同じって決めつけていいの?
(計算すると同じだとわかるけど)

330 :ご冗談でしょう?名無しさん:2023/08/01(火) 19:12:38.36 ID:???.net
>>329
高さは同じです
選択は
1 「お」の面積が大きい
2 「か」の面積が大きい
3 「お」と「か」の面積は等しい
4 「お」と「か」の面積は、この条件では比べられない

多くの誤答理由は「高さがわからない」

331 :ご冗談でしょう?名無しさん:2023/08/01(火) 19:28:27.41 ID:???.net
>>327
冒頭にあった「朝鮮学校の」の文字を故意に外したのか
あ、違うか、2割も正解するわけないし

332 :ご冗談でしょう?名無しさん:2023/08/01(火) 19:37:35.52 ID:???.net
辺の長さ全部書いちゃってるのがミスリード
その3辺で厳密に同じ高さになるんか?
と、気が利いた子なら当然疑問に思うだろう
そしてまだ計算方法を知らんので、えいやと勘で答えを選んでしまうってわけ

333 :ご冗談でしょう?名無しさん:2023/08/01(火) 19:38:41.10 ID:???.net
>>330
>高さは同じです
三辺の長さがそれぞれ与えれれているだけの状態では高さが同じであることは証明すべきことで、自明なことではない。三平方の定理を会得した者がきちんと計算すると同じだとはわかるんだけど、

あまり深く考えずに幅が一定の補助線上に頂点があると決めつけた子は高さが同じとして3を選ぶだろうけど、より深く考えた小学生が三平方の定理を知らないがゆえに「高さがわからない」としたのだとすればむしろほめるべきだと思う

それにしても3:4:5, 5:12:13, 8:15:17のピタゴラス数をうまく使ってるな

334 :ご冗談でしょう?名無しさん:2023/08/01(火) 22:12:59.07 ID:EvkVX5BI.net
www.kitokito.tym.ed.jp/wp-content/uploads/2023/05/R5_ss_all.pdf

問題の最初にテープのはばは一定であると明記されているので屁理屈の入り込む余地はない。

335 :ご冗談でしょう?名無しさん:2023/08/01(火) 22:57:15.43 ID:???.net
>>323
磁気モノポールって見つかってないけど
存在は確実じゃないの?

336 :ご冗談でしょう?名無しさん:2023/08/02(水) 06:10:43.85 ID:???.net
存在しないことがほぼ確実な

337 :ご冗談でしょう?名無しさん:2023/08/02(水) 08:06:02.32 ID:???.net
アメリカではLK99が盛り上がって電力株が上がってる

338 :ご冗談でしょう?名無しさん:2023/08/02(水) 08:16:23.60 ID:???.net
>>334
そういう重要な前提条件を明示せずに切り取ってスレをミスリードする>>327が糞

339 :ご冗談でしょう?名無しさん:2023/08/02(水) 09:29:28.46 ID:???.net
>>334
明記されてても三角形の頂点がそのテープの辺上にあるかどうかは定かではなく証明すべきこと
これを屁理屈というやつは数学的思考力皆無であることの自白でしかない

340 :ご冗談でしょう?名無しさん:2023/08/02(水) 09:35:26.34 ID:???.net
(三平方の定理を知らない前提の)中学入試で出されたら全員正解扱いにしなきゃいけないレベルの欠陥問題

341 :ご冗談でしょう?名無しさん:2023/08/02(水) 09:43:19.23 ID:???.net
ムニちゃーんダンス
ムニちゃーんしんぷ

342 :ご冗談でしょう?名無しさん:2023/08/02(水) 10:33:54.29 ID:???.net
トラックと衝突するぐらいの運動エネルギーで異世界に行けるのなら
電車に衝突だと相当すごいところに飛ばされるのでないか
トラックが5トンだとして、時速40キロで
5*40^2=8000J

電車が30トンで山手線だと20両編成ぐらいありそうだから
600トン
時速は100キロぐらい出そう
600*100^2=6000000J
トラックの750倍ものエネルギーがある
より高度な異世界、なろう異世界どころじゃない神の世界に行けるかも

343 :ご冗談でしょう?名無しさん:2023/08/02(水) 11:12:33.23 ID:KIflsHc4.net
>>338,340
同意

>>339
バカ

344 :ご冗談でしょう?名無しさん:2023/08/02(水) 12:24:53.85 ID:???.net
LK-99の超電導は本当だろうか

345 :ご冗談でしょう?名無しさん:2023/08/02(水) 12:41:10.14 ID:???.net
マイスナー効果は完全反磁性なので
反磁性体なだけの可能性

346 :ご冗談でしょう?名無しさん:2023/08/02(水) 13:47:20.72 ID:???.net
>>343
337だけ評価が違う理由は?

347 :ご冗談でしょう?名無しさん:2023/08/02(水) 17:06:37.75 ID:Yu+t/WAv.net
電磁気学のガウスの法則はなぜ定理ではなく「法則」なんですか?

クーロンの法則を仮定すれば、ガウスの法則は定理ではないんですか?

348 :ご冗談でしょう?名無しさん:2023/08/02(水) 19:04:15.04 ID:???.net
数学に基なる定理があるから、お前がそれを知らないだけ

クーロンの法則は運動する電荷では成り立たない、お前がそれを知らないだけ

349 :ご冗談でしょう?名無しさん:2023/08/02(水) 19:06:15.15 ID:???.net
カモが来るとシッタカが喜ぶ

350 :poem:2023/08/02(水) 21:18:43.77 ID:Ab+D/B3Z.net
>>348
裸の電荷なら成り立つんじゃない?
導線内は電荷と正孔(陽子と電子の運動の差分が正荷の流れに見える説)が同速度で逆行してるから、電磁界が電荷と正孔の逆行速度差分以外外界では打ち消されるから、外界観測者の速度によらないから、成り立たないって考えてる

351 :ご冗談でしょう?名無しさん:2023/08/02(水) 21:32:56.08 ID:???.net
>>342
MKS単位で計算しなおせ

352 :poem:2023/08/02(水) 21:44:48.44 ID:Ab+D/B3Z.net
>>351
ツッコミ待ちだろうから、ツッコんでしまったのが俗に言う負け

ちなみにツッコミ待ちの場合の勝ち判定は、大喜利したり気の利いた返ししたりして、唸らせたら勝ち

ツッコミ待ちに返答無しも、暴言も負け

マジレスはわからない

353 :poem:2023/08/02(水) 21:45:46.62 ID:Ab+D/B3Z.net
ちなみに
>>352
に書いた主張は
全て適当である

354 :ご冗談でしょう?名無しさん:2023/08/03(木) 08:50:26.01 ID:AHiIkQVB.net
力学の本に、単位ベクトルは無次元であると書いてあります。
なぜですか?

355 :ご冗談でしょう?名無しさん:2023/08/03(木) 10:09:14.29 ID:???.net
無次元だからです

356 :ご冗談でしょう?名無しさん:2023/08/03(木) 10:52:42.17 ID:???.net
荒らしじゃん

357 :ご冗談でしょう?名無しさん:2023/08/03(木) 13:20:42.93 ID:???.net
ポインティングベクトルとも言う
MATLAB用語では1次元配列がベクトルで、2次元配列がマトリックス、0次元がスカラー

358 :ご冗談でしょう?名無しさん:2023/08/03(木) 13:36:57.67 ID:AHiIkQVB.net
作用反作用の法則で、作用と反作用はかならず同種の力であると言えますか?

359 :ご冗談でしょう?名無しさん:2023/08/03(木) 14:13:09.16 ID:???.net
>>357
>ポインティングベクトルとも言う
単位ベクトルを?聞いたことない用法だが

物理で出てくるポインティングベクトルは通常、S=E×Hで定義される、
電磁場のエネルギーフラックスを示す量のこと
ちなみにこのポインティングは人名由来のPoyntingであってpointingではない

360 :poem:2023/08/03(木) 14:47:02.69 ID:aoCVLpcb.net
>>358
嘘言うと
等速運動や加速運動の反作用は時空間の歪み
相対性理論とは等速運動や加速運動のニュートン力学的反作用を計算する
光速に近づくと無限の質量や光速超えられないのはニュートン力学的反作用による

361 :ご冗談でしょう?名無しさん:2023/08/03(木) 15:32:11.19 ID:???.net
以前は速度の反作用とか言ってたが、今度は運動の反作用とか言っちゃってる基地害
とっとと病院に戻れ。

362 :ご冗談でしょう?名無しさん:2023/08/03(木) 15:49:58.60 ID:89m1LCm1.net
音声(空気の縦波)を電気信号に変換するのってどうやっているんですか?電気信号(波形)も縦波ってこと?

363 :poem:2023/08/03(木) 15:55:26.25 ID:aoCVLpcb.net
>>361
等速度差 加速度差 で
運動だと絶対速度になる?
相対速度は速度のみまたは速度差?

364 :poem:2023/08/03(木) 15:58:19.81 ID:aoCVLpcb.net
>>362
昔の電話の受話器は炭素粒の振動により抵抗変化で声電気信号化していたと見た
今は知らない
コンデンサマイクはコンデンサの容量変化

365 :ご冗談でしょう?名無しさん:2023/08/03(木) 19:09:44.47 ID:aUskb+ZV.net
荒らしに反応するヤツも荒らし

366 :ご冗談でしょう?名無しさん:2023/08/03(木) 20:23:34.18 ID:BGMqJMZM.net
【猛暑】携帯扇風機が原因で熱中症になる人が増加…使用時に注意すべき点とは [おっさん友の会★]
https://asahi.5ch.net/test/read.cgi/newsplus/1691049856/

物理学が分からない人だらけで議論が錯綜しております
風量と気化熱の関係が分かりません
何かヒントだけでも教えてください
お願いいたします

367 :ご冗談でしょう?名無しさん:2023/08/04(金) 00:59:04.56 ID:???.net
皮膚から水分が蒸発すれば空気の湿度が上がる
風があれば湿度の大きい空気が吹き飛ばされて
皮膚に接する空気の湿度が下がる
湿度が下がれば蒸発が促進されて気化熱が奪われる

368 :ご冗談でしょう?名無しさん:2023/08/04(金) 01:01:56.65 ID:???.net
>>358
力は全て同種
運動量の変化

369 :ご冗談でしょう?名無しさん:2023/08/04(金) 01:26:14.35 ID:???.net
頭さえ冷やしていれば身体が熱くても機能は大丈夫なものなのか
それが気になる

370 :ご冗談でしょう?名無しさん:2023/08/04(金) 12:10:27.90 ID:IRbvTqIn.net
単振り子の問題について質問です。

戸田盛和著『力学』で、

「円周に沿う力と運動を考えればよい」と結論して、

(d/dt)^2 θ = -g/l * sin θ

という微分方程式を導いています。

ですが、重りにはたらく円周に沿う力の方向は変化します。
軌道が円周ではなく直線で、直線に沿う力が変化しているのならば問題はないと思います。

戸田さんの結果は以下に示すように結果的には正しいですが、正しい結論が導き出せたからといってその導出過程が正しいとはかならずしもいえないと思います。

この戸田さんの導出過程は正しいと考えていいのでしょうか?

371 :ご冗談でしょう?名無しさん:2023/08/04(金) 12:16:09.22 ID:???.net
その変化がsinθで表されてるんじゃないの?

372 :ご冗談でしょう?名無しさん:2023/08/04(金) 12:17:59.57 ID:IRbvTqIn.net
正しいやり方は極座標を使う以下のやり方だと思います。

以下大文字はベクトルを、小文字はスカラーを表します。

R = r * E_r
V = r' * E_r + r * θ' * E_θ
A = (r'' - r * θ'^2) * E_r + (r * θ'' + 2 * r' * θ') * E_θ

F = m * g * cos θ * E_r - m * g * sin θ * E_θ

m * A = F を解くと、

θ'^2 = -g/l * cos θ
θ'' = -g/l * sin θ

となります。

この2番めの式が戸田さんが導出した式と等しいです。

373 :ご冗談でしょう?名無しさん:2023/08/04(金) 12:19:19.37 ID:IRbvTqIn.net
ところで、θに関する2つの微分方程式が出てきましたが、これらは互いに矛盾しないということを簡単に確認できますか?

θ'^2 = -g/l * cos θ
θ'' = -g/l * sin θ

374 :ご冗談でしょう?名無しさん:2023/08/04(金) 12:28:02.48 ID:IRbvTqIn.net
上のR, V, Aについてはリンクは貼りませんがヨビノリの動画で詳しく解説されています。

375 :ご冗談でしょう?名無しさん:2023/08/04(金) 14:17:28.50 ID:IRbvTqIn.net
あ、張力を考えるのを忘れていました。

376 :ご冗談でしょう?名無しさん:2023/08/04(金) 14:19:09.62 ID:IRbvTqIn.net
言いたかったのは、

>>372

のように極座標を使ってθに関する微分方程式を導出しないといけないのではないかということです。

戸田さんの議論は結果だけ正しいだけで、議論自体は乱暴な議論ですよね?

377 :ご冗談でしょう?名無しさん:2023/08/04(金) 14:21:28.15 ID:???.net
>>372

重りに働く力は重力だけなのか?

378 :ご冗談でしょう?名無しさん:2023/08/04(金) 14:23:28.58 ID:???.net
すまん入れ違いになった

379 :ご冗談でしょう?名無しさん:2023/08/04(金) 14:41:40.05 ID:???.net
>>376

戸田の議論は円周に限らずどんな曲線に沿った運動に対しても成立することなのだから、
乱暴だとは思わない。

380 :ご冗談でしょう?名無しさん:2023/08/04(金) 16:08:40.99 ID:???.net
>>370
頭の悪い奴らだな(接線方向の運動が理解できないらしい)
まづ
教科書問題ならば簡単な前提条件しかない、それすら分からんバカが多い
単振り子ならば慣性系の平面運動、錘重心と繋ぐ紐(棒)が直線で伸縮・質量無視だ。
と数え上げればこの位あるのだよ!

ニュートン力学の3法則を適用すれば、紐の張力と重力の法線成分は釣り合って等しい
から円周上の拘束運動となる。
円周の接線方向つまり垂線からの角度θの運動方程式(第二法則)は(図を書けば直ぐ解る)
ml・d^2θ/d^2t = -mg・sinθ  -> d^2θ/d^2t = -g/l・sinθ

この微分方程式は楕円関数を理解してないと解けないから
テイラー展開で3次以上の項を無視できる小さいθとすれば
d^2θ/d^2t = -g/l・θ

この程度の簡単な2階線形微分方程式が解けないなら物理やめた方が幸せになれる。

381 :ご冗談でしょう?名無しさん:2023/08/04(金) 16:24:17.92 ID:IRbvTqIn.net
張力 T = m*g*cos θ + m*l*(d/dt θ)^2

なので、

「紐の張力と重力の法線成分は釣り合って等しい」

は成り立ちませんね。

382 :ご冗談でしょう?名無しさん:2023/08/04(金) 16:26:07.97 ID:???.net
法線方向にも加速度運動してるから釣り合うわけないわな

383 :ご冗談でしょう?名無しさん:2023/08/04(金) 16:44:30.14 ID:???.net
>>380
頭の悪い奴らは死ぬまで治らんらしい

数学・物理学の「(自由)ベクトル」の意味を全く理解してない
簡単に言えば
「(自由)ベクトル」が座標系の選び方には無関係に定義されるということだ。

384 :ご冗談でしょう?名無しさん:2023/08/04(金) 16:59:34.18 ID:???.net
>>383
つまり
>>379 もレスしてるように
平面上の運動曲線の接線方向と接線に垂直(円ならその法線)方向の成分で運動方程式が
成り立つのだよ
>>381 >>382 のような
頭悪い奴にはまったく理解できない、数学・物理はやめた方がいいと前に言ってるだろ

385 :ご冗談でしょう?名無しさん:2023/08/04(金) 17:11:05.10 ID:???.net
>>384
垂直方向に力が釣り合ってたら円運動できんわ
運動方程式を知らないのはどっちだよ

386 :ご冗談でしょう?名無しさん:2023/08/04(金) 17:14:16.06 ID:???.net
向心力も知らない382 m9(^Д^)プギャー(ぷぎゃー)

>頭悪い奴にはまったく理解できない、数学・物理はやめた方がいい
自己紹介乙

387 :ご冗談でしょう?名無しさん:2023/08/04(金) 17:40:57.15 ID:???.net
>>384-385
馬鹿
中学校からやり直せ

388 :ご冗談でしょう?名無しさん:2023/08/04(金) 17:45:03.39 ID:???.net
馬鹿は単振り子問題の前提条件も力の成分さえ分からない

389 :ご冗談でしょう?名無しさん:2023/08/04(金) 18:15:45.24 ID:???.net
ムニちゃーんポポ

390 :ご冗談でしょう?名無しさん:2023/08/04(金) 18:36:33.06 ID:???.net
>>380
補足
石頭で、単振り子問題をx,y座標系でなければイメージできないが数学計算できる人なら

前提条件と4個の未知量 x, y, θ, τ (τ 紐の張力)の連立方程式を4式つくり
消去してθ(t)だけに変形することで  >>380 の式が求まるはずだが、実際にやると
凡人には計算が困難になる。

最初からテーラー展開して sinθ=θ, cosθ=1 にして4式を計算すれば
d^2θ/d^2t = -g/l・θ
が高校生レベルでも簡単に求まる。(初歩的な解き方)

ボケ防止の頭の体操になったわ おしまい

391 :ご冗談でしょう?名無しさん:2023/08/04(金) 19:12:11.73 ID:???.net
>>370
せっかく疑問を持ったんなら
「円周に沿う力と運動を考えればよい」
の意味と成立条件を考えれば良いだろ。
その意味とは「曲線に沿った運動で
慣性の法則と力の法則 (F = Mα) が成り立つ事」なのさ
これは曲線に拘束する力が接線に垂直なら成り立つ
あるいは摩擦のない曲線路と言っても良い

392 :ご冗談でしょう?名無しさん:2023/08/04(金) 19:33:04.18 ID:BfkQF9vQ.net
高校生レベルでテーラー展開って完全に老人ボケ進行しとるやん

393 :ご冗談でしょう?名無しさん:2023/08/04(金) 19:39:07.87 ID:???.net
>>0383

一般座標系では慣性力が現れて力が釣り合う
そうでないと変数=定数のような解が得られない
(今の円運動の場合ならr=l)

394 :ご冗談でしょう?名無しさん:2023/08/04(金) 19:56:02.22 ID:BfkQF9vQ.net
登記老人、ボケ老人特有の、とにかく相手をバカ呼ばわりするだけの
ロジックゼロの書き込みをした挙げ句に
「ボケ防止の頭の体操になったわ おしまい」
とのことです

395 :ご冗談でしょう?名無しさん:2023/08/04(金) 19:57:11.66 ID:BfkQF9vQ.net
こういう老人にだけはならないように気をつけたいものです

396 :ご冗談でしょう?名無しさん:2023/08/04(金) 21:12:37.63 ID:BfkQF9vQ.net
人は誰しも老いて低きに流れ
登記老人のよう醜い(みにくい)存在に成り下がる危険があります

397 :ご冗談でしょう?名無しさん:2023/08/04(金) 22:07:03.57 ID:???.net
お前が馬鹿なだけ

成人日本人の大部分が単振り子の問題すらマトモに解けないからどうでもいいが

398 :ご冗談でしょう?名無しさん:2023/08/04(金) 22:10:50.18 ID:BfkQF9vQ.net
俺がバカなのは認めるけど
「成人日本人の大部分が単振り子の問題すらマトモに解けない」
っていう主張はどうやって証明できるの?
あと、俺がバカなことと、
「成人日本人の大部分が単振り子の問題すらマトモに解けない」
ことの間に何か関連があるの?

399 :ご冗談でしょう?名無しさん:2023/08/04(金) 22:15:53.20 ID:Bfg8IWyJ.net
そんなもの解いてなんの役に立つのかいな?
世の中物理だけで成り立っている訳でもあるまいに。

400 :ご冗談でしょう?名無しさん:2023/08/04(金) 22:17:34.55 ID:BfkQF9vQ.net
>>399
「世の中物理だけで成り立っている」
って誰が主張したの?

401 :ご冗談でしょう?名無しさん:2023/08/04(金) 22:48:06.21 ID:Bfg8IWyJ.net
8月4日のオマエらのやりとり、嫁や娘,ご近所さんに

「5ちゃんねるでレスしてこういうやりとりがあったんだが、どう思う?」

「俺の主張正しいだろ?397の奴おかしいだろ?」と聞いてみろよ。

402 :ご冗談でしょう?名無しさん:2023/08/04(金) 23:21:46.02 ID:???.net
馬鹿なお前の隣人に聞いてみ

大部分の日本人は物理学どころかベクトルの定義すら知らない

403 :ご冗談でしょう?名無しさん:2023/08/04(金) 23:24:40.86 ID:???.net
そいつらに
力の定義?、加速度の定義? ベクトルも知らずに解るわけーだろ

404 :ご冗談でしょう?名無しさん:2023/08/04(金) 23:45:48.74 ID:Bfg8IWyJ.net
もう少しマシなものが返ってくるかと思えばこれか

ガッカリだ....

405 :ご冗談でしょう?名無しさん:2023/08/04(金) 23:47:35.80 ID:???.net
今時調べりゃすぐに出てくることなんて覚えても゙あんまり役に立たんぞ
ベクトルの定義なんか覚えるより徳川の将軍15人覚えてたほうがまだ日常会話で使える場面ありそうだわ

406 :ご冗談でしょう?名無しさん:2023/08/05(土) 00:12:39.08 ID:???.net
>徳川の将軍15人
そんな物はググってコピペすれば十分だろが、馬鹿でも分かるだろ
ところが
ベクトルの定義をコピペだけしても無駄だよ、馬鹿どころ殆どの人は見ただけでは分からない
定義の数学内容が理解できなきゃ全く意味がないということだ。

現代人より2000年以上前のユークリッドのほうが直ぐに理解できるだろ

407 :ご冗談でしょう?名無しさん:2023/08/05(土) 00:27:26.05 ID:???.net
>>398
>俺がバカなのは認めるけど

物理学の分野では馬鹿なだけだよ、物理やめれば成功して幸福になれるかも

単純作業者の中に放り込まれれば俺も馬鹿にされるだけ

408 :ご冗談でしょう?名無しさん:2023/08/05(土) 01:34:24.30 ID:dg8z0Yzo.net
単純作業者に馬鹿にされてしまったw

409 :ご冗談でしょう?名無しさん:2023/08/05(土) 11:36:17.18 ID:???.net
物理をやる場合どんなコンピュータ言語を使えばいいのでしょう?
ExcelやVBA以外で

410 :ご冗談でしょう?名無しさん:2023/08/05(土) 12:00:21.01 ID:4NoiCM/+.net
Wolfram Languageと
PythonまたはJuliaは
どうでしょうか?

411 :ご冗談でしょう?名無しさん:2023/08/05(土) 12:15:11.20 ID:4NoiCM/+.net
Matlabを忘れていました。

412 :ご冗談でしょう?名無しさん:2023/08/05(土) 12:57:40.56 ID:???.net
>>399
実際に、物理だけで成り立っているじゃん

413 :ご冗談でしょう?名無しさん:2023/08/05(土) 12:59:22.24 ID:???.net
>>407
落ちこぼれは、そう信じるしか救いがない

414 :ご冗談でしょう?名無しさん:2023/08/05(土) 13:08:03.54 ID:???.net
>>410
MATLABならまだしも、
PythonやJuliaをつかうぐらいならFortranの方がよくないですか
結局LAPACKやBLASを直接使った方が速い

415 :ご冗談でしょう?名無しさん:2023/08/05(土) 14:15:30.53 ID:???.net
fortranはライブラリしょぼすぎてちょっと…

416 :ご冗談でしょう?名無しさん:2023/08/05(土) 14:34:01.68 ID:???.net
熱力学の内部エネルギーがイマイチわかりません
定積変化で10度上がった場合と
定圧変化で10度上がって体積も増えた場合とで内部エネルギーは同じになるんですか?
粒子数に変化がないのなら内部エネルギーは温度によってだけ変化して体積や圧力は関係ないんでしょうか

417 :ご冗談でしょう?名無しさん:2023/08/05(土) 16:08:40.82 ID:???.net
>>416
そういう疑問が起こる人は、熱力学の第一法則を正しく理解してないから
そのために定義したのが内部エネルギーといえる

一定量の理想気体という条件ならば
>内部エネルギーは温度によってだけ変化して体積や圧力は関係ない

418 :ご冗談でしょう?名無しさん:2023/08/05(土) 22:18:39.45 ID:???.net
できれば第一法則の正しい理解というのを教えてほしいです…

419 :ご冗談でしょう?名無しさん:2023/08/06(日) 00:20:33.97 ID:???.net
エネルギー保存則だけだろ
熱エネルギーと力学エネルギーの和だ

420 :ご冗談でしょう?名無しさん:2023/08/06(日) 12:09:56.66 ID:???.net
ピカー

421 :ご冗談でしょう?名無しさん:2023/08/06(日) 14:53:41.62 ID:ODQrVd93.net
原島鮮著『力学I』

1つの量が直交座標系 S について (Ax, Ay, Az) の3個の実数で与えられ、他の直交座標系 S'
(S と原点を共通に持つとする)について (Ax', Ay', Az') の3個の実数で与えられるとし、
これらの成分間の変換が、点の位置を表わす座標の変換と同様に行なわれるとき、 (Ax, Ay, Az)
または (Ax', Ay', Az') は1つのベクトル A であるとよぶ。

422 :ご冗談でしょう?名無しさん:2023/08/06(日) 14:56:15.64 ID:ODQrVd93.net
この成分間の変換ですが、直交行列で変換されるということが書いてあります。

ですが、単なる直交行列ではなく、その行列式が+1であるような直交行列でないと駄目ですよね?

423 :ご冗談でしょう?名無しさん:2023/08/06(日) 15:31:25.56 ID:???.net
右手系と左手系を入れ替えるような変換を考えたい場合はどうするの?

424 :ご冗談でしょう?名無しさん:2023/08/06(日) 16:09:46.96 ID:ODQrVd93.net
>>423
もし、行列式が-1であるような直交行列を許してしまうと外積がベクトルじゃないということになってしまいますよね。

425 :ご冗談でしょう?名無しさん:2023/08/06(日) 16:41:46.93 ID:SbOXnUNf.net
外積は擬ベクトル

426 :ご冗談でしょう?名無しさん:2023/08/06(日) 16:45:32.61 ID:ODQrVd93.net
>>425
そういう話はどんな本に書いてあるんですか?
初歩的な力学の本には書いてないように思います。

427 :ご冗談でしょう?名無しさん:2023/08/06(日) 16:56:54.09 ID:SbOXnUNf.net
行列式にケチつけるのに擬ベクトルすら知らないってどういう層なの?

428 :ご冗談でしょう?名無しさん:2023/08/06(日) 21:12:46.09 ID:???.net
>>416
こんなとこで質問してないで
まともな本を読んだ方がいい

田崎,前野あたり

429 :ご冗談でしょう?名無しさん:2023/08/07(月) 08:24:57.19 ID:???.net
>>423
古典物理学では右手系と左手系を入れ替えても物理法則は変わらない
右手系(または左手系)だけで演繹推論(実験)しても矛盾が起こらない。

量子論でも同じだと信じられていたが、左右対称性(パリティ)の破れが
1956年頃にベータ崩壊で発見され、当時の物理学者達は愕然とした。

宇宙は人間の僅かな知識など超越しておるのだよ。

430 :ご冗談でしょう?名無しさん:2023/08/07(月) 12:30:36.12 ID:???.net
的外れ・頓珍漢

431 :ご冗談でしょう?名無しさん:2023/08/07(月) 13:10:30.00 ID:jXbsubA9.net
「膨張した赤色巨星に飲み込まれたはずなのに、なぜか生きている惑星」が500光年先で発見される→一体どういうトリック?
https://news.yahoo.co.jp/articles/03c0d3f600b580ab77b633131d0f85a1a51123c4?page=2


赤色巨星の中で惑星が軌道を保って公転し続ける事は出来ないのですか?

432 :ご冗談でしょう?名無しさん:2023/08/07(月) 14:32:31.07 ID:???.net
素人がふと思った疑問です。
恒星が核融合や恒星風などで軽くなることで惑星の軌道が変わったり追い出されたりしますか?

恒星は無数にあるので、質量減少で起こる(観測不能な程弱い)重力波は常に地球にやってきており
たまたま計測器のところで強め合って観測できた、ということはありえますか?

433 :ご冗談でしょう?名無しさん:2023/08/07(月) 14:54:35.59 ID:???.net
軽くなって軌道が変わるのは微小なので観測不能
質量減少は対称なので重力波は出ない

434 :ご冗談でしょう?名無しさん:2023/08/07(月) 16:38:40.49 ID:jXbsubA9.net
逆に潮汐摩擦が発生して巨星内部に引っ張り込まれたりしてな・・・

435 :ご冗談でしょう?名無しさん:2023/08/07(月) 17:28:12.63 ID:???.net
>>429
中学生が導体中を流れる電流の正体が「マイナス電気の電子」だと理科で知ったとしよう
電子がマイナス電気なら電池のマイナス極からプラス極に向かって流れるはずである
ところが理科では電流は電池のプラス極からマイナス極に導体中を流れると教えている

ここで問題
@ 導体中を電子が運動する方向に教科書が電流の定義を訂正しないのはなぜか?

A 現実の電流が逆方向なのに電磁気学の理論に矛盾が起こらない理由を説明せよ

436 :ご冗談でしょう?名無しさん:2023/08/07(月) 17:58:04.50 ID:???.net
qqq臭がするな

437 :poem:2023/08/07(月) 18:17:18.51 ID:4hl75urY.net
電子殻にあるニュートリノ中性微子が電池の+から電池の-に流れてるから

電気の流れはニュートリノ流の向き
電子流の向きの反対だから

教科書も正しい
歴史は間違ってなかった

438 :poem:2023/08/07(月) 18:18:02.74 ID:4hl75urY.net
教科書は変える必要ないんだZE

439 :poem:2023/08/07(月) 18:18:46.04 ID:4hl75urY.net
+も流れちゃう☆!だから

440 :poem:2023/08/07(月) 18:20:34.58 ID:4hl75urY.net
あと電気の力は

陽子、中性子、電子、中性微子の合力
かつ光はクォーク物理だからクォークも合力に介在してると思うから
+と-電気性質同じ。合力だから

441 :poem:2023/08/07(月) 18:21:25.81 ID:4hl75urY.net
とにかく
歴史は真☆実

442 :poem:2023/08/07(月) 18:22:02.56 ID:4hl75urY.net
くっくっくならおひさ

暴れて
楽しいから

443 :poem:2023/08/07(月) 18:23:21.17 ID:4hl75urY.net
皆くっくっく実は好きだから
久しぶりにレスバできるなら楽しんでレスつけてくれるから
毎度そうじゃん

444 :poem:2023/08/07(月) 18:24:38.21 ID:4hl75urY.net
久しぶりにくっくっく登場かー

自分は宗教解明に専念中
くっくっくレス返すことあったらよろね

445 :ご冗談でしょう?名無しさん:2023/08/07(月) 19:53:29.04 ID:???.net
キチガイ脳には物理は無理だから巣に池

446 :ご冗談でしょう?名無しさん:2023/08/07(月) 19:57:33.63 ID:Xd5JsHVM.net
今、戸田盛和著『力学』を読んでいますが、もう少しで剛体の力学の章を読み始められます。

テコの原理というのを中学校で習います。
高校でニュートンの運動の法則を習います。
運動の法則によってすべての力学的現象が説明されることになっていますが、テコの原理も証明できるんですか?

447 :ご冗談でしょう?名無しさん:2023/08/07(月) 20:06:57.66 ID:???.net
>>446
当然だ
ただし、ニュートン力学の3法則を正しく適用しないとダメだ

448 :ご冗談でしょう?名無しさん:2023/08/07(月) 20:26:20.43 ID:???.net
ニュートン力学の3法則から演繹して角運動量とモーメントを導出し剛体のつり合い
条件を出せばよい
「テコの原理」や「浮力の原理」は原理ではなくニュートン力学の定理

449 :ご冗談でしょう?名無しさん:2023/08/07(月) 20:27:48.16 ID:Xd5JsHVM.net
>>447-448
ありがとうございます。
高校物理や中学校の物理で嫌いだったのが力学で質点以外の対象を扱うときでした。

450 :ご冗談でしょう?名無しさん:2023/08/07(月) 21:03:18.53 ID:???.net
>運動の法則によってすべての力学的現象が説明される

さらに、ニュートン力学から演繹できる力学現象ならば
空間を反転した世界(または鏡に映った世界)でも成り立つことが証明できる。

力学的現象が無数に有るのになぜ証明できるのか?
その秘密はニュートン力学の3法則が基本原理だから
空間を反転した対称性から3法則が同じになるか調べればいいだけ。

451 :ご冗談でしょう?名無しさん:2023/08/07(月) 21:12:03.17 ID:???.net
>>450
物理現象が無数に有る、万有引力理論や電磁気学でも同じことが言える

452 :ご冗談でしょう?名無しさん:2023/08/07(月) 21:21:13.13 ID:???.net
>>451
電流の向きを導体中の電子の運動方向に定義し直すために、教科書だけでなく
世界中の国家・産業が莫大なコストを掛ける必要が全く無いことも証明できる。

453 :ご冗談でしょう?名無しさん:2023/08/07(月) 21:37:02.90 ID:???.net
>>435
昔の電流定義を知ってるか?
銀の陽イオンの流量で定義してたんだぜ
現在でも流量測定は質量のあるイオンの方が高精度だ
プラス電荷で定義して何の問題もない
デバイスで使うなら微小質量の電子が便利なだけだ
目的も区別せずに文句つけるな

454 :poem:2023/08/07(月) 21:47:44.62 ID:4hl75urY.net
>>452
自分は
電気の向きはニュートリノが流れる方向で正解だから、世界産業とかに莫大なコストで電気の向きを修正させるのはそもそも根本間違いだと思うから、根本的に否定だけど
世界産業が莫大なコスト賭けても、水路で電気回路をイメージするのは根本的に誤り(電圧は水圧でなく少なくともスプラッシュ力)で水路イメージを根本にしてる物事産業があれば誤りだから、治して欲しい。代案はわからないけど。少なくともスプラッシュ力

455 :poem:2023/08/07(月) 21:49:24.89 ID:4hl75urY.net
>>453
陽イオン流の方が質量大きいから測りやすいんだ
すっごく便利だね

456 :poem:2023/08/07(月) 21:53:48.73 ID:4hl75urY.net
電子も測れる技術には今なってるから
電子でもすっごく便利に測れる(そりゃ陽イオンの方が質量大きいから有効桁数は陽イオンだけど、電子もすっごく便利の域には既に到達)
陽イオンと電子の質量の違いで
陽イオンは有効桁数、電子は微小の感知を有効桁数大きいけどで
両方で今の技術限界を駆使し無双

科学技術の勝利だね
すごいね科学技術
研究者のがんばりだよ

457 :poem:2023/08/07(月) 21:57:17.18 ID:4hl75urY.net
中性子と中性微子の相互作用研究して。必ずあるから

陽子は電子で観測
中性子は中性微子で観測
電子は陽子で観測
中性微子は中性子で観測

必ず中性子と中性微子相互作用できるから
そして研究者ら中性子と中性微子の相互作用解明できるから!
研究者は勝利するから!中性子中性微子の解明できるから!
行けるから!できるよ研究始めれば!

458 :神の僕 ◆P10fR708T6 :2023/08/08(火) 01:37:42.44 ID:xm1hymoL.net
エネルギーの定義と、その存在証明をしてください。

459 :poem:2023/08/08(火) 02:50:02.34 ID:S0AJEMcp.net
kg(m/s)^2って
m/s^2…加速度
m^2/s…減速度
kg速度系…運動量系
↓よって
加速しながら減速する速度系の、運動量系の一種

エネルギーって物理量と違った量にしないと言い換えなだけじゃん。物理量の中の物理量の1つkg(m/s)^2だけを全物理量の中でエネルギーと名指しするだけの、単 なる物理量より狭い意味じゃん。
じゃなくてエネルギーとエントロピーは共に物理量とは別の計算方式。
物理量はかけ算や冪算や足し算の数値計算だから
エネルギーエントロピーは数値計算でない論理計算は…?いやエネルギーエントロピーは論理計算では無理か?
数値計算と論理計算と何か計算と他他他…の中のどれかがエネルギーエントロピー計算ってアイデアどう?

460 :ご冗談でしょう?名無しさん:2023/08/08(火) 09:33:32.01 ID:???.net
いい加減、お前の脳内にしかない妄想粒子poemonを
現実の粒子と同じ中性微子の名で呼び続けるのをやめろ

461 :ご冗談でしょう?名無しさん:2023/08/08(火) 13:27:34.26 ID:7Q6yDYSV.net
戸田盛和著『力学』

2体問題について、
一方の質点を座標系の原点にしたときの他方の質点の運動がどうなるかということ、
2質点の重心を座標系の原点にしたときの1つの質点の運動がどうなるかということ
をかなりのページ数を使って書いています。

このような単なる数学的なテクニックって物理学的に重要なんですか?

462 :ご冗談でしょう?名無しさん:2023/08/08(火) 13:42:29.37 ID:???.net
>>433
ありがとうございます。
超巨大恒星であっても燃料を使い切って減る質量は微々たるものなのですね。

463 :ご冗談でしょう?名無しさん:2023/08/08(火) 13:53:55.19 ID:???.net
いや、質量放出による中心星の質量減少と惑星軌道への影響はそれなりにあるぞ。
太陽でさえ赤色巨星化時の質量減少で地球軌道は1.4倍に広がるとかいう予想もある。
重力波はほとんど出ないだろうが

464 :ご冗談でしょう?名無しさん:2023/08/08(火) 14:03:00.38 ID:???.net
>>461
座標系の変換は物理の本質の一つだよ

465 :ご冗談でしょう?名無しさん:2023/08/08(火) 14:06:22.20 ID:???.net
二体問題が相対座標と換算質量によって一体問題に帰着されることを
単なる数学的なテクニックとしかとらえられないやつにとっては無意味な議論かもな
何に意味を見出すかは人それぞれなので、勝手にドヤってればよろしい

466 :ご冗談でしょう?名無しさん:2023/08/08(火) 14:16:05.22 ID:???.net
いつも思うんだけど、松坂くんは自分で教科書書けばいいじゃん
そうすれば非の打ち所のないものが作れるでしょ

467 :ご冗談でしょう?名無しさん:2023/08/08(火) 15:00:41.41 ID:f0FoCCTS.net
重心系と実験室系の変換は原子核実験をはじめとして重要な手法だから力学の教科書で詳しく解説する必要がある。

468 :ご冗談でしょう?名無しさん:2023/08/08(火) 16:43:33.19 ID:7Q6yDYSV.net
戸田盛和著『力学』

「剛体の簡単な運動」という章を読み始めました。

剛体の位置と配向について説明しています。
コマの絵を使って配向について説明しているのですが、分かりにくいです。
剛体の重心を原点とする3次元の座標軸を描いて、x軸の位置を決めるのに、2変数必要で、
さらにy軸とz軸の位置を決めるのに1変数必要という説明をしていたとしたら分かりやすかったと思います。

469 :ご冗談でしょう?名無しさん:2023/08/08(火) 17:28:55.80 ID:???.net
>>468
スレ立てたんならそっちでやってろボケ

470 :ご冗談でしょう?名無しさん:2023/08/08(火) 19:25:42.03 ID:CzVEDypm.net
座標変換って大事だと思うけど、中々難しいわな。数学的な問題だけど。

関係ないけど、重力が強いとこと弱いところがあるとき、液体は弱い方に集まり膨らむはずだけど、磁性流体の動画見たとき磁極に盛り上がるように集まってて、おかしいなと思ったんだけど、おかしくないかな?

重力が強いとこと弱いとこがあると、固体なら何となく強いとこに山ができそうなんだけども。

471 :ご冗談でしょう?名無しさん:2023/08/08(火) 22:13:04.58 ID:CzVEDypm.net
無重力状態で磁性流体を磁石に十分な量、近づけたら、どんな形になるのだろうか。それと、どっから磁性流体が詰まってくのかな。こういう実験も、宇宙に住んでればできるのにな。

472 :ご冗談でしょう?名無しさん:2023/08/08(火) 23:49:41.88 ID:CzVEDypm.net
太陽も惑星との共通重心で公転してるというし、共通重心がわりと太陽の外に出るらしいから、そうなると、太陽から降り注ぐ光量とかも当然変化するんだから、太陽自身の活発度とかも考えると、どのくらい光量が変化してるのか、地球温暖化問題じゃ気になるね。

太陽系の太陽や惑星の運動が地球の環境にどんな影響持つんだろうか。ないわけないと思うし。星間物質とかも変動してるのかなぁさ

473 :ご冗談でしょう?名無しさん:2023/08/09(水) 00:25:59.98 ID:???.net
>>470
引力が強い所に集まると思わんお前の方がおかしい

474 :ご冗談でしょう?名無しさん:2023/08/09(水) 03:48:59.34 ID:???.net
光子って質量0なのにエネルギーは0じゃないのなぜ?
E=mc^2 

475 :ご冗談でしょう?名無しさん:2023/08/09(水) 04:41:20.81 ID:???.net
>>474
それは静止質量についての等価式だ

476 :無学な私:2023/08/09(水) 07:25:37.42 ID:CHbxTm2/.net
>>454
私は20年以上前に工業高校の電気科を卒業したエリートだけど、
今だに電流と電圧のイメージは謎だ

477 :ご冗談でしょう?名無しさん:2023/08/09(水) 08:28:36.00 ID:/Bk/NEZH.net
>>473
月とか太陽、その他の惑星とかの影響で地球の重力場が変化したとき、水面はどんな変化をするのかなと考えてたんだよね。それと水がどんな動きをするかなとかさ。

北極とか南極が一番引力が強くなるはずだから、そこの海面高さとか見れば、色んな情報が得られるんじゃなかろうかと思ったりさ。

ふと、重力が強いとこが海面低くなるというのも感覚では変だから、なぜかと考えてたんだわ。

478 :ご冗談でしょう?名無しさん:2023/08/09(水) 10:16:48.35 ID:???.net
>>476
>今だに電流と電圧のイメージは謎だ
日常経験に近いニュートン力学では感覚的なイメージが使えるだけだ
万有引力、電磁気の学習で初めて数学的な抽象イメージが必要になる

ブレークスルーが出来ない人(女性に多い)は相対性理論、量子力学の学習は無理

>>477
>重力が強いとこが海面低くなるというのも感覚では変
感覚イメージは通用しない
力のベクトルの抽象イメージができないだけ

479 :ご冗談でしょう?名無しさん:2023/08/09(水) 10:19:26.96 ID:???.net
このマヌケな評論家は何がしたいの?

480 :ご冗談でしょう?名無しさん:2023/08/09(水) 10:38:42.92 ID:???.net
お前はメコ爺の成れの果てだろ

481 :ご冗談でしょう?名無しさん:2023/08/09(水) 10:47:46.31 ID:/Bk/NEZH.net
まぁいいけど、等位曲面が平面なら水面も水平だけど、等位曲面が窪んでたら、水面も窪む。

流体が満たされた状態での話だから、流体がこれから満たされようとする場合や固体の物体が落下してく場合が磁性流体の挙動なのかな。力のベクトルだの何の話かよくわからないが。

482 :ご冗談でしょう?名無しさん:2023/08/09(水) 12:02:45.41 ID:mM90B8cn.net
磁性流体の場合は、磁力で磁極に集まった流体が表面張力で盛り上がってるということでは?

483 :ご冗談でしょう?名無しさん:2023/08/09(水) 12:21:54.41 ID:???.net
>>481
>等位曲面が平面なら水面も水平だけど、等位曲面が窪んでたら、水面も窪む。

'等位曲面'とやらの物理的意味と計算方法を何も言えないで妄想してるだけだろ

484 :ご冗談でしょう?名無しさん:2023/08/09(水) 13:16:15.24 ID:???.net
なるほどね
重いから沈むという感覚か
流体表面を膜と勘違いしてるな

485 :ご冗談でしょう?名無しさん:2023/08/09(水) 13:22:49.15 ID:/Bk/NEZH.net
月とかの重力で地球の合成された重力場が弱まると、等位曲面ももちろん変化して、水も低いところに移動するはずだけど、そもそも重力が弱まるなら、水の重さが少し減るんだから、いくらか膨張するはずだよね。そうなると、潮汐による海面の変化は重力場の変化とそれによる水の膨張の2つの要因で決まるのだろうか。

486 :ご冗談でしょう?名無しさん:2023/08/09(水) 16:03:41.90 ID:???.net
重力で水かどのように運動するかを始めに考えようとしないで

馬鹿は水の膨張とか連想するのか?  思考がオカシイのが判る

487 :ご冗談でしょう?名無しさん:2023/08/09(水) 16:09:57.24 ID:???.net
正常な知能の学生ならパスカルの原理を直ぐに連想するだろう

488 :ご冗談でしょう?名無しさん:2023/08/09(水) 17:11:02.48 ID:8n2HMawF.net
熱力学や統計力学の本の前提となる知識は力学だけでOKですか?

489 :ご冗談でしょう?名無しさん:2023/08/09(水) 17:28:03.11 ID:???.net
>>488
熱力学は実験観測の事実が第一、第二法則(原理)だと理解すれば微積分の学習で十分

統計力学は確率論・統計の数学を学習してないと力学だけでは理解できない。(特に第二法則)

490 :ご冗談でしょう?名無しさん:2023/08/09(水) 18:03:26.42 ID:???.net
高校1年目は物理と化学の授業を半年間潰して、微積分と線形代数の振り替え授業を
文科省に陳情しよう。

491 :ご冗談でしょう?名無しさん:2023/08/09(水) 19:11:03.95 ID:8n2HMawF.net
>>489
ありがとうございます。
物理学の中での前提知識は少ないんですね。
確率論、統計はそれほど難しくないと思うので、力学の本を読み終わったら、熱力学や統計力学の本を読んでみようと思います。

492 :ご冗談でしょう?名無しさん:2023/08/09(水) 19:37:20.56 ID:mM90B8cn.net
確率論ってボレル集合がどうのこうのとかいう物理には役立たない数学だから統計力学の理解には全く不要だよ
統計力学で必要な数学はテイラー展開の技術的習熟と、ガウス積分関連の計算技法だね。
ガンマ関数とかゼータ関数とかが突然出てきたりするけど、適当な物理数学の本ですぐ理解できる

493 :ご冗談でしょう?名無しさん:2023/08/09(水) 19:38:29.57 ID:mM90B8cn.net
>>489
いい加減なことを書くな

494 :ご冗談でしょう?名無しさん:2023/08/09(水) 19:58:54.28 ID:???.net
水が低いところに移動するのは
重力が均等な場合のみ
原理からの演繹ができず
常識に囚われた理屈しか捏ねん
ようじゃ物理はできん

495 :ご冗談でしょう?名無しさん:2023/08/09(水) 20:00:17.63 ID:???.net
>>492 >>493
計算馬鹿が湧いたか
苦悶馬鹿は計算式ありきで統計力学が解ったと勘違いするらしい

496 :ご冗談でしょう?名無しさん:2023/08/09(水) 20:04:13.39 ID:???.net
>>492
そういうこと言ってる奴は
出鱈目な確率計算に陥って出られない
教科書に載ってない事は自力で計算できない
引用主義者になるしかない

497 :ご冗談でしょう?名無しさん:2023/08/09(水) 20:11:14.61 ID:mM90B8cn.net
>出鱈目な確率計算に陥って出られない

具体例を示せますか

498 :ご冗談でしょう?名無しさん:2023/08/09(水) 20:21:27.73 ID:mM90B8cn.net
「確率論の数学を学習しないと、出鱈目な確率計算に陥って出られない」
という主張をしたからには、その根拠と具体例を示す必要があります。
ボレル集合や測度論を知らないと、統計力学の計算でどのような間違いを犯すのですか?

499 :ご冗談でしょう?名無しさん:2023/08/09(水) 20:48:52.42 ID:???.net
>>498
おぜん立てした計算式しかできないお前には分らんだろが

確率論と定義が特定の偶然現象に無関係でなければ、広範囲に適用できないということだ

仮に確率をサイコロの出目で定義したら、分子運動に適用できるかも全く判らんだろ

500 :ご冗談でしょう?名無しさん:2023/08/09(水) 20:49:26.66 ID:mM90B8cn.net
具体例を示せ、という日本語が理解できないのですか?

501 :ご冗談でしょう?名無しさん:2023/08/09(水) 20:52:40.98 ID:mM90B8cn.net
>仮に確率をサイコロの出目で定義したら

そんな仮定は誰もしてません。
あなたの主張は
「確率論の数学を学習しないと、出鱈目な確率計算に陥って出られない」
です。
これの根拠と具体例を示してください。

502 :ご冗談でしょう?名無しさん:2023/08/09(水) 20:57:28.07 ID:???.net
>>500
具体的だろがお前は「サイコロの出目で定義」が使えるでも言うのか

503 :ご冗談でしょう?名無しさん:2023/08/09(水) 20:59:55.44 ID:???.net
>>501
お前はおぜん立てした計算しか出来ないから、何がデタラメかも解らないだけ

504 :ご冗談でしょう?名無しさん:2023/08/09(水) 21:01:55.15 ID:OBizg801.net
測度論難しいーーーー

505 :ご冗談でしょう?名無しさん:2023/08/09(水) 21:02:12.28 ID:mM90B8cn.net
>>502
あなたは愚鈍なので、あなたにもわかるように質問を噛み砕きます。

二人の学生AくんとBくんがいます。
Aくんは確率論の数学を理解しています。
Bくんは確率論の数学を理解していません。
AくんとBくんが統計力学の教科書を読んで学習を進めました。
Aくんだけが正しく理解できて、Bくんが理解できない統計力学の内容はどれですか?

506 :ご冗談でしょう?名無しさん:2023/08/09(水) 21:06:21.77 ID:???.net
ニュートン力学の分子運動から、熱力学の様な熱平衡状態が一つしかないとどう説明する?

507 :ご冗談でしょう?名無しさん:2023/08/09(水) 21:08:17.82 ID:mM90B8cn.net
愚鈍は言いすぎたかもしれない。申し訳な。クソバカとでも書くべきだった。

508 :ご冗談でしょう?名無しさん:2023/08/09(水) 21:08:59.34 ID:???.net
>>505
お前はおぜん立てした計算しか出来ないのに、「統計力学を理解した」と思い込んでるだけだろ

509 :ご冗談でしょう?名無しさん:2023/08/09(水) 21:10:26.61 ID:mM90B8cn.net
>>508
質問への答えを待っています。
質問が理解できたら考えてみてください。

510 :ご冗談でしょう?名無しさん:2023/08/09(水) 21:32:42.12 ID:???.net
確率論を詳細に学習できた学生なら、統計力学のエルゴード定理も確率論から証明できるのが解る。

511 :ご冗談でしょう?名無しさん:2023/08/09(水) 21:57:16.30 ID:Vsm91JXg.net
>>488
量子力学

512 :ご冗談でしょう?名無しさん:2023/08/09(水) 21:58:46.02 ID:Vsm91JXg.net
エルゴード定理は不要

513 :ご冗談でしょう?名無しさん:2023/08/09(水) 22:12:19.32 ID:mM90B8cn.net
野球選手に必要なのはバットの振り方を会得することで、バットの作り方を知る必要ない。

514 :ご冗談でしょう?名無しさん:2023/08/09(水) 22:22:16.15 ID:???.net
>>510
>統計力学のエルゴード定理も確率論から証明できる

確率論はルベーグ測度が公理なので
確率論を基にした統計力学ならば量子力学を仮定しなくても熱力学と同じ結果が得られる
のではないか
入門者向きでないが、そういう統計力学教科書を誰か見かけたら教えて

515 :ご冗談でしょう?名無しさん:2023/08/09(水) 22:37:01.44 ID:Vsm91JXg.net
量子力学から統計力学なしで第二法則が導けるとさ

ttps://www.jstage.jst.go.jp/article/butsuri/73/6/73_361/_pdf/-char/ja

516 :ご冗談でしょう?名無しさん:2023/08/09(水) 22:40:53.02 ID:mM90B8cn.net
>確率論はルベーグ測度が公理

なんすかこれ・・・

517 :ご冗談でしょう?名無しさん:2023/08/09(水) 22:45:18.99 ID:mM90B8cn.net
確率論の公理はコルモゴロフが定式化した一連のアイディアでしょ

518 :ご冗談でしょう?名無しさん:2023/08/09(水) 22:51:55.26 ID:7ydAqLKI.net
測度変わると確率変わっちゃうんだよ

だから確率を求める時は必ずどの測度かを指定しなきゃいけない

519 :ご冗談でしょう?名無しさん:2023/08/09(水) 22:53:21.94 ID:mM90B8cn.net
「確率論の公理がルベーグ測度」という間抜け極まる書き込みは初めてお目にかかった

520 :ご冗談でしょう?名無しさん:2023/08/09(水) 22:55:15.82 ID:mM90B8cn.net
>>518
統計力学で「公理選択」を誤ると間違った結論に至る例があれば提示してください。

521 :ご冗談でしょう?名無しさん:2023/08/09(水) 23:19:27.05 ID:???.net
>>519
アホか
ルベーグ測度を確率の公理に使ってるからおかしくない、公理はそれだけではないだけ

522 :ご冗談でしょう?名無しさん:2023/08/09(水) 23:21:18.01 ID:???.net
簡単に言えば数え上げられるということが統計力学では重要なのだよ

523 :ご冗談でしょう?名無しさん:2023/08/09(水) 23:23:28.08 ID:mM90B8cn.net
>>521
「確率の公理にルベーグ測度を使ってる」
と君は主張しているけど、その主張の根拠を示してくれ。
コルモゴロフが定式化した確率の公理にはルベーグ測度のような具体例な概念は無かったし不要だ。

524 :ご冗談でしょう?名無しさん:2023/08/09(水) 23:27:12.31 ID:mM90B8cn.net
>>522
「簡単に言えば」とか言って毎回毎回逃げようとするけど
君は君の主張の根拠を一度も示せてないよね?

525 :ご冗談でしょう?名無しさん:2023/08/09(水) 23:31:55.85 ID:???.net
>>523
>>522 に簡単に書いてあるだろが

526 :ご冗談でしょう?名無しさん:2023/08/09(水) 23:33:42.34 ID:mM90B8cn.net
>>525
すまんが、>>522では全く説明になってない。
公理まで立ち戻ろうとするなら、省略せずにロジカルに説明してくれ。

527 :ご冗談でしょう?名無しさん:2023/08/09(水) 23:34:02.28 ID:???.net
数えられない数は使い物にならない、馬鹿でも分かる

528 :ご冗談でしょう?名無しさん:2023/08/09(水) 23:35:37.22 ID:???.net
>>526
確率論の教科書買って調べればいいだけだろ

529 :ご冗談でしょう?名無しさん:2023/08/09(水) 23:36:08.77 ID:mM90B8cn.net
>>527
念のため一応確認したいのだけど、コルモゴロフの確率公理って何か分かってる?

530 :ご冗談でしょう?名無しさん:2023/08/09(水) 23:41:17.75 ID:mM90B8cn.net
>>528
コルモゴロフの古典は一応読んで理解して、伊藤の本は挫折して舟木のあれでようやく理解できたんだけど
きみはどの本で確率論を理解したの?

531 :ご冗談でしょう?名無しさん:2023/08/10(木) 11:13:05.49 ID:???.net
そんなに知りたいなら本家ロシアの確率論の教科書を買って読め

532 :ご冗談でしょう?名無しさん:2023/08/10(木) 12:37:59.07 ID:???.net
確率論くらいで古典とか本家とか大袈裟やな
本質はルベーグ測度で片付く

533 :ご冗談でしょう?名無しさん:2023/08/10(木) 13:19:49.53 ID:???.net
確率論の公理でルベーグ測度w
知ったかも大変だなぁw

534 :ご冗談でしょう?名無しさん:2023/08/10(木) 13:35:51.24 ID:nlSy2hmr.net
量子力学でヒルベルト空間を使うらしいですけど(まだ勉強してない)
どのヒルベルト空間を使うとかありますか?
数列空間l^pとかでもいいんでしょうか
それとも特に指定せずに何かしらのヒルベルト空間であると仮定するのでしょうか?

535 :ご冗談でしょう?名無しさん:2023/08/10(木) 13:36:35.24 ID:nlSy2hmr.net
訂正
l^p→l^2

536 :ご冗談でしょう?名無しさん:2023/08/10(木) 13:51:10.94 ID:???.net
しんぷさんもムーニーちゃん プー

537 :ご冗談でしょう?名無しさん:2023/08/10(木) 14:10:54.40 ID:???.net
メコ爺 -> しんぷさんもムーニーちゃん

538 :ご冗談でしょう?名無しさん:2023/08/10(木) 14:15:35.79 ID:Ln8SYL+v.net
戸田盛和著『力学』

剛体の章を読んでいますが、球や円筒系の物体を坂道に置くとなぜ転がるのでしょうか?

539 :ご冗談でしょう?名無しさん:2023/08/10(木) 14:36:45.54 ID:4gBtDqld.net
気持ちいいから

540 :ご冗談でしょう?名無しさん:2023/08/10(木) 15:04:23.17 ID:Ln8SYL+v.net
>>538
このような誰でも抱くような基本的な疑問を完全に無視しています。

541 :ご冗談でしょう?名無しさん:2023/08/10(木) 16:43:49.98 ID:???.net
その疑問に完璧に答える素晴らしい教科書をぜひ書いてくれ

542 :ご冗談でしょう?名無しさん:2023/08/10(木) 17:15:44.89 ID:eLu9Ar/c.net
物理は現象を予測できればいい
転がると言っとけば当たる
理由は不問

543 :ご冗談でしょう?名無しさん:2023/08/10(木) 17:34:00.16 ID:Ln8SYL+v.net
『Sears & Zemansky's University Physics with Modern Physics』

という本に、

斜面を滑らずに転がり落ちる車輪の重心の速さを v とし、車輪の半径を R とし、車輪の角速度を ω とする。
このとき、 v = R * ω
が成り立つことの説明が書いてあります。

その説明ですが、

滑っていないのだから、斜面と接触している車輪の点の斜面に対する速さは常に 0 でなければならない。
よって、 v = R * ω
が成り立つ。

という説明です。

普通の説明は以下の説明ではないでしょうか?

時刻 0 での車輪の回転角 θ(0) = 0 とする。
時刻 t での車輪の回転角を θ(t) とすると、時刻 0 から時刻 t までに車輪の重心が移動した距離は、車輪の半径が R で、車輪は
滑らないから、 R * θ(t) である。
よって、車輪の重心の速さは、 R * d/dt θ(t) = R * ω(t) である。

どっちの説明が優れていますか?

544 :ご冗談でしょう?名無しさん:2023/08/10(木) 18:05:46.51 ID:gp+a9fam.net
車輪が滑っている時も有効な説明としては車輪の密度が円周方向に一様であるなら接地点での摩擦力だけが力のモーメントを与えるからそのモーメントの方向に車輪は回る

545 :ご冗談でしょう?名無しさん:2023/08/10(木) 18:56:46.22 ID:P9BwJnRl.net
サイクロイドをびろーんと延ばしたり、短くしてそれを微分するのが一般的な定式化なのかな。

車輪ならともかく球体で球体のある場所の軌道とか考えると、複雑だよね、こういうの。天体の動きとかさ。

546 :ご冗談でしょう?名無しさん:2023/08/10(木) 19:03:49.43 ID:SZk9mtpj.net
>>534
背伸びしすぎだな
先に普通のベクトル空間を勉強しなよ

547 :ご冗談でしょう?名無しさん:2023/08/10(木) 21:24:21.43 ID:???.net
>>533
その程度で知ったか認定とはレベル低いな

548 :ご冗談でしょう?名無しさん:2023/08/10(木) 21:46:43.52 ID:???.net
>>534
ヒルベルト空間はどれも同型だから
何のヒルベルト空間でも良いけど
実例を作る時はそれに便利なのを使うよ

549 :ご冗談でしょう?名無しさん:2023/08/10(木) 21:50:45.95 ID:???.net
>>538
重心に働く力のベクトルが接点を通らなければ
トルクが発生する事くらい分からん?

550 :ご冗談でしょう?名無しさん:2023/08/11(金) 01:54:38.99 ID:wcbc4zhR.net
>>544
ちなみに摩擦力が0だと車輪は回らずに滑り出す

551 :ご冗談でしょう?名無しさん:2023/08/11(金) 13:07:34.67 ID:???.net
米フェルミ国立加速器研究所などの国際チームは10日、
素粒子の一種「ミュー粒子」の性質が、
現代物理学の基盤である「標準理論」の予測の一部から
大きくずれているとの実験結果を発表した。
ずれが確定すれば、
標準理論では想定されていない
未知の粒子の存在を示す成果という。

ミュー粒子は磁力を持ち、
質量は電子の約200倍と重い。
実験では、同研究所内に設置したリング状の巨大磁石(直径14メートル)の内部にミュー粒子を送り、
高速で回転させて磁力を測定した。
標準理論に基づき予測されるミュー粒子の磁力と比べ、差が大きいことを確認した。

https://www.yomiuri.co.jp/science/20230811-OYT1T50083/

どういう意味よ

552 :ご冗談でしょう?名無しさん:2023/08/11(金) 13:14:06.10 ID:???.net
標準理論での計算が間違ってた可能性もあるんだぜ

553 :ご冗談でしょう?名無しさん:2023/08/11(金) 13:20:15.18 ID:EhZobmcY.net
https://pbs.twimg.com/media/F3Nu_dua8AAxxpX.jpg
     

554 :ご冗談でしょう?名無しさん:2023/08/11(金) 14:08:35.46 ID:qU5PqmNw.net
>>553
'もう338円分も増えた'

555 :ご冗談でしょう?名無しさん:2023/08/11(金) 14:08:52.97 ID:???.net
>>551
ほう
どういう意味?

556 :ご冗談でしょう?名無しさん:2023/08/12(土) 05:34:41.85 ID:???.net
ピカー

557 :無学な私:2023/08/12(土) 09:25:08.60 ID:J/0CiP+w.net
ロケットの速度の上限について

燃焼エンジン、イオンエンジン、ソーラーセイル、等々、
これらは、電磁的な反作用を利用しているので、得られる推力の上限は、
電磁気力の伝搬速度に制限される

この説明は、概ね正しいですか?

また、相対論的に物体を光速まで加速できない事と関連はありますか?

558 :ご冗談でしょう?名無しさん:2023/08/12(土) 12:26:35.63 ID:???.net
体重が120kgあるんですが
床に座って作業してます
ダンボールを敷くと楽になるんですが、クッションだとキツイです
ハニカム構造?のゲルクッションを利用しても腰がきつくなります
ダンボールがベストなのですが
ダンボールだと数日で完全に潰れてしまいます・・・
潰れないようにする方法
もしくは
ダンボールに代わる物って何か無いでしょうか?

559 :ご冗談でしょう?名無しさん:2023/08/12(土) 12:40:54.32 ID:???.net
情報不足
ダンボールのどこが良いのか書かなきゃ答えはない

560 :無学な私:2023/08/12(土) 14:28:57.79 ID:HY3lDGmD.net
>>558
段ボールが潰れる事が、エネルギーを吸収している証拠なんだから、
定期的に段ボールを取り替えるのが良いでしょう

561 :ご冗談でしょう?名無しさん:2023/08/12(土) 14:53:42.54 ID:QtwVnWSg.net
弾性を簡単に変化させられるものないかなと考えたことある。

一番は空気圧かなぁと。ただ難しいのは変形の大きさに応じて求める反発力が微妙に変わるとしたら、果たしてどんな構造が考えられるかなとか。

圧縮空気を電子制御で出し入れしてやれば目的の弾性が得られるだろうとか思うけど、そんな健康商品無理だなとか。

562 :ご冗談でしょう?名無しさん:2023/08/12(土) 15:49:07.11 ID:552sg3HI.net
>>561
可塑性のあるギブスってあるよね。
空気を入れると固くなり、抜くと柔らかくなるやつ。
リハビリがやりやすくなる。

563 :ご冗談でしょう?名無しさん:2023/08/12(土) 16:53:11.42 ID:QtwVnWSg.net
指圧とかも弾性を可変にできれば、本格的指圧マッサージ機になると思うんだ。人間みたいに押して、ちょっと反発強いかなと思ったら、圧力下げたり、その微調整できればさ。

564 :ご冗談でしょう?名無しさん:2023/08/12(土) 20:19:14.21 ID:???.net
>>558
座布団、ござ、ミニ畳
あと座椅子とか見てみたら?

565 :ご冗談でしょう?名無しさん:2023/08/13(日) 06:06:01.73 ID:???.net
ムニちゃんポポだのんのおじさんはすごいんだぞ

566 :ご冗談でしょう?名無しさん:2023/08/13(日) 06:19:22.40 ID:???.net
やばっ、思いっきり舌噛んだ

567 :ご冗談でしょう?名無しさん:2023/08/13(日) 06:24:08.98 ID:Y9Id/5yv.net
>>558
少し食べる量を減らして体重を減らすのが1番いいかも

568 :ご冗談でしょう?名無しさん:2023/08/13(日) 08:40:11.21 ID:???.net
高校生でも分かる相対性理論の夏休み練習問題を出してやろう

自動車のスピードメーターの原理は地面に接した車輪の単位時間当たりの回転数になる
1秒間に1回転すると1m自動車が運動するとすれば1m/sと表示される。

思考実験の自動車が加速しながら平面を直進すればスピードメーターの制限速度である
光速を超えられるか?
@ 光速を超えられる
A 光速を超えられない
相対性理論ではどちらが正しいか

569 :poem:2023/08/13(日) 10:21:21.81 ID:YG2cnn2H.net
>>568 さんちょっと横入りごめん
自分のはどうせコメント無いはず
今ネット検索してて知ったこと
「音は風上に大きく届き、風下に小さく届く」
って初めて知った逆かと思ってた
で「振幅が違くなる」って説明されてるけど自分は間違いだと思う「届くタイミングが早くなる(音速が速くなる)」この理由と大きく聞こえる原理なんだけど
まず「風が止まってるときに音源が移動する、のと風が吹く音源が止まるは違う」これは「風が吹くときは空気に静圧だけでなく動圧が発生してる」かつ「風と音源の速度差は音源が感じる動圧だが、風が吹くときは大気全体の動圧。この2つの違いは動圧の発生範囲が音源が移動するのは局所、音源のみに風の動圧があり、風が動くときは大所、大気に動圧がある。だから違う」
そしてコルクの栓を抜くとスポンと音が鳴るのは「風は継続速度m/sだが大気圧は瞬間速度msであるから」「コルクを抜くとそこだけ大気がないから大気が叩かれる。これは瞬間速度だから一瞬ほど大きな大気の速度になる。ようは大気は無いところがあると駄目で、地球辺縁を空気が一個の固体のようになっていて、無いところは叩かれる」
から風上に音が早く到達する理由は
「大気に真空が起こってはいけないから風が吹いたら玉突きのように風が突然途切れることがない。だから風は続く。ここで風の動圧が起こると後ろに真空になりそうな力があるけど真空にならない、とすると真空になるとコルクの栓を抜いたように大気が叩かれる。だから大気が叩かれる発生する速さで、音が速くなる」もう少し「音速は大気中速さだから、風が起こると風上は大気の風下への移動の分風上への音速が遅くなる。これを加味した上で風上へ真空成分により叩かれて結果的に音速が速くなる。風下へは遅くなる。これらは大気全体の移動音源停止の大所だから起きることであり、音源のみが移動する局所では起こりえない」

と予想したの正解ジャッジ求む

570 :poem:2023/08/13(日) 10:26:13.05 ID:YG2cnn2H.net
>>568
ニュートリノは水中光速は少なくとも超えられるはチェレンコフ光から
ニュートリノは真空光速も超えられると予想
だから相対性理論は誤り
だからそれは光速だせる

571 :poem:2023/08/13(日) 10:33:16.27 ID:YG2cnn2H.net
光速はクォークの物理
/m何かの変位度の距離成分当たり
s留まる時間のクォーク物理が
1/c数値時間だから
光速度になるクォークの固有状態が光速を産む実は遅度であり
固有状態だから定速で光によりどんな光源からも全部光速は同じ速度
普通の速度と相互作用しない光速不変は遅度と速度が逆数の加減算だからで
2つを説破
固有状態な光は光速未満にも以上にもならないが
ニュートリノは固有状態で無く粒子の飛行だから光速未満にも以上にもなる
だからニュートリノが光速以上だせれば
相対性理論誤りが実証でもあるから
その問題は光速出せる

572 :ご冗談でしょう?名無しさん:2023/08/13(日) 10:35:42.78 ID:???.net
キチガイしか湧かないな

573 :poem:2023/08/13(日) 10:37:40.82 ID:YG2cnn2H.net
キチガイ
✊👁_👁✊グー✌👁_👁✌チョキ👌👁_👁👌おけおけ✋👁_👁✋あたまぱぁ

574 :ご冗談でしょう?名無しさん:2023/08/13(日) 11:29:11.26 ID:Ws/DTEdU.net
相対論は光速以上のモノを否定していない。
光速以下のモノが光速以上に加速されることを否定している。
始めから光速なら仕方ない。それでも光速は一定である。

575 :ご冗談でしょう?名無しさん:2023/08/13(日) 11:44:33.96 ID:???.net
>>574
>光速以下のモノが光速以上に加速されることを否定している。

それが >>568 の解答だというなら

(特殊)相対性理論による >>568 の問題の正解は
@ 光速を超えられる

このバラドックが解ける優秀な高校生には東大京大の推薦入学を認めるべき

576 :ご冗談でしょう?名無しさん:2023/08/13(日) 12:19:28.65 ID:???.net
ねぇねぇ
呪術廻戦でアキレスと亀って話があったけど
なんでアキレスは亀に追いつけないの?

577 :ご冗談でしょう?名無しさん:2023/08/13(日) 12:30:47.50 ID:???.net
>>576
無限に分割して無限に足し算するという数学概念が理解できない低脳・キチガイには
解けないだけ

つまり、低脳は亀に追いつけない と言うのが正しい。

578 :ご冗談でしょう?名無しさん:2023/08/13(日) 12:31:55.21 ID:???.net
追いつくまでの有限時間を無限に分割してるだけだから

579 :ご冗談でしょう?名無しさん:2023/08/13(日) 12:49:19.10 ID:???.net
なんでいきなり低脳呼ばわりしてんの?
頭大丈夫?

580 :ご冗談でしょう?名無しさん:2023/08/13(日) 13:07:44.23 ID:???.net
>>577
いや、普通に早いほうが遅い方を追いかけたら追いつくよね

581 :poem:2023/08/13(日) 13:09:35.60 ID:YG2cnn2H.net
>>578の説明文に同意

582 :ご冗談でしょう?名無しさん:2023/08/13(日) 13:18:08.49 ID:???.net
>>580
>普通に早いほうが遅い方を追いかけたら追いつく
だからパラドックスという

お前はアキレスと亀問題の内容すら理解できないようだから
詳しい内容説明を100回読め、それでもダメなら諦めろ

583 :ご冗談でしょう?名無しさん:2023/08/13(日) 13:22:48.26 ID:Ws/DTEdU.net
「アキレスと亀」のパラドクスのキモは、物理的にどうとか、無限級数の収束がどう
とかという問題ではない。
「ゼノンの推論方法を真正面から論破できるか?」という問題だ。

すなわち、
「アキレスは追い着く以前に、亀が走りはじめた点に着かなければならず、したがって、
亀は常にいくらかずつ先んじているので、アキレスは亀に追いつけない」
という推論を正面から否定できるのか?

584 :ご冗談でしょう?名無しさん:2023/08/13(日) 13:43:24.38 ID:Ws/DTEdU.net
>>583
これのスッキリした解答のひとつは、「長さは連続ではなく、最小単位がある」というものだ。
そうすれば、最後の最小単位のところでアキレスは亀に追いつき、次の最小単位で亀を超える
ことができ、論理の矛盾はない。
この連続の否定こそ、ゼノンが突きつけた問題である。

585 :ご冗談でしょう?名無しさん:2023/08/13(日) 13:49:53.17 ID:???.net
>>583
1/2 + 1/4 + 1/8 + 1/16 + ........
無限の足し算ができない、または 1より小さい。 とニュートン以前の人は信じてただけ。

現代数学では
1 = 1/2 + 1/4 + 1/8 + 1/16 + ........
であり、矛盾が起こらない数学体系になっている。

古代人のように無限の足し算出来ないと執着するか、出来る数学を学習するかは
個人の自由だ!

586 :ご冗談でしょう?名無しさん:2023/08/13(日) 13:53:13.16 ID:???.net
>>584
長さの゙最小単位があると仮定しなきゃ説明できないような理屈なら、それは何一つ解けてないってことだ

587 :ご冗談でしょう?名無しさん:2023/08/13(日) 13:56:13.85 ID:???.net
>>584
お前のような義務教育もマトモに受けてないキチガイ馬鹿が
思いつきで「最小単位」とかわめいても矛盾だらけになることを
2000年以上前の古代ギリシャ人ピタゴラスが証明している。

588 :ご冗談でしょう?名無しさん:2023/08/13(日) 13:57:11.18 ID:Ws/DTEdU.net
>>585
今のところ、どちらを信じるかの問題ではある。
その決着を事実に求めるのは、一見、順当であるかのようだが、
そこには思考の飛躍があって、知性が及んでいない。

無限は依然として人の知性が及ばないところであり、
個人の選択の問題以上のものだ。

589 :ご冗談でしょう?名無しさん:2023/08/13(日) 14:01:05.24 ID:Ws/DTEdU.net
>>587
連続と無限をどのように扱うかは、近/現代数学の主要な問題であった。
一見、それは解決されたかのように見えるがそうではない。
解決したと思いたいのは、>>587のように思考の怠惰に他ならない。

590 :ご冗談でしょう?名無しさん:2023/08/13(日) 14:03:26.16 ID:???.net
>>588
そうだよ
お前のような義務教育もマトモに受けてないキチガイの知性が及ばないだけだ。

591 :ご冗談でしょう?名無しさん:2023/08/13(日) 14:03:52.91 ID:Ws/DTEdU.net
>>585
この主張も、結局、ゼノンの論法に正面から反論できていない。

592 :ご冗談でしょう?名無しさん:2023/08/13(日) 14:04:38.48 ID:???.net
解決されていない問題だという妄想が出てくるのですね

593 :ご冗談でしょう?名無しさん:2023/08/13(日) 14:05:14.73 ID:???.net
>>589
キチガイ馬鹿がコピペしてるだけだろ

594 :ご冗談でしょう?名無しさん:2023/08/13(日) 14:05:24.10 ID:Ws/DTEdU.net
>590
大口を叩いているが、正面から論破できていないのだから、
単なる虚勢にすぎない。

595 :ご冗談でしょう?名無しさん:2023/08/13(日) 14:08:42.05 ID:Ws/DTEdU.net
>>592 >>593
妄想でも、コピペではないが、仮にそうだとしても、
正面から論破できないのはだらしない。

596 :ご冗談でしょう?名無しさん:2023/08/13(日) 14:09:07.95 ID:???.net
>>594
(笑)コピペしかできないんだろ

597 :ご冗談でしょう?名無しさん:2023/08/13(日) 14:12:02.92 ID:Ws/DTEdU.net
不甲斐ないなぁ。
結局、正面から論破できてる推論は、>>584しかないではないかw
ってか、他には誰も試みてすらいないw

598 :ご冗談でしょう?名無しさん:2023/08/13(日) 14:12:17.65 ID:???.net
ゼノンのパラドクスの゙解放なんてググればいくらでも出てくるが、それとは違うよくわからん妄想を論破することはできない

599 :ご冗談でしょう?名無しさん:2023/08/13(日) 14:14:31.33 ID:Ws/DTEdU.net
>>598
この人とコピペ体質ね。自分で考えずにググってばかりw
自分の思考を話せないw

600 :ご冗談でしょう?名無しさん:2023/08/13(日) 14:15:58.20 ID:???.net
妄想上の゙問題を論破できるのは妄想してる本人だけ

601 :ご冗談でしょう?名無しさん:2023/08/13(日) 14:17:38.43 ID:Ws/DTEdU.net
人身攻撃を専らとし、議論の内容にひとつも踏み込まないのは、頭の悪さの証左w

602 :ご冗談でしょう?名無しさん:2023/08/13(日) 14:19:57.75 ID:???.net
議論なんて存在してないからね
何を問題にしてるのか本人以外誰もわからない

603 :ご冗談でしょう?名無しさん:2023/08/13(日) 14:22:13.58 ID:Ws/DTEdU.net
>>602
頭の悪さを韜晦する哀しさよ。
ゼノンの議論まんまなのだから、何を問題にしてるのか解らないのは頭の悪さw

604 :ご冗談でしょう?名無しさん:2023/08/13(日) 14:24:27.48 ID:???.net
キチガイは物理・数学の理論というものがまったく理解できない
正しい理論の必要条件とは矛盾が起こらないことだ。

ニュートン力学や相対性理論などでは距離、時間に最小値があると論理矛盾する。
また
ニュートン力学では速度に上限値があると論理矛盾する。
特殊相対性理論では速度の上限値cがないと論理矛盾する。
しかし
理論的にニュートン力学と特殊相対性理論のどちらも論理的に正しい。

目的よってどの理論を選択するかだけの問題なのだ。

605 :ご冗談でしょう?名無しさん:2023/08/13(日) 14:28:07.84 ID:???.net
>>603
ゼノンのパラドックスそのままなら解決など山のように出てくるが、それの何が不満なのかがさっぱり見えてこない
つまりおまえはゼノンのパラドックスを騙った妄想上の未解決問題を作り出してるだけだ

606 :ご冗談でしょう?名無しさん:2023/08/13(日) 14:28:11.35 ID:Ws/DTEdU.net
>>604
そんなことを話しているのではない。
>>583でちゃんと釘をさしてたのに、詰まらない話を始めたw

607 :ご冗談でしょう?名無しさん:2023/08/13(日) 14:30:46.57 ID:Ws/DTEdU.net
>>605
「山のようにある」のなら、ゼノンの論法を「真正面から」論破した
具体例をひとつ見せてくれw

608 :ご冗談でしょう?名無しさん:2023/08/13(日) 14:41:06.39 ID:???.net
>>607
逆になんでもいいからググってみて真正面から論破(解決)していないという理由を述べれば?
どのサイトでも普通に説明してんだから。
それが真正面からの解決じゃないなんておまえがいくら言ってみたところで何の意味もないことだ。

609 :ご冗談でしょう?名無しさん:2023/08/13(日) 14:41:35.56 ID:???.net
>>606-607
お前の厨房オレ様説は簡単もいいとこだろが

”有限の個数しか数えられない””有限個しか計算できない”

そんだけ

610 :ご冗談でしょう?名無しさん:2023/08/13(日) 14:44:06.19 ID:Ws/DTEdU.net
>>608
あくまで逃げる。
韜晦しても頭の悪さは隠せないw

611 :ご冗談でしょう?名無しさん:2023/08/13(日) 14:46:26.78 ID:Ws/DTEdU.net
>>609
あまりにも舌足らずのたどだしい反論(?)。

>”有限の個数しか数えられない””有限個しか計算できない”
>そんだけ

これで何かを論じたと思ってる幼稚なアタマw

612 :ご冗談でしょう?名無しさん:2023/08/13(日) 14:49:32.72 ID:???.net
>>610
お前のような、>>609厨房キチガイは無敵だ

と認めてやるからさっさと物理板から消えてくれ

613 :ご冗談でしょう?名無しさん:2023/08/13(日) 14:56:51.04 ID:Ws/DTEdU.net
>>612
結局、議論の中身には一歩も踏み込まない。
逃げるのは腹立たしいので、こんどは追い出しかw
アタマが悪いだけでなく、小心/臆病な奴だ。

614 :ご冗談でしょう?名無しさん:2023/08/13(日) 15:09:49.22 ID:F/BPMOTE.net
>>553
Amazonギフトに交換できるのが嬉しい

615 :ご冗談でしょう?名無しさん:2023/08/13(日) 16:04:10.97 ID:???.net
人間もコンピュータも無限に足し算の操作を繰り返すことは現実に出来ない
そこで有限個しかないと誤魔化して矛盾だらけの結果を知らんぷりするか。

それとも矛盾が起こらない無限の分割、加算の数学理論を発明するかだ
アルキメデス、ニュートンの発明に続き19世紀に微分積分学が完成した
人類の偉大な勝利。

616 :ご冗談でしょう?名無しさん:2023/08/13(日) 16:33:35.89 ID:Ws/DTEdU.net
>有限個しかないと誤魔化して
 →有限個とするのは、べつに誤魔化しではない。

>矛盾だらけの結果を知らんぷりするか
 →無限にだって矛盾はある。
 
無限小については、「ε-δ論法」で解決したと言われるが、
これは解決ではなく、無限を避けたこと。上手くいってるようには見える。

無限大については、実質、カントールの考察以上のものは無いだろうが、
ブラウワーなどの直観主義も依然として否定されえない。

ラッセルのパラドクスも止揚されたと言うが、そうでもない。

617 :ご冗談でしょう?名無しさん:2023/08/13(日) 16:45:06.16 ID:???.net
>>616
(笑)コピペしかできない厨房キチガイ

スレ荒らしてないで夜間中学からでもやり直せ

618 :ご冗談でしょう?名無しさん:2023/08/13(日) 16:49:36.69 ID:Ws/DTEdU.net
>>617
コピペ? 論点を示してるだけだろ。
新しい話題を示したらコピペとな?w
その違いが解らないのはアタマが悪いか、よほどの無知w

619 :ご冗談でしょう?名無しさん:2023/08/13(日) 16:55:48.14 ID:???.net
最小ナンチャラのキチガイ馬鹿は意識霊魂キチガイだったのか、IDでバレバレ

620 :ご冗談でしょう?名無しさん:2023/08/13(日) 16:58:52.94 ID:???.net
しょせん言葉遊び

621 :ご冗談でしょう?名無しさん:2023/08/13(日) 17:00:41.26 ID:???.net
物理板にオカルト、キチガイが棲みつくのは何故かな

622 :ご冗談でしょう?名無しさん:2023/08/13(日) 17:19:32.57 ID:???.net
キチガイは微積分学がなければ円の面積が厳密にπr^2になるのも証明できない
ことさえ全く解らない。
当たり前だが有限個の足し算では証明できない。

オカルトキチガイになりたくないなら苦労を厭わず勉強しようね。

623 :ご冗談でしょう?名無しさん:2023/08/13(日) 20:53:19.66 ID:dkzxXb5A.net
真空中にボールを投げて、このボールの回転軸の方向を変えたいとき、どんな方法がありえますか?

624 :ご冗談でしょう?名無しさん:2023/08/13(日) 21:04:03.65 ID:???.net
>>623
空気中の場合と大して変わりません

625 :ご冗談でしょう?名無しさん:2023/08/13(日) 22:12:40.69 ID:omoiYL/+.net
そうだよ、無限の分割というのがすでにおかしいのだよ
最小単位が存在するとすればすっきり解決するのだよ
微積分とは一種の最小単位肯定でもあるのだよ

626 :ご冗談でしょう?名無しさん:2023/08/13(日) 22:15:44.31 ID:omoiYL/+.net
0ではない最小単位を肯定することで解法とする
これが微積分なのだから
つまり0は存在しない

627 :ご冗談でしょう?名無しさん:2023/08/13(日) 22:17:10.79 ID:omoiYL/+.net
慣性質量スレから出張したったわ

628 :ご冗談でしょう?名無しさん:2023/08/13(日) 22:21:27.70 ID:omoiYL/+.net
がちアホは微積分は無限と0を肯定するためのものと勘違いしている
逆だ!

629 :ご冗談でしょう?名無しさん:2023/08/13(日) 23:12:15.18 ID:???.net
「アキレスと亀」のキモは
単に追いつくところまで言わないだけ
追いつく手前に無限の言葉を並べて
その先が無いかのように誤魔化す

言葉を無限に並べる方法を使えば
どんな事でも不可能に見せかけられるのさ

630 :無学な私:2023/08/13(日) 23:25:55.74 ID:Qz7NEHgH.net
素人が思い付いた解決案だが

そもそも、時間は分割できないと定義してみてはどうか?

631 :ご冗談でしょう?名無しさん:2023/08/13(日) 23:26:38.77 ID:omoiYL/+.net
つまり、無限大は無い
よって0も無いということですね

632 :ご冗談でしょう?名無しさん:2023/08/14(月) 00:17:41.43 ID:???.net
しょせん言葉遊び

633 :無学な私:2023/08/14(月) 02:27:55.95 ID:+xBYLOEX.net
素人が考えた解決案2

アキレスと亀は並ばないと定義する

634 :ご冗談でしょう?名無しさん:2023/08/14(月) 05:27:52.35 ID:???.net
池沼は池沼隔離スレから出てくるな

635 :ご冗談でしょう?名無しさん:2023/08/14(月) 07:27:22.37 ID:ZwE2Aodm.net
わはは、受け売りの痴呆ボロ負けざまぁw

636 :ご冗談でしょう?名無しさん:2023/08/14(月) 07:34:54.12 ID:ZwE2Aodm.net
受け売り野郎、こんなところで巣を作って人を喰ったようなふざけたことしてやがったのか!

637 :ご冗談でしょう?名無しさん:2023/08/14(月) 09:44:22.04 ID:???.net
>>636
(笑)コピペしか出来ないお前が”受け売り”とわめき散らす

お前のキチガイ脳が矛盾してるのが自分で判らんのか、キチガイだからな

638 :ご冗談でしょう?名無しさん:2023/08/14(月) 09:59:15.87 ID:???.net
キチガイにならないように数学の基本から理解しようね

有理数が前提条件として、0より大きい最小の有理数の値はあるか?

最小の有理数があると仮定しよう、nを任意の自然数として
最小の有理数を1/nとすると 1/(n+1) < 1/n だから矛盾する

正解は 0より大きい最小の有理数の値はない。

639 :ご冗談でしょう?名無しさん:2023/08/14(月) 10:18:28.37 ID:???.net
>>638
補足
 ”最小の有理数がない”と書かずに”最小の有理数の値はない”と親切で言い換えてる
ない(無い)だけだと”全くなにも無い”などと勘違いするキチガイがいるからだ。

640 :ご冗談でしょう?名無しさん:2023/08/14(月) 10:40:24.76 ID:???.net
>>639
現代物理学によれば”電子の大きさがない”という仮定も健常者には理解できるだろ
電子に大きさ(不連続になる殻)があると理論が矛盾するという意味だ。

641 :ご冗談でしょう?名無しさん:2023/08/14(月) 11:29:05.41 ID:???.net
逆だろ
電子の大きさにまで理論を適用すると
矛盾する

642 :ご冗談でしょう?名無しさん:2023/08/14(月) 11:55:08.47 ID:???.net
>>641
そう言うなら、電子に大きさ(不連続になる殻)が有るという
(数学、相対論等と)矛盾が無い新理論を作ればいいだろ。

作れないなら”電子に大きさがある!”も無意味だ

643 :ご冗談でしょう?名無しさん:2023/08/14(月) 12:00:30.39 ID:???.net
超弦理論とかで”電子が矛盾なく記述できた”などというニュースは聞いたことがない。

644 :ご冗談でしょう?名無しさん:2023/08/14(月) 16:36:59.74 ID:???.net
>>642
大きさが無いも無意味だぜ
どっちにしろ矛盾してんだからな

645 :ご冗談でしょう?名無しさん:2023/08/14(月) 16:44:01.43 ID:???.net
超弦理論に対する素人の疑問
1.超弦理論のいう本来の重力の大部分がコンパクト化した次元に行くけどそこは重力崩壊とかしないの?
2.逆2乗の法則からこの世界は3次元空間じゃないの?
3.宇宙誕生時には4つの力は同等で重力が分かれた時に余剰次元ができたってこと?
4.万有引力定数は不変だから閉じたひもがこの4次元時空間に及ぼす影響はいつも同じってことになるの?

646 :ご冗談でしょう?名無しさん:2023/08/14(月) 17:28:44.35 ID:???.net
>>644
>大きさが無い
お前も何度説明しても電子の’大きさがない'という数学や物理の意味が
理解できない奴と同じだ。

647 :ご冗談でしょう?名無しさん:2023/08/14(月) 17:30:32.20 ID:???.net
馬鹿は0か1かしか考えられない

648 :ご冗談でしょう?名無しさん:2023/08/14(月) 17:34:23.25 ID:W92FwPlP.net
どの立場にも言えることですが、反論するときは論点を整理して具体的に反論できることについて書きましょう。
印象操作だけでは反論になってません。

649 :ご冗談でしょう?名無しさん:2023/08/14(月) 17:36:18.04 ID:W92FwPlP.net
反論だけでなく、ある論述をする場合も全く同じです。
論点を整理して具体的に主張できることについて書きましょう。
印象操作だけでは論述になってません。

650 :ご冗談でしょう?名無しさん:2023/08/14(月) 18:33:59.11 ID:???.net
>>649
お前はキチガイのオカルト野郎だろ、いつまで物理板に巣くってんだよ

物理は弁論大会ではない義務教育も済んでないなら夜間中学からやり直せ

651 :ご冗談でしょう?名無しさん:2023/08/14(月) 18:36:13.06 ID:W92FwPlP.net
>>650
あなたの書き込みは全く理解できません。>>649のどこにオカルト要素があるのか説明してください。

652 :ご冗談でしょう?名無しさん:2023/08/14(月) 18:51:15.74 ID:???.net
無限は数学で上手く手懐けれ最強なのだよ、低脳には数学が理解不能なだけ

むしろ宇宙に無限が必要不可欠だともいえる

653 :ご冗談でしょう?名無しさん:2023/08/14(月) 18:59:06.78 ID:???.net
無限の宇宙の前では’人間が存在するのは神の奇跡’など簡単に吹き飛んでしまう

654 :ご冗談でしょう?名無しさん:2023/08/15(火) 00:11:01.45 ID:???.net
>>649
お前もどれが印象操作なのか明記しろよ

655 :無学な私:2023/08/15(火) 02:06:38.33 ID:yvck+EIW.net
>>647
宇宙の真理は1と0の可能性はあるから

656 :ご冗談でしょう?名無しさん:2023/08/15(火) 02:51:13.80 ID:???.net
引き籠って3Dゲームばっかやってると0と1しか分からないキチガイ脳になる

657 :ご冗談でしょう?名無しさん:2023/08/15(火) 09:18:40.08 ID:PgSLrQjk.net
標準理論も不完全極まりないからね

658 :ご冗談でしょう?名無しさん:2023/08/15(火) 09:20:23.38 ID:???.net
ダンシングムーニー

659 :ご冗談でしょう?名無しさん:2023/08/15(火) 09:32:30.15 ID:???.net
>>652
>無限は数学で上手く手懐ける
エネルギーの無限大問題
電子の大きさ(殻の半径)の一定値が無いとエネルギーが無限大になるから矛盾する。
簡単に
2つの電子同士を近づけて重なる極限は高校生でも計算できる 1/r,r->0 なら∞
ところが
量子論の電子にはパウリの排他原理が成り立つ
2つ以上のフェルミ粒子は、同一の量子状態を占めることはできない。

つまり、無限大のエネルギーは”始めから矛盾した計算をした結果”ということ
これは1個の電子のエネルギー計算についてもいえる。
現実のエネルギー計算などは「繰り込み理論」により非常に正確に計算されている。

660 :ご冗談でしょう?名無しさん:2023/08/15(火) 09:52:46.15 ID:VnS2fFHE.net
どんな物理理論も近似理論だ。近似の範囲でしか有効性を主張しない。
理論には前提やら仮定やらがあり、それを無視しての主張や批判をするのは的外れ。

661 :ご冗談でしょう?名無しさん:2023/08/15(火) 09:58:12.04 ID:???.net
>>657
具体的に不完全極まりない理由は?

662 :ご冗談でしょう?名無しさん:2023/08/15(火) 10:15:12.79 ID:???.net
>>660
>理論には前提やら仮定やらがあり
そうだが、キチガイは「〜あります!」だけしか言えないからゴミ箱行き

古典物理の電子理論では古典電子半径というのが仮定として論理矛盾を回避する。
現代でも目的によって使ってる。

量子論では電子の大きさ(殻の半径)の一定値が無いという仮定で論理矛盾を回避する


ニュートン力学では物体の速度に制限値がないことで論理矛盾を回避している。

特殊相対性理論では物体の速度に制限値がある(c)ことで論理矛盾を回避している。

物理学の理論体系が キチガイ、相間、量間の脳ミソは全く理解できない。

663 :ご冗談でしょう?名無しさん:2023/08/15(火) 13:14:21.09 ID:VnS2fFHE.net
>>662
>そうだが、キチガイは「〜あります!」だけしか言えないからゴミ箱行き

せっかちなヤローだな。徐々に言おうとしてるのに。
相手の否定から入って得意になる厨房。

>論理矛盾を回避している。

どんな矛盾を回避してるかを言えないからゴミ箱行きw

664 :ご冗談でしょう?名無しさん:2023/08/15(火) 13:39:11.41 ID:???.net
ある地点の次元を測定する実験方法はありますか
3.0000次元とか3.0002次元のように
整数値からズレた分が量子効果になるのでは

665 :ご冗談でしょう?名無しさん:2023/08/15(火) 15:22:54.99 ID:???.net
>>663
>どんな矛盾を回避してるか
お前のように義務教育もマトモでない奴が電磁気学の教科書が理解できるわけない
まして場の量子論の教科書の理解など不可能だ。

>徐々に言おうとしてるのに。
キチガイは「〜あります!」だけしか言えないのを誤魔化してるだけ
それともスレ荒らしでわめけば誰かがヨイショしてくれるとでも期待してるのか?

>>664
>3.0000次元とか3.0002次元
新手の「〜あります!」キチガイ

666 :ご冗談でしょう?名無しさん:2023/08/15(火) 15:38:35.22 ID:???.net
ガウスの法則(クーロン逆2乗法則)がどれだけの精度で成り立つのかの実験

静電気で帯電した金属導体の殻の内部では電場が理論的に0になる
内部の電場の強さを精密に測定すればよい、マックスウェルも自分で実験した

667 :ご冗談でしょう?名無しさん:2023/08/15(火) 15:56:09.51 ID:VnS2fFHE.net
>>665
なぁ〜んだ。説明できないでやんのw
鼻息荒く誤魔化してるだけじゃん。トンダ三枚目w

668 :ご冗談でしょう?名無しさん:2023/08/15(火) 16:12:03.66 ID:???.net
>>664
2つの物体の間に働く万有引力を測ればいい。
距離の2乗に比例していれば3次元、
n乗に比例してたらn+1次元

669 :ご冗談でしょう?名無しさん:2023/08/15(火) 16:15:44.56 ID:FGPlBc1a.net
あり〼

670 :ご冗談でしょう?名無しさん:2023/08/15(火) 16:35:47.56 ID:???.net
>>664
非可換幾何のほうが筋がいい

671 :ご冗談でしょう?名無しさん:2023/08/15(火) 16:56:38.02 ID:???.net
>>668
頭悪い奴の素人考え

重力は弱すぎるしクーロン力も直接に距離と力を測るやり方は精度が粗い

672 :ご冗談でしょう?名無しさん:2023/08/15(火) 17:07:54.57 ID:???.net
ニュートンやマックスウェルは理論の天才だけでなく物理実験の名人でもあるのだ

673 :ご冗談でしょう?名無しさん:2023/08/15(火) 17:08:25.96 ID:???.net
>>668
なるほど、頭いいと思ったが,
>>671さんのいうように測るのが難しい

674 :ご冗談でしょう?名無しさん:2023/08/15(火) 17:30:16.24 ID:???.net
現実の空間が完全一様な3次元空間ならば逆2乗の法則が厳密に成り立たなければならない
ということが理解できる。

675 :ご冗談でしょう?名無しさん:2023/08/15(火) 18:08:12.94 ID:UFUOc3th.net
ジャクソンの電磁気の最初のほうに逆二乗からのズレを計測する話がかいてあるな
現代ではものすごい精度で確認されてる

676 :ご冗談でしょう?名無しさん:2023/08/15(火) 18:12:04.43 ID:???.net
>>671
現在の科学技術舐めすぎw

677 :ご冗談でしょう?名無しさん:2023/08/15(火) 18:54:14.79 ID:???.net
頭悪いとか人のアイデア罵倒する前に、よりよいアイデア当然持ってるんだろうからまずそれを開示すればいいのに

678 :ご冗談でしょう?名無しさん:2023/08/15(火) 19:20:02.15 ID:???.net
5ちゃんで開示してもお金にならないからやらないだけ。

679 :ご冗談でしょう?名無しさん:2023/08/15(火) 19:44:52.44 ID:???.net
>>678
誰よりも一番頭の悪いやつがやるレスやんw

680 :ご冗談でしょう?名無しさん:2023/08/15(火) 19:54:07.30 ID:???.net
俺のように電磁波を打ち消すアイデアで1000万円でも稼いでみろ

681 :ご冗談でしょう?名無しさん:2023/08/15(火) 21:34:38.04 ID:???.net
現代人は病んでるからキチガイにならないよう論理思考を鍛え、実益を兼ね揃えた
物理学の基本学習が最適。

それなのに物理板にキチガイが巣くってしまうのは何故かな。

682 :ご冗談でしょう?名無しさん:2023/08/15(火) 23:03:55.00 ID:???.net
自己承認欲求

683 :ご冗談でしょう?名無しさん:2023/08/16(水) 00:15:47.99 ID:???.net
>>681
そりゃキチガイにならないために物理学の基本学習が最適という、きみの前提がまるで間違ってるということだろう

684 :ご冗談でしょう?名無しさん:2023/08/16(水) 01:50:19.15 ID:???.net
学習?してるか?
せん奴だろ

685 :ご冗談でしょう?名無しさん:2023/08/16(水) 04:18:34.18 ID:02a6Eiub.net
>>681
そう言っているお前さんが一番のキチガイであることに気付けよ!w

686 :ご冗談でしょう?名無しさん:2023/08/16(水) 04:21:30.82 ID:02a6Eiub.net
まあ、物理板にキチガイが巣くうというのは正しい
そして、一番のキチガイが>>681ということ

687 :ご冗談でしょう?名無しさん:2023/08/16(水) 12:00:59.71 ID:U5UqOMh2.net
平面上の点 P を考える。
座標系 O-x, y とそれを原点を中心として θ だけ回転させた座標系 O-x', y' を考える。
座標系 O-x, y での点 P の座標を (x, y)
座標系 O-x', y' での点 P の座標を (x', y')
とする。

このとき、

x = x' * cos θ - y' * sin θ
y = x' * sin θ + y' * cos θ

が成り立つ。

このことを以下のように説明している本を見かけませんがなぜでしょうか?

x 軸と線分 OP のなす角を φ とする。
このとき、 x' 軸と線分 OP のなす角は θ + φ である。
点 Q を x' 軸と線分 OQ のなす角が φ であるような点とする。
座標系 O-x', y' での点 Q の座標は (x, y) である。
点 P は点 Q を原点を中心として θ だけ回転させた位置にある。

よって、

x = x' * cos θ - y' * sin θ
y = x' * sin θ + y' * cos θ

が成り立つ。

688 :ご冗談でしょう?名無しさん:2023/08/16(水) 12:57:24.35 ID:???.net
まどろっこしい

689 :ご冗談でしょう?名無しさん:2023/08/16(水) 14:26:42.10 ID:???.net
ピカー

690 :ご冗談でしょう?名無しさん:2023/08/16(水) 15:24:45.96 ID:???.net
キチガイ馬鹿はオンゲー馬鹿 義務教育の落ちこぼれ救いようもない

691 :ご冗談でしょう?名無しさん:2023/08/16(水) 15:35:44.45 ID:???.net
>>687
言い換えだけで成り立つのが証明できなければ意味がない

自分で証明できないなら、数学教科書の証明を調べろ

証明の意味も理解できない丸暗記馬鹿は数学も物理もやめろ

692 :ご冗談でしょう?名無しさん:2023/08/16(水) 15:48:33.46 ID:???.net
「最小の空間があります!」キチガイ説では >>687の式もまったく証明できない!

693 :ご冗談でしょう?名無しさん:2023/08/16(水) 18:06:51.85 ID:???.net
>>692
マトモな義務教育を受けた人間同士ならば、原子が(性質を持つ最小の)球体のような
イメージを共有できる。
ところが
引き籠りのオンゲーキチガイはディスプレーの最小升目ありきで、原子が升目の倍数の
大きさだと妄想するらしい。 
物理学とはまったく関係ない、デジタルゲームだらけ現代の精神病にすぎない。

694 :ご冗談でしょう?名無しさん:2023/08/16(水) 18:46:00.54 ID:QM3BMda1.net
数式による魔除けないと、物理はこうなるんだろな。ある種のフィルターが科学社会の健全さ維持には必要か。悪貨が科学に入り込まないためには。

数学的表現優先の数理物理みたいなものは、どうも好きじゃないけど、これ見ると仕方ないのか。

こんなのがいたら、科学とか成り立たないもんな。アカデメイアの入学条件は至言だな。

幾何学を学ばざるもの入るべからず。

695 :ご冗談でしょう?名無しさん:2023/08/16(水) 20:41:28.84 ID:02a6Eiub.net
え?
でも升目がとんでもなく小さければ問題おきないすっよ!
そんな事もわからねえんすか?

696 :ご冗談でしょう?名無しさん:2023/08/16(水) 20:50:46.16 ID:???.net
>>695
頭悪いザクだな

簡単にいえば、幾らでも区間を小さくできるというのが数学と物理の空間の概念だ!

0とはちがうのだよ、0とは!

697 :ご冗談でしょう?名無しさん:2023/08/16(水) 20:53:47.59 ID:EVBjyqtx.net
lim

698 :ご冗談でしょう?名無しさん:2023/08/16(水) 21:10:02.98 ID:02a6Eiub.net
電子とかクォークの大きさが10の-18乗以下なら
空間の最小単位が10の-50乗くらいなら
余裕でクリアできるよ

699 :ご冗談でしょう?名無しさん:2023/08/16(水) 21:12:38.92 ID:02a6Eiub.net
いくらでも小さくなったら無限大が生じちゃうだろ?
大好きな逆2乗が破綻せんのかい?

700 :ご冗談でしょう?名無しさん:2023/08/16(水) 21:15:34.85 ID:02a6Eiub.net
数学と物理は違うだろ?
数学なら何次元だって作れるが
物理はそうじゃないだろ。
だからこそいろいろ制約があるわけで

701 :ご冗談でしょう?名無しさん:2023/08/16(水) 21:18:54.64 ID:02a6Eiub.net
というわけで数学的空間と実際の空間は違うと思うねw

702 :ご冗談でしょう?名無しさん:2023/08/16(水) 21:34:59.39 ID:???.net
物理理論とは数学の理論だ、数学が厳密に成り立たない説など即ゴミ箱

物理学は

俺様説の実際???とは違うのだよ、俺様説とは

703 :ご冗談でしょう?名無しさん:2023/08/16(水) 21:44:26.16 ID:???.net
物理学の「原子」とは(量子論の)数学で創られた原子モデルのことだ!
本当は
「電子」、「ニュートリノ」、「クォーク」なども、標準理論などの数学がなければ存在しないのだよ
現実の人間感覚では存在しないものだ、数学理論を理解できない奴には無意味な言葉でしかない。

704 :ご冗談でしょう?名無しさん:2023/08/16(水) 22:00:30.25 ID:???.net
円周率、円の面積πr^2 など、つまり微積分学が成り立たない俺様説など
物理学でも何でもないのだよ。

ということがキチガイは死ぬまでわからない。

705 :ご冗談でしょう?名無しさん:2023/08/16(水) 22:34:09.91 ID:02a6Eiub.net
物理と数学を区別できない狂人オッス!w

706 :ご冗談でしょう?名無しさん:2023/08/16(水) 22:39:43.43 ID:???.net
自分で円周率πを計算するコンピュータプログラムを書いた学生なら解るが

微積分学により円周率πは無限個の有理数の加算で完全に記述される

どんなデジタルコンピュータにも最小数があり、限られたメモリー容量しかない

コンピュータプログラムは円周率πの適当な桁数までの計算で停止するようにセットする

つまり、微積分学の数学が始めにありきの近似計算だということだ。

707 :ご冗談でしょう?名無しさん:2023/08/16(水) 22:40:29.88 ID:02a6Eiub.net
ゼロの大きさのものみ見つかってねぇし
真円のものも見つかってねえだろ
見つかってもねえなら
物理と数学が同じとは言えねえだろが、このタコ

708 :ご冗談でしょう?名無しさん:2023/08/16(水) 22:42:48.61 ID:???.net
>>705
お前のオレ様説のように数学がない物理???などデタラメでしかない

709 :ご冗談でしょう?名無しさん:2023/08/16(水) 22:44:40.37 ID:02a6Eiub.net
そう、空間の最小単位が有ることで
物理は数学のあくまで近似となるんですよ
分かりますか?このタコ

710 :ご冗談でしょう?名無しさん:2023/08/16(水) 22:47:18.62 ID:02a6Eiub.net
無限に小さいなんてのは、無限大と同じで
実際には存在しないのだよ

711 :ご冗談でしょう?名無しさん:2023/08/16(水) 22:47:47.04 ID:???.net
>>707
お前の物理?とは自分の感覚だけだろが

物理学とは違うのだからさっさと感覚ありきの5ちゃん板にでも池

712 :ご冗談でしょう?名無しさん:2023/08/16(水) 22:49:59.42 ID:02a6Eiub.net
ほらほら、ゼロの大きさのものは見つかったのかい?
真円のものは見つかったのかい?
え?どうなんですか?w

713 :ご冗談でしょう?名無しさん:2023/08/16(水) 22:50:53.09 ID:???.net
>>706
で具体的に「最小単位がある場合」を見せても感覚馬鹿には理解不能なようだ

714 :ご冗談でしょう?名無しさん:2023/08/16(水) 22:52:30.51 ID:02a6Eiub.net
無限に小さいが許されるなら
原子の中にもっと小さい宇宙があっても許されるわな

715 :ご冗談でしょう?名無しさん:2023/08/16(水) 22:56:38.95 ID:02a6Eiub.net
ホレホレ、逃げてないでこたえてよこのタコ!

ゼロの大きさのものは見つかったのかい?
真円のものは見つかったのかい?

716 :ご冗談でしょう?名無しさん:2023/08/16(水) 23:02:32.54 ID:???.net
完全な円周率πがなけば、中学生でも習う一定速度で回転する物理運動すら解けない。

キチガイ馬鹿にはそれが全く理解できない。

717 :ご冗談でしょう?名無しさん:2023/08/16(水) 23:05:53.41 ID:???.net
デタラメの俺様説では任意の回転ができないのだからな

718 :ご冗談でしょう?名無しさん:2023/08/16(水) 23:12:12.42 ID:???.net
分数の理解も分数計算も出来ない高校生、大学生も実在するからな

719 :ご冗談でしょう?名無しさん:2023/08/16(水) 23:18:09.95 ID:???.net
馬鹿馬鹿しい言説にも有意な部分はあるが
言う意味のない状況だな

720 :ご冗談でしょう?名無しさん:2023/08/16(水) 23:19:19.57 ID:???.net
分数も分数計算も出来ない高校生大学生レベルで微積分は理解不能

721 :ご冗談でしょう?名無しさん:2023/08/16(水) 23:19:35.41 ID:02a6Eiub.net
やれやれ┐(-。-;)┌
結局、0の実在もπの実在もしめせなかったね。
物理は数学の近似でも成り立つことが理解できない
痴呆老人様ですね。

722 :ご冗談でしょう?名無しさん:2023/08/16(水) 23:23:39.96 ID:???.net
>>721
実在??? お前のキチガイ感覚だけだろ

物理学の実在とはアインシュタインが言ってるように物理(数学)理論で定義するのだよ。

723 :ご冗談でしょう?名無しさん:2023/08/16(水) 23:33:55.13 ID:???.net
>>722
つまり
数学理論で0,1,π...が定義され、ニュートン力学で物理量に使えば物理的実在となる。

724 :ご冗談でしょう?名無しさん:2023/08/16(水) 23:42:30.66 ID:???.net
>>693
格子ゲージ理論って知ってる?
>>694
ゲージ理論の幾何学的言い換え知ってる?

725 :ご冗談でしょう?名無しさん:2023/08/16(水) 23:57:42.45 ID:???.net
>物理学の実在とは物理(数学)理論で定義する

ニュートン力学で定義した真空空間は幾ら分割しても空っぽ

現代素粒子論によれば真空空間を分割していくと無数の素粒子が生成消滅を繰り返し
ているダイナミックな状態

726 :ご冗談でしょう?名無しさん:2023/08/17(木) 00:50:08.30 ID:???.net
オマエラって聞きかじりでおかしな想像して、おかしな世界観を作り上げることが物理だと思ってる節あるよな

727 :ご冗談でしょう?名無しさん:2023/08/17(木) 11:41:35.67 ID:???.net
言える

728 :ご冗談でしょう?名無しさん:2023/08/17(木) 13:21:40.60 ID:JL0cbVtc.net
全く何もない空間に座標系などというものを考えられるのでしょうか?

何か拠り所がないと原点すら決められないと思います。

729 :ご冗談でしょう?名無しさん:2023/08/17(木) 13:34:19.09 ID:JL0cbVtc.net
なにかものがただ一つある空間には原点は考えることができると思います。
そのものがある場所を原点と考えることができます。

730 :ご冗談でしょう?名無しさん:2023/08/17(木) 13:34:53.21 ID:JL0cbVtc.net
ですが、座標軸は考えることはできないと思います。

731 :ご冗談でしょう?名無しさん:2023/08/17(木) 13:36:21.71 ID:???.net
ムニちゃーんポポ

732 :ご冗談でしょう?名無しさん:2023/08/17(木) 13:36:46.27 ID:JL0cbVtc.net
ちょうど2つのものがある空間には原点と1つの座標軸を考えることができると思います。
1つのものがある場所を原点と考え、原点から他のものがある場所を正の方向とする座標軸を考えることができると思います。

733 :ご冗談でしょう?名無しさん:2023/08/17(木) 13:37:17.23 ID:JL0cbVtc.net
ですが、他の2つの座標軸は考えることができないと思います。

734 :ご冗談でしょう?名無しさん:2023/08/17(木) 13:38:35.31 ID:JL0cbVtc.net
ものが3つある空間ではじめて座標系を考えることができると思います。

735 :ご冗談でしょう?名無しさん:2023/08/17(木) 14:35:59.45 ID:JL0cbVtc.net
『物理学序論としての力学』という本に、「図2.2のように空間に直交直線座標系を設定しよう。」などと無造作に書かれています。

ものがない空間にどうやって座標系を設定できるのでしょうか?

736 :ご冗談でしょう?名無しさん:2023/08/17(木) 15:14:28.98 ID:???.net
たくさんの点間の正確な距離を測定します
次にその点を起点に架空の座標京を脳内、AI、理論上に作ります

737 :ご冗談でしょう?名無しさん:2023/08/17(木) 17:51:18.37 ID:???.net
何の根拠もなく考えてる奴が大勢いるやろ

738 :ご冗談でしょう?名無しさん:2023/08/17(木) 18:12:59.58 ID:???.net
>>735
幾何や多様体の基礎の基礎から勉強し直せ

739 :ご冗談でしょう?名無しさん:2023/08/17(木) 21:06:39.18 ID:K2/lNVSs.net
どうして、なになにし直せ、みたいな言い方しか出来んのかね?
ちゃんと分かりやすく理路整然と手取り足取り説明できてこそ
上級者というもの。

740 :ご冗談でしょう?名無しさん:2023/08/17(木) 21:11:50.21 ID:K2/lNVSs.net
これはこれで素朴な質問ではないの?
現宇宙外の真に何も無いということを想定していらしたのかもしれんぞ!
確かにそんなところでは座標もなにも無いのかもしれん。空間ですらない。

741 :ご冗談でしょう?名無しさん:2023/08/17(木) 21:17:40.89 ID:K2/lNVSs.net
たいがい、なになにし直せ、みたいに高飛車な態度を取る人って
実は自分の未熟を誤魔化すためにそういう態度を取ってるんだよね。

742 :ご冗談でしょう?名無しさん:2023/08/17(木) 21:32:34.17 ID:???.net
物理学はオカルトのように何でもありではない。

相間、量間が認めらないどころか、基礎の数学も否定するだけの妄想など当然。

743 :ご冗談でしょう?名無しさん:2023/08/17(木) 21:49:13.44 ID:???.net
無限遠の平行光源があれば平行移動とホロノミーは定義できるか。
無限遠の点光源でも「射影」的なことは定義出来て遠近法や幾何光学は成立する。

744 :ご冗談でしょう?名無しさん:2023/08/17(木) 22:50:30.37 ID:K2/lNVSs.net
いや、だから無限遠って、あるの?

745 :ご冗談でしょう?名無しさん:2023/08/17(木) 23:02:58.42 ID:???.net
>>744
ゼロとか無限遠とか無限小とかが先見的かどうかはご自分で考察為されては?

746 :ご冗談でしょう?名無しさん:2023/08/18(金) 00:50:45.61 ID:???.net
>>740
宇宙の外なんて無い
という意味でしかない

747 :ご冗談でしょう?名無しさん:2023/08/18(金) 00:54:02.62 ID:???.net
>>744
普通にあるぜ
複素平面に無限遠点を付け加えたのが
リーマン球面だ
何の矛盾も不思議もない

748 :ご冗談でしょう?名無しさん:2023/08/18(金) 00:56:44.59 ID:???.net
ついでに言うと
リーマン球面の斉次座標がスピノルだ
これから球面の回転とスピノルの関係が出る

749 :ご冗談でしょう?名無しさん:2023/08/18(金) 05:48:25.59 ID:???.net
ムーニーちゃんしんぷの大冒険だい

750 :ご冗談でしょう?名無しさん:2023/08/18(金) 11:09:18.82 ID:CY3oUB5e.net
いや、だから、無限遠というのが実在してるのか?とういことね。数学的に考えられるかどうかを聞いてないの。わかりますか?

751 :ご冗談でしょう?名無しさん:2023/08/18(金) 11:13:41.58 ID:???.net
現代物理学に実在など無意味
哲学板に行け

752 :ご冗談でしょう?名無しさん:2023/08/18(金) 11:48:30.71 ID:???.net
実在してる"物"や"現象"を説明するのが物理じゃないの?

753 :ご冗談でしょう?名無しさん:2023/08/18(金) 12:23:48.96 ID:???.net
>>752
違うね、物理やってると「実在」という言葉がいかに我々の感覚に依存した曖昧な概念がというのがわかってくる

754 :ご冗談でしょう?名無しさん:2023/08/18(金) 12:24:47.79 ID:???.net
>>750
知的障害者かよ

755 :ご冗談でしょう?名無しさん:2023/08/18(金) 12:48:44.78 ID:???.net
アースとか表面とかポテンシャル論と等温座標系

756 :ご冗談でしょう?名無しさん:2023/08/18(金) 15:40:26.00 ID:JrrQH6bL.net
ああ、なるほど。
物理やり過ぎて、現実と数学的仮想が区別出来なくなったのね。可哀想に。

757 :無学な私:2023/08/18(金) 16:45:41.33 ID:k+2T/v6e.net
このスレで、空間や物質や時間の、最小単位を主張する人がいるけど
最小単位が存在するなら、この世界は、
最小単位の積分で成り立っているの?

758 :ご冗談でしょう?名無しさん:2023/08/18(金) 17:55:08.56 ID:ktzZ27VX.net
そういう考え方だよ。

759 :ご冗談でしょう?名無しさん:2023/08/18(金) 18:43:33.57 ID:???.net
最小単位が存在する実験的根拠は一切ない
ただのオカルトだよ

760 :ご冗談でしょう?名無しさん:2023/08/18(金) 20:17:56.20 ID:twLnFdlN.net
光が横波なのは皆さんご存知でしょう。
物質の移動というのは空間の最小単位による縦波なのでは?

761 :ご冗談でしょう?名無しさん:2023/08/18(金) 20:38:33.30 ID:???.net
>>760
違うけど

762 :ご冗談でしょう?名無しさん:2023/08/18(金) 21:24:34.98 ID:4/fLyMYy.net
この宇宙の最大のエネルギーで2つの粒子を衝突させて何も出てこなかったら、
その粒子は完全な基本粒子と言ってよい。
そんなことはできないから、最小の粒子があるかは不明のまま。

763 :ご冗談でしょう?名無しさん:2023/08/18(金) 23:45:33.75 ID:???.net
>>753
実在=測定結果 でも?

764 :ご冗談でしょう?名無しさん:2023/08/18(金) 23:51:57.27 ID:???.net
粒子同士を運動エネルギーで衝突させれば素粒子は幾らでも発生する

陽子の数千倍のエネルギーLHC加速器による実験でも強い力で結合してる
陽子は不思議なことに破壊して分解できない
陽子と陽子を衝突させても結果的に p + p -> p + p + etc となるだけ

クォークに分解できない、etcで発生した多数の粒子を観測してるだけ
現在でもクォークそのものは観測されていない、理論でも不可能とされてる

それでもQCD理論が公認されている限りクォークは物理的実在なのだ。

765 :ご冗談でしょう?名無しさん:2023/08/19(土) 07:57:17.70 ID:lZCsAPYx.net
近くなると自由になるとか実にあやしい
別の理由があると思われる

766 :無学な私:2023/08/19(土) 08:32:36.69 ID:cGceaRzB.net
最小単位を考える時、次の様な課題が考えられる

・時間の最小単位
そもそも、時間とは何か、それは実在か、分割可能か

・距離の最小単位
最小の距離を計る時、その物差しは何か

これらが、アキレスと亀の、最小単位による解決の課題になるはず

767 :ご冗談でしょう?名無しさん:2023/08/19(土) 09:51:46.17 ID:???.net
>>766
ならないけど

768 :ご冗談でしょう?名無しさん:2023/08/19(土) 12:13:33.23 ID:CHpeMNFT.net
あらゆる体験を整合的に説明することが望みであり、それがまっとうな物理学だ。
@体験=観測
A整合的な説明=数学
と考えてよい。

しかし、問題がある。
@すべての体験(観測)が数学で表現できるわけではない。
A我々は世界のすべてを体験してるわけではない。
つまり、本質的な問題が残り続ける。

769 :ご冗談でしょう?名無しさん:2023/08/19(土) 12:27:04.52 ID:JCHwNFPy.net
中間子の構成みると、なにこれ?みたいのがいっぱい。むちゃくちゃと言ってもよいレベル。
崩壊過程の粒子の質量とかいみあるのかよ?

770 :ご冗談でしょう?名無しさん:2023/08/19(土) 12:33:49.13 ID:CHpeMNFT.net
>>758 つづき
さらなる問題もある。
@数学は信頼に値するか? →不完全性定理、三体問題、過渡現象、複雑性・・
A我々の体験は確実なものか? →五感の制約、幻想と現実、認識形式・・・

ゴールは遠いだけではなく、出発点さえ危うい。

771 :ご冗談でしょう?名無しさん:2023/08/19(土) 12:37:27.58 ID:???.net
>>763
測定結果は解釈次第
解釈がなきゃ無意味
現在の解釈は実在が無意味

772 :poem:2023/08/19(土) 12:38:54.92 ID:Se7RrlhH.net
>>768,770
支持
いいこと言ってる

773 :ご冗談でしょう?名無しさん:2023/08/19(土) 14:14:42.77 ID:???.net
つまり測定結果は実在ではなく妄想の類

774 :ご冗談でしょう?名無しさん:2023/08/19(土) 14:50:16.79 ID:???.net
IDでageるオカルト・キチガイはいつまでスレ荒らし続けるのか

775 :ご冗談でしょう?名無しさん:2023/08/19(土) 15:47:11.55 ID:HSTEBaiT.net
物理で線形代数が役立つ場面を教えてください。
確かにベクトルは使いますし、内積や外積なども使いますが、それだけですか?

776 :ご冗談でしょう?名無しさん:2023/08/19(土) 16:25:27.94 ID:???.net
>>775
足し算引き算が物理のどこで出てきますかって言ってるようなもんやぞ

777 :ご冗談でしょう?名無しさん:2023/08/19(土) 16:44:23.43 ID:???.net
量子力学は線形性が真に必要だと思うの

778 :ご冗談でしょう?名無しさん:2023/08/19(土) 16:48:32.32 ID:???.net
ピカー

779 :ご冗談でしょう?名無しさん:2023/08/19(土) 17:32:29.48 ID:L/UR6R7o.net
 
 
 
絶対零度ではあらゆる原子の振動が止まるというが
 
ではなんで電子の移動は滞りなくされるわけや
 
温度が下がるにつれて電子も止まって電流流れにくくなる、
抵抗値が上がってしかるべきではないのか
 
 

780 :ご冗談でしょう?名無しさん:2023/08/19(土) 17:46:55.34 ID:HSTEBaiT.net
ジョルダン標準形なんて使いますか?

781 :ご冗談でしょう?名無しさん:2023/08/19(土) 17:50:18.50 ID:???.net
使う

782 :ご冗談でしょう?名無しさん:2023/08/19(土) 17:50:44.15 ID:HSTEBaiT.net
多変数の微分積分、ベクトル解析、微分方程式

他に物理に必要な数学は何ですか?

783 :ご冗談でしょう?名無しさん:2023/08/19(土) 19:47:51.22 ID:CHpeMNFT.net
既存の物理学を学ぶなら、自ずと必要な数学が判る。
新しい物理学を探求するなら、新しい数学を創る覚悟をせねばならない。

784 :ご冗談でしょう?名無しさん:2023/08/19(土) 20:49:31.14 ID:6EXyKn4a.net
>>782
物理数学の本の目次を見てみるといいよ
たとえば
http://www.pleiades-publishing.co.jp/pdf/pdf16.html
http://www.pleiades-publishing.co.jp/pdf/data/pdf16.pdf

785 :ご冗談でしょう?名無しさん:2023/08/19(土) 20:51:45.27 ID:nJQeOu17.net
髪の毛が逆立つと髪が電極の役目を果たすため落雷の危険があると聞きましたが、その仕組みに関する質問です。
電磁気学に関することだと思ったためこちらで質問させていただきます。スレチでしたら申し訳ないです。

落雷の危険を避けるため髪の毛が逆立った場合は屋内などの安全な場所に避難することが必要だと聞き及んでいるのですが、仮に屋内に避難できない状況に置かれた人が、逆立った髪の毛を濡らしたり金属と接触させたりして静電気を除去した場合と何もしなかった場合で落雷の危険度に差は出ますでしょうか。

786 :ご冗談でしょう?名無しさん:2023/08/19(土) 21:19:38.48 ID:???.net
濡らしたら更に危険
金属はアースする形状なら危険が減るが
そうでなければ雷を呼び寄せて危険
それぞれジッパー効果, ボタン効果と言うが
ジッパー効果は
日本大気電気学会 雷から命を守るための心得
https://www.saej.jp/publications/hint.html
にあるがボタン効果はググっても出てこんな
危険だから教えんのか?

787 :ご冗談でしょう?名無しさん:2023/08/19(土) 21:22:30.18 ID:nJQeOu17.net
髪の毛が逆立つと髪が電極の役目を果たすため落雷の危険があると聞きましたが、その仕組みに関する質問です。
電磁気学に関することだと思ったためこちらで質問させていただきます。スレチでしたら申し訳ないです。

落雷の危険を避けるため髪の毛が逆立った場合は屋内などの安全な場所に避難することが必要だと聞き及んでいるのですが、仮に屋内に避難できない状況に置かれた人が、逆立った髪の毛を濡らしたり金属と接触させたりして静電気を除去した場合と何もしなかった場合で落雷の危険度に差は出ますでしょうか。

788 :ご冗談でしょう?名無しさん:2023/08/19(土) 21:26:08.35 ID:???.net
>>782
位相幾何、微分幾何、代数幾何、複素関数論

789 :ご冗談でしょう?名無しさん:2023/08/20(日) 01:06:01.00 ID:???.net
>>787
>髪の毛が逆立つと髪が電極の役目を果たすため落雷の危険がある
検電器の原理と同じ

髪の毛が逆立つのはその人間と付近の地表が誘電している、上空の帯電雲から
付近に落雷の可能性がある。
濡らしたり金属接触は効果ない、その地域から離れるか屋内で身を屈めて過ごす。

790 :ご冗談でしょう?名無しさん:2023/08/20(日) 03:05:18.40 ID:???.net
髪を逆立たせている静電気の電荷が雲底の電荷と同符号なら
危険を減らしてくれそうな気もするが

791 :ご冗談でしょう?名無しさん:2023/08/20(日) 12:16:21.68 ID:???.net
>髪を逆立たせている静電気の電荷が雲底の電荷と同符号なら

雲底の電荷の静電誘導だから人体の電荷は逆符号

792 :無学な私:2023/08/20(日) 12:25:22.69 ID:F1ClYrqr.net
>>773
観測が力学的に観測対象に干渉する事が、観測の不完全性を見事に証明しているという指摘もあるけど

むしろ、量子レベルでは、それを逆用して、量子の一見して不可解な振る舞いは、観測の影響である等と、疑問の解決策に利用されているという皮肉

しかし、ここで観測の影響を克服したのが、弱測定
ところが、弱測定により、観測の影響を取り除くと、その結果はさらに不可解で、その代表が、

未来から過去への干渉

793 :ご冗談でしょう?名無しさん:2023/08/20(日) 15:41:23.79 ID:???.net
>>787
こいつ答を見ない奴じゃん

794 :ご冗談でしょう?名無しさん:2023/08/20(日) 17:25:52.10 ID:ope06y8v.net
>>789
788.789

回答ありがとうございます。
髪の毛の静電気を除去した程度では、何もしなかった場合と比較しても危険度に差は出ないんですね。

>>791
>>雲底の電荷の静電誘導だから人体の電荷は逆符号

これは雲底が負の電荷が集まっていて、人体は正の電荷を持っているから、髪の毛の静電気を除去したとしても人体そのものが正の電荷のため大した意味がないという理解であってますでしょうか?
またその場合、近くに髪の逆立った人間や何かしらの構造物があったとして、そんな状況で1人だけが髪の毛の静電気を除去した場合にはほんの少しでも落雷の危険性が減るということはありますでしょうか。

795 :ご冗談でしょう?名無しさん:2023/08/20(日) 18:58:55.57 ID:zhVAZeQV.net
皮膚を透けて見える血管が青く見えるのはなぜですか?
本当に青い光がでているのか、周りとの対比で青く見えてるだけ?

796 :poem:2023/08/20(日) 20:15:08.97 ID:oS8FjQpi.net
>>792

最後の方

\ナンダッテーッッッ/
な感想

まじか

797 :ご冗談でしょう?名無しさん:2023/08/21(月) 00:32:04.88 ID:???.net
でぼぼぼぼぼ でぼぼぼぼぼぼぼぼぼぼ でぼぼぼぼぼぼぼぼぼぼぼ

798 :無学な私:2023/08/21(月) 01:12:44.58 ID:DCyi0oPl.net
>>796
勉強不足ですね、日経サイエンスを読みましょう

宇宙の未来が決める現在
https://www.nikkei-science.com/page/magazine/0910/200910_024.html

799 :ご冗談でしょう?名無しさん:2023/08/21(月) 09:30:25.64 ID:???.net
>>791
雷雲の帯電で地上の人間の髪の毛が逆立つほど静電気帯びることはなかろう
人間の静電気はその他の要因(下敷きでこするとかやりましたよね)だろうから、
何で静電気起こしたかでどっちの符号の静電気になるかは変わる

800 :ご冗談でしょう?名無しさん:2023/08/21(月) 09:44:22.35 ID:K80w4fQa.net
巨視的非実在性(月がないかもなど)、重力の非実在性など(概念として)ありますが
重力は観測するまで幾らかわからないなら、
量子力学の強い力等も観測にしだいでバラツキがあるんですか

801 :ご冗談でしょう?名無しさん:2023/08/21(月) 09:50:03.31 ID:K80w4fQa.net
重力の量子化、量子重力理論ってそれをそのまま考えるよりも
ホログラフィック原理、AdS/CFT対応を厳密に解いて、それを手かがりにしたらといいとおもうのですが
重力と量子をつなぐ公式とおもいますが

802 :ご冗談でしょう?名無しさん:2023/08/21(月) 13:21:42.61 ID:???.net
>>795
血管が青く見えるんじゃなく
血が青く見えるんだよ
腕をしごいて静脈の血を退けると
青色が消えるからすぐ分かるぜ
血は透過光で見ると赤く見え
反射光で見ると青く見える
考えてみりゃ当然だろ
反射光で見て赤く見えると思ってるのは
透過光が反射したのを見てるだけ
反射光だけを見えるのは静脈を見る時くらいだ

803 :ご冗談でしょう?名無しさん:2023/08/21(月) 13:26:35.41 ID:???.net
>>801
量子重力理論はもうできてる

高エネルギー加速器研究機構
量子重力理論と宇宙論
https://research.kek.jp/people/hamada/quantum%20gravity%20and%20cosmology1.pdf

804 :ご冗談でしょう?名無しさん:2023/08/21(月) 15:12:16.82 ID:???.net
量子と言えば、量子水素エネルギー

常温核融合か?と思われてた時期は、重水素と重水素が核融合してヘリウムになっていると思われていた。
しかし中性子が出ない、おかしい。

それから30年後。現在は、ニッケルと銅の電極に軽水素と導入して発熱が観測されている。
軽水である。陽子。
実は陽子と陽子が核融合する反応ではなく、
ニッケルと陽子が核融合して銅が生成する反応で熱が発生しているらしい。
銅と水素が核融合して亜鉛も出来ているかもしれない。

805 :ご冗談でしょう?名無しさん:2023/08/21(月) 18:17:40.10 ID:iChwjtxd.net
血を反射光で見ると青いとか聞いたこともない。

806 :ご冗談でしょう?名無しさん:2023/08/21(月) 20:16:22.96 ID:???.net
床に垂れた血は赤いよね

807 :ご冗談でしょう?名無しさん:2023/08/21(月) 20:17:39.11 ID:???.net
https://scienceportal.jst.go.jp/newsflash/20140627_01/#:~:text=%E4%BA%BA%E9%96%93%E3%81%AE%E9%9D%99%E8%84%88%E3%81%AF%E3%80%81%E5%91%A8%E5%9B%B2,%E6%B5%AE%E3%81%8B%E3%81%B3%E4%B8%8A%E3%81%8C%E3%82%89%E3%81%9B%E3%82%8B%E3%81%93%E3%81%A8%E3%81%8C%E3%81%A7%E3%81%8D%E3%82%8B%E3%80%82

錯覚よのうだ

808 :ご冗談でしょう?名無しさん:2023/08/21(月) 20:45:55.55 ID:xjGeyQ0u.net
>>799
https://www.msn.com/ja-jp/news/world/%E7%A0%82%E6%BC%A0%E3%81%A7%E9%AB%AA%E3%81%AE%E6%AF%9B%E3%81%8C%E9%80%86%E7%AB%8B%E3%81%A4%E3%80%81%E5%B0%82%E9%96%80%E5%AE%B6%E3%80%8C%E5%8D%B1%E3%81%AA%E3%81%84%E3%80%8D%E2%80%95%E4%B8%AD%E5%9B%BD/ar-AA1fkIs7

ここに書いてあるのをみてここで質問したのですが、ごく稀に雷雲の下だと髪の毛が逆立つということもあるみたいです。
このような状況下において、静電気を除去することによりどれくらいの落雷の危険性が変わるのか気になった次第です。危険性がごく僅かでも変わるのか、それともまったく持って意味ないのか。

809 :ご冗談でしょう?名無しさん:2023/08/21(月) 21:04:20.13 ID:Wtk6HrkP.net
地面に這いつくばるのが良かろう

810 :ご冗談でしょう?名無しさん:2023/08/21(月) 21:10:05.09 ID:BdmfNrs0.net
血液が吸収する光の波長を計算して出した色なのか?

血管が吸収する光の波長を計算して出した色なのか?

811 :ご冗談でしょう?名無しさん:2023/08/21(月) 21:30:43.02 ID:???.net
説明を理解するより
貶して喜ぶ方がお好き

812 :ご冗談でしょう?名無しさん:2023/08/21(月) 21:41:22.07 ID:BdmfNrs0.net
ド・ブロイの関係式で出すのでは

813 :無学な私:2023/08/21(月) 22:04:05.22 ID:DCyi0oPl.net
>>801
宇宙が存在する条件として、
ゼロへの収束と、無限大への発散の回避、
この2つが必要

量子と重力の課題もこれだな、つまり、
量子レベルのミクロの距離と、重力の強さの関係

2点間の距離が0に近付くと、重力は無限大に近付く、だから、重力理論に残された課題は、量子スケールにおける、
0と無限大

814 :ご冗談でしょう?名無しさん:2023/08/21(月) 22:16:14.51 ID:Wtk6HrkP.net
空間の最小単位が有るから無問題
0の大きさは有りません

815 :ご冗談でしょう?名無しさん:2023/08/21(月) 22:30:17.63 ID:???.net
最小単位があると言い切るなら具体的にいくつよ?

816 :ご冗談でしょう?名無しさん:2023/08/22(火) 00:41:45.58 ID:???.net
>>815
トンデモどもの股間の矮小なオングストローム砲のスケールがこの世の宰相単位。

817 :ご冗談でしょう?名無しさん:2023/08/22(火) 07:37:46.89 ID:u2g1Ho2j.net
光は波、最小単位どうしの結合力と最小単位の重さで光の速度が決まる

818 :ご冗談でしょう?名無しさん:2023/08/22(火) 09:54:30.59 ID:???.net
でぼーーん

819 :ご冗談でしょう?名無しさん:2023/08/22(火) 10:14:29.35 ID:A9AsSehf.net
がちアホの痴呆爺はしんだか?

820 :ご冗談でしょう?名無しさん:2023/08/22(火) 10:18:08.32 ID:???.net
普通の微積分学によるコーシーの極限の定義を誰でも分かる言えば

位置の物理量の仮の誤差範囲εを 0< |ε|として区間を 0<δとすれば
δをさらに小さい値にすることで 0<y<|ε| にできるならば極限値が有る
と定義している。
具体的に
速度の定義に適用すれば空間の任意の距離、時間経過も0より大きい!
仮に距離・時間の最小単位を決めてもそれより小さい最小単位を定義できる
という意味だ。

キチガイが
物理根拠が全く無いオレ様説で「10^-50 m の最小単位があります!」と
幾らわめいても、他の人が「10^-51 m の最小単位があります!」と反論できる。
最小オレ様説とは仮の区間を言い換えただけと誰でも判る。

821 :ご冗談でしょう?名無しさん:2023/08/22(火) 11:28:47.84 ID:oy4JLwmW.net
お、がちアホ爺、
生きていたか!
すぐにエサに引っ掛かって出てくるところが良いぜ!w

822 :ご冗談でしょう?名無しさん:2023/08/22(火) 11:35:27.34 ID:oy4JLwmW.net
このがちアホ爺は前提条件というものを理解する能力がまるで無い。
こんなのが物理、数学をやってることが大間違い。

823 :ご冗談でしょう?名無しさん:2023/08/22(火) 11:41:23.36 ID:???.net
>>817
>光は波、最小単位どうしの結合力と最小単位の重さで光の速度が決まる

物質中の音波と速さと同じ説を馬鹿の一つ覚えで言い換えただけだろが
試しに
速度vで運動してる観測者からみた光の速度がどうなるかを説明してみせろ

824 :ご冗談でしょう?名無しさん:2023/08/22(火) 12:29:11.84 ID:???.net
相手にするだけ無駄なのは放置で

825 :ご冗談でしょう?名無しさん:2023/08/22(火) 12:48:35.41 ID:???.net
>>817
最小単位同士の結合力と重さでどうやってcが決まるのか具体的に書いてみ
どうせ出来ないんだろうけど

826 :無学な私:2023/08/22(火) 13:17:46.69 ID:Au/HAD2y.net
最小単位の質量を、最小単位の距離に置いた時の重力、これを求めたらノーベル賞では?

827 :ご冗談でしょう?名無しさん:2023/08/22(火) 13:40:39.64 ID:rdUDJ9vH.net
仮に最小単位があったとして、それも不確定性原理に従うから、

最小単位の距離(ΔL)と最小単位の時間(ΔT)が、不確定性関係にあったとしたら、
ΔL・ΔT ≧ ħ/2 そうでなければ、普通にΔQ・ΔP ≧ ħ/2 。

828 :ご冗談でしょう?名無しさん:2023/08/22(火) 14:24:25.42 ID:???.net
>>827
お前は他人の質問には何一つ答えらえないのに

>最小単位があったとして、それも不確定性原理に従う
コピペキチガイの一つ覚え用語(不確定性原理)を並べただけのデタラメ
並べでスレ荒らし

物理板からさっさと消えて オマエの巣のオカルト板に池

829 :ご冗談でしょう?名無しさん:2023/08/22(火) 14:46:34.59 ID:???.net
最小単位と言えば半導体プロセス
少し前までは3nmとか言っていたが
現在のインテルは20Aとか18Aである
Aはアンペアではなくオングストローム
そのうち1Aとかの世代になるに違いない

830 :ご冗談でしょう?名無しさん:2023/08/22(火) 15:06:24.44 ID:???.net
>ΔL・ΔT ≧ ħ/2
単位の合わない式を書くのはマヌケの証

831 :ご冗談でしょう?名無しさん:2023/08/22(火) 15:15:23.05 ID:???.net
>>829
>>829
>そのうち1Aとかの世代になるに違いない

推測のネタになってるムーアの法則が破綻する(2020年代後半)理由は
Aスケールの原子構造が実在することで統計的な半導体理論が成り立たなくなるから
coming soon

量子力学そのまんまの原子モデルに置き換わるから、マクロの因果律に従う論理素子
モデル(ダイオード、トランジスタ)は理論的に不可能になる

832 :ご冗談でしょう?名無しさん:2023/08/22(火) 15:32:27.02 ID:???.net
>>831
>ムーアの法則が破綻する(2020年代後半)
量子コンピュータ、自律AIは日本の科学技術復活の最後のチャンスでもある
が現実は先進国から周回遅れで差が更に開き続けている。

5ちゃん物理板の惨状から日本の科学教育水準の底辺がまる見えで不可能に近い。
impossible dream

833 :ご冗談でしょう?名無しさん:2023/08/22(火) 15:51:13.97 ID:53R2Gc6T.net
わはは、半導体プロセスが1Åになるとか、さすが最小単位のわからないがちアホ爺。

834 :ご冗談でしょう?名無しさん:2023/08/22(火) 16:30:02.24 ID:???.net
>>833
お前の空間最小???とかと、半導体プロセスは何の関係も無い!

キチガイは自分の妄想とコピペ記事を自分勝手に引っ付けると脳内麻薬が出て
気持ちいい脳疾患らしい

835 :ご冗談でしょう?名無しさん:2023/08/22(火) 20:07:30.92 ID:u2g1Ho2j.net
そうカリカリしなさんな、短い寿命がさらに縮みますぜ
がちアホの爺さんw

836 :ご冗談でしょう?名無しさん:2023/08/22(火) 20:22:15.90 ID:???.net
cpuの細線化は理論的にここまでと言われて技術革新で更新されたんだったかな
ただ、それでも限界で、
違った技術を用いてムーアの法則が続くかって
原子サイズの幅になるとねえ無理だろうと思うよ試算するまでもなく

電流が流れるってことは電磁場の発生があって、
細線化するとそのウェイトが大きくなるから
無理じゃないの

837 :ご冗談でしょう?名無しさん:2023/08/22(火) 20:36:15.67 ID:???.net
可能な断面積の最小化はいくつか候補があるんだろうけど
炭素系しか思い浮かばないが
メタルって抵抗値あるからどうだろうね
ドープしたセミコンの伝導帯だと・・できるならもう存在してるかそんなもん隣接に移動するのにエネルギー食うんだっけ?

838 :ご冗談でしょう?名無しさん:2023/08/22(火) 22:40:44.93 ID:???.net
もう回路じゃなく空間をポインテイング・ベクトルが伝播する素子になるんじゃね?

839 :無学な私:2023/08/23(水) 05:38:45.19 ID:295y7Fjx.net
アナログ回路に進化するとか?

電流の代わりにスピン流を使う着想もあったような

840 :ご冗談でしょう?名無しさん:2023/08/23(水) 09:47:04.66 ID:???.net
3次元とか9次元とか言うが、そもそも1次元がどうして発生するのかがわからない
1次元だけに集中して解明してほしい

841 :ご冗談でしょう?名無しさん:2023/08/23(水) 11:33:08.04 ID:???.net
半導体のプロセス数値は最小解像度みたいなものでMOSのゲート長ではない
でもまあ解像度が上がればMOSやら配線も小さくできるから
プロセスが微細化すれば、スマホサイズで1000コア100TBとか可能になるかも

842 :ご冗談でしょう?名無しさん:2023/08/23(水) 11:35:46.84 ID:???.net
>>840
>1次元だけに集中して解明
簡単
物理なら運動があるから1次元がある

運動が無ければ距離、時間も分からず現実の空間も無意味

つまり、数学の空間とは天才が物理から抽象化したもの

馬鹿な奴ほど始めに数学の空間ありきだと思い込む

843 :ご冗談でしょう?名無しさん:2023/08/23(水) 11:38:22.59 ID:???.net
>>841
電子のトンネル効果も知らんのか

844 :ご冗談でしょう?名無しさん:2023/08/23(水) 12:43:58.37 ID:???.net
当然、別の作動原理だろ

845 :ご冗談でしょう?名無しさん:2023/08/23(水) 12:58:03.83 ID:???.net
ぷっちーポポ ぷちプラーイ
ムニちゃーんポポ

846 :ご冗談でしょう?名無しさん:2023/08/23(水) 13:53:35.96 ID:???.net
>>844
>別の作動原理
それが日本で開発できたらTSMC,Samsungも潰せるだろな

847 :無学な私:2023/08/23(水) 15:40:11.44 ID:CreMIcRy.net
そこで、スピントロニクスですよ

848 :ご冗談でしょう?名無しさん:2023/08/23(水) 16:36:10.25 ID:dW7wyzKc.net
1次元なんて無いの!
あるのは、この世界、三次元+時間だけ。
時間ですら怪しいが
1次元とか2次元とかは数学的空想なのよ。

849 :ご冗談でしょう?名無しさん:2023/08/23(水) 17:05:37.70 ID:???.net
リアルの2次元の物理がどれだけあると思ってるんだよw

850 :ご冗談でしょう?名無しさん:2023/08/23(水) 19:20:15.79 ID:???.net
〇角形で平面を埋められるか?という数学の問題が、結晶の解だったり

851 :ご冗談でしょう?名無しさん:2023/08/23(水) 19:25:38.34 ID:???.net
正三角形の角度の合計が180度
正四角形の角度の合計が360度
正五角形の角度の合計が540度
どういう法則化と言うと、十の位が2づず減って、百の位が2づつ増える

たぶん正六角形の角度の合計は720度だろう
3.5角形の角度の合計は270度

852 :ご冗談でしょう?名無しさん:2023/08/23(水) 20:16:49.16 ID:dW7wyzKc.net
アホじゃね?
4角形は3角形が2個
5角形は3角形が3個
6角形は3角形が4個
7角形は3角形が5個

となるだけだよ、図を書いてみろ

853 :ご冗談でしょう?名無しさん:2023/08/23(水) 20:28:35.62 ID:???.net
ああ、正〇角形
辺の長さが同じで電気的均衡が取れてる
立方格子、面心立方格子、最密立方格子、全てそうか?

854 :ご冗談でしょう?名無しさん:2023/08/23(水) 20:29:10.21 ID:???.net
最密六方か

855 :ご冗談でしょう?名無しさん:2023/08/23(水) 20:32:24.23 ID:???.net
正三角形もできるはずだよね〜

856 :ご冗談でしょう?名無しさん:2023/08/23(水) 22:17:41.59 ID:???.net
高次元の結晶格子を3次元に射影したのが
準結晶とか聞いたな

857 :ご冗談でしょう?名無しさん:2023/08/24(木) 10:57:10.90 ID:???.net
8/24 福島原発のALPS処理水放出
海水希釈後のトリチウム濃度が1,500ベクレル/リットル未満
WHO飲料水基準 10,000ベクレル/リットル

1秒間に1,500個のトリチウム原子がβ崩壊するという物理意味である
練習問題
1リットル中に含まれるトリチウム原子の数を計算せよ
1リットルが全部水分子としてトリチウム水分子の比率を計算せよ

この程度の計算ができなければ反日勢力の風評デマに操られるだけだぞ

858 :ご冗談でしょう?名無しさん:2023/08/24(木) 11:12:11.33 ID:???.net
放射線のカウンタ装置は理論的に1個の放射線でも観測可能だから
地球上どこでも常に放射線をカチカチカチとカウントしている

859 :ご冗談でしょう?名無しさん:2023/08/24(木) 11:23:25.01 ID:???.net
トリチウムのベータ線はエネルギーが低すぎて測定できない
どうしているのかというと、放射線で発光する液体である液体シンチレータの中に
トリチウムを含むサンプルを混ぜて、光電子増倍管でその発光を長時間測定する
死ぬほど手間と時間がかかる

860 :ご冗談でしょう?名無しさん:2023/08/24(木) 13:47:12.07 ID:???.net
>>859
>トリチウムのベータ線はエネルギーが低すぎて測定できない
トリチウム崩壊の電子線のエネルギーは最大18.6keV でテレビブラウン管の蛍光面に当たる
電子以下のエネルギーでしかない。
ところが
地上の屋外に降り注ぐミューオンのエネルギーはなんと1GeV超えで毎秒約160個/m^2も
降り注いでいる 破壊エネルギーは100000倍!
人体中のDNAを直撃すれば完全に破壊され、現実に日常的に破壊されているだろう。

殆どの人間はその事実を無視して何一警戒も防護も一切せず、日光浴、アウトドア
作業してるのに、簡単に内外の反日勢力の風評デマに操られてしまうのだ!

861 :ご冗談でしょう?名無しさん:2023/08/24(木) 14:49:06.87 ID:???.net
物理的に考えれば海洋放出は正解ですよね

862 :ご冗談でしょう?名無しさん:2023/08/24(木) 15:16:35.89 ID:???.net
トリチウムってトリウムとは違うんだ
せっかくできたんだから、貴重資源として有効活用できないのかな
半導体製造時の洗浄とか、ラドンに代わる健康用途に

863 :ご冗談でしょう?名無しさん:2023/08/24(木) 15:39:28.13 ID:???.net
トリチウムは核融合炉の燃料だろ

864 :ご冗談でしょう?名無しさん:2023/08/24(木) 15:41:06.41 ID:???.net
>>861
中国はボイラーで炊いて蒸発させろと言ってたそうです

865 :ご冗談でしょう?名無しさん:2023/08/24(木) 16:13:19.10 ID:???.net
トリチウムも結局はただの水素でしょ

866 :ご冗談でしょう?名無しさん:2023/08/24(木) 16:28:47.09 ID:ny+THuCw.net
トリチウムはグラム300万円もする高価なもの。ただし、汚染水からの分離では費用が
かかり過ぎて合わない。

867 :ご冗談でしょう?名無しさん:2023/08/24(木) 19:20:43.28 ID:???.net
海洋放出すれば距離の指数関数で濃度が減衰するので放出が正解です

868 :ご冗談でしょう?名無しさん:2023/08/24(木) 20:41:08.98 ID:vjXZnprC.net
海洋放出せずに人工湖でも作って放置しておく方が問題少なくないかな?

869 :ご冗談でしょう?名無しさん:2023/08/24(木) 21:02:38.17 ID:???.net
処理水のトリチウムTは水分子として存在している主に HTO (H-O-T)

普通の水分子より蒸発しにくいから解放水槽に放置すればトリチウム濃度が増えていく。

870 :ご冗談でしょう?名無しさん:2023/08/24(木) 21:09:06.24 ID:???.net
>>869
水(H2O)と振動モード違うくね?
対称伸縮振動とかしなくなさそう

871 :ご冗談でしょう?名無しさん:2023/08/24(木) 21:29:19.97 ID:X1/KkCYW.net
極座標というのがあります。
極座標系の基底ベクトルはなぜあのようなものを選ぶのでしょうか?
同径方向の単位ベクトルを直交基底を構成するベクトルの1つに選ぶのは分かります。
2次元の場合には、直交基底を構成するもう一つのベクトルは、右手系にしたければあのようなベクトルを選ばざるを得ません。
3次元の場合に、直交基底を構成する他の2つのベクトルが、なぜあのベクトルたちでなければならないのかが分かりません。

872 :ご冗談でしょう?名無しさん:2023/08/24(木) 22:16:32.57 ID:???.net
角度が増加する方向を素直に取れば
ああなる

873 :ご冗談でしょう?名無しさん:2023/08/24(木) 23:03:52.06 ID:???.net
日本政府はトリチウムの濃度が基準以下だから大丈夫と言っていますがそれを30年続けたらアウトですよね?

874 :ご冗談でしょう?名無しさん:2023/08/24(木) 23:40:23.59 ID:???.net
なんで?

875 :ご冗談でしょう?名無しさん:2023/08/25(金) 04:08:43.34 ID:???.net
>>873
世界中の原発から海洋放出されてるトリチウム水は海水で拡散しながら
半減期12年で指数関数的に減少していく。

地球上の大部分のトリチウムは宇宙線と大気原子の衝突から放射線量換算で
年間7.2京ベクレルが常に供給されている。

876 :ご冗談でしょう?名無しさん:2023/08/25(金) 05:55:39.20 ID:???.net
人間の心理行動は物理学で直接計算不可能だが、風評デマに簡単に操られるのは事実

トリチウム(T)崩壊のエネルギーの10000倍超の上空からの放射線に裸で日光浴し
Tより遥かに発がん率が高いファーストフード化学物質を毎日むさぼり食い続けてる
という非常に危険な事実を平気で無視できるという一般人の精神状態は矛盾のだらけ

877 :ご冗談でしょう?名無しさん:2023/08/25(金) 07:27:25.78 ID:???.net
”ALPS処理水が安全なら水道水に使え” という類の主張が認められないわけ

放射能=原水爆: 大量殺傷の核兵器との連想刷り込み
未知の恐怖心: トリチウムなどが自然と体内に一定量が常に有るのを全く知らない
0か1かで判断: 量(濃度)の比較で危険かどうか判断できない、日本人に多い

実施すれば一般人の拒否反応をさらに増長させるだけで何のメリットもない。

878 :ご冗談でしょう?名無しさん:2023/08/25(金) 09:38:48.74 ID:???.net
>>868
タンクという人工湖での保管が限界に来たからの処置なんですが。
仮に人工湖での保管を続けるとして、これ以上どこにそれを作るの?
設置場所の自治体を説得できる?

879 :ご冗談でしょう?名無しさん:2023/08/25(金) 10:02:36.31 ID:???.net
>>873
トリチウムの半減期は10年ぐらいなので
事故から今までで半分
30年後には事故後に比べて0.5^4=6%ぐらいになっている。1/20ぐらい

880 :ご冗談でしょう?名無しさん:2023/08/25(金) 10:24:53.53 ID:kOcVIH+a.net
>>878
ダムを日本中に作ってたくさんの集落を沈めてきたんだからやる気さえあればなんとかなるのでは?

881 :ご冗談でしょう?名無しさん:2023/08/25(金) 10:52:13.98 ID:???.net
汚染水ガーと言ってる連中をやる気だけで説得できると本気で思ってるならオメデタイ

882 :ご冗談でしょう?名無しさん:2023/08/25(金) 11:00:09.17 ID:???.net
トリチウム水を飲んで放射線を測れば胃カメラ代わりにならないの?
胃カメラより放射線強度ははるかに弱いので安全なはず

883 :ご冗談でしょう?名無しさん:2023/08/25(金) 11:05:23.23 ID:???.net
サランラップで止まるような弱い放射線で胃の何を測れると?

884 :ご冗談でしょう?名無しさん:2023/08/25(金) 11:29:56.16 ID:???.net
>>883
そんな弱かったの
>>882撤回します

885 :ご冗談でしょう?名無しさん:2023/08/25(金) 12:12:55.58 ID:???.net
半減期が長い方が安全って本当ですか?

886 :ご冗談でしょう?名無しさん:2023/08/25(金) 12:16:15.38 ID:2TbH7bSP.net
>>872
なぜそのようにとるのですか?

887 :ご冗談でしょう?名無しさん:2023/08/25(金) 12:45:57.33 ID:???.net
素直だから

888 :ご冗談でしょう?名無しさん:2023/08/25(金) 12:55:01.50 ID:???.net
>>885
安全(基準)かどうかは個々の半減期ではなく、被ばく線量(シーベルト)で比較する。

889 :ご冗談でしょう?名無しさん:2023/08/25(金) 13:05:50.34 ID:???.net
>>886
角度(ラジアン)の変化を単位円の接線ベクトルと解釈すればいいだけ
極座標でも3ベクトルが直交してるのが直観的に分るだろ

890 :ご冗談でしょう?名無しさん:2023/08/25(金) 15:11:28.80 ID:???.net
>>888
半減期が長い(崩壊する粒子が少ない)ってことは放射線も少ないってことですよね?

891 :ご冗談でしょう?名無しさん:2023/08/25(金) 15:47:03.11 ID:???.net
親核の数が同じなら、ね。当たり前だが、膨大な数の親核があれば長寿命核でも放射線量も膨大。
安全かどうかは線量で比較すべきもの

892 :ご冗談でしょう?名無しさん:2023/08/26(土) 11:02:43.63 ID:???.net
風評被害を懸念する漁業者が自ら風評被害を拡散している
これが経済などでいう負のスパイラルの典型的な例

最初にそれを仕掛ける内外の反日勢力が風評デマをSNSマスコミなどで扇動ことで
漁業者が処理水放出反対する、ことで一般大衆が不安になり風評デマが拡散を繰り返す

科学的基準による安全性などハナから無視してのが分かる

893 :ご冗談でしょう?名無しさん:2023/08/26(土) 11:30:08.64 ID:???.net
しんぷさんはムーニーちゃんダンスをおどれ
ムニちゃーんしんぷ
ムニちゃーんダンス

894 :ご冗談でしょう?名無しさん:2023/08/26(土) 12:22:12.31 ID:???.net
風評を煽るのは常にマスゴミ

895 :ご冗談でしょう?名無しさん:2023/08/26(土) 13:01:15.16 ID:???.net
今はネットだろ

896 :ご冗談でしょう?名無しさん:2023/08/26(土) 13:23:27.47 ID:???.net
いいえ、マスゴミですよ

897 :ご冗談でしょう?名無しさん:2023/08/26(土) 15:10:55.19 ID:???.net
真空は何もない(エネルギーはあるが)と何かある物質の解釈を反対にしたらどうなのでしょう
真空には何かがびっしりあり、隙間の空白になったところが素粒子と
海中の水が真空、泡が素粒子のイメージ、ディラックの電子と陽電子の話にも似ている
解釈を変えても基本的には何かが変わるわけでないが、膨大な真空エネルギーを説明しやすい
ブラックホール、ダークエネルギーも説明が変わるかも

これまでも考えた人はいたのだろうが、うまくいかないのかな

898 :ご冗談でしょう?名無しさん:2023/08/26(土) 15:11:09.01 ID:???.net
真空は何もない(エネルギーはあるが)と何かある物質の解釈を反対にしたらどうなのでしょう
真空には何かがびっしりあり、隙間の空白になったところが素粒子と
海中の水が真空、泡が素粒子のイメージ、ディラックの電子と陽電子の話にも似ている
解釈を変えても基本的には何かが変わるわけでないが、膨大な真空エネルギーを説明しやすい
ブラックホール、ダークエネルギーも説明が変わるかも

これまでも考えた人はいたのだろうが、うまくいかないのかな

899 :ご冗談でしょう?名無しさん:2023/08/26(土) 15:11:39.53 ID:???.net
あれ、二回になっちゃった

900 :ご冗談でしょう?名無しさん:2023/08/26(土) 15:12:37.38 ID:qWcJfsEv.net
エーテル

901 :ご冗談でしょう?名無しさん:2023/08/26(土) 15:20:06.08 ID:???.net
>>900
それだ、あったな、それのバージョンアップ版

902 :ご冗談でしょう?名無しさん:2023/08/26(土) 17:29:13.40 ID:???.net
>>894
中国人朝鮮人は自国河川が有害物質で汚染されまくりの水産物を死ぬまで食ってればいい

全ての日本人が新鮮で美味の海の食材を安く食べられるように政府が漁業者を支援すべきだ

903 :ご冗談でしょう?名無しさん:2023/08/26(土) 17:38:23.26 ID:Kt46poce.net
ちうごくの農薬まみれ野菜の輸入を禁止すべきだろ?

904 :ご冗談でしょう?名無しさん:2023/08/26(土) 18:56:04.89 ID:???.net
>ちうごくの農薬まみれ野菜の輸入を禁止すべき
賛同
微弱なトリチウムより遥かに危険だ、なぜか左翼マスコミは報道しない

905 :ご冗談でしょう?名無しさん:2023/08/26(土) 20:11:18.46 ID:Kt46poce.net
日本のマスコミは基本、シナチョンのお先棒担ぎですから
反日反米は熱心に煽ります

906 :ご冗談でしょう?名無しさん:2023/08/27(日) 04:42:04.77 ID:ZruweeoG.net
極座標について質問です。

θ = arctan(y/x)

という誤った等式がかならずといっていいほど教科書に書かれているのはなぜですか?

点 (-1, -1) の極座標は、 (√2, (5/4)*π) です。
arctan(-1/-1) = arctan(1) = π/4 ≠ (5/4)*π です。

907 :ご冗談でしょう?名無しさん:2023/08/27(日) 06:05:25.82 ID:???.net
>>906
お前は関数の主値が理解できないだけだろが
y = x^2 の逆関数がどうなるか中学からやり直せ

908 :ご冗談でしょう?名無しさん:2023/08/27(日) 10:16:37.57 ID:ZruweeoG.net
>>907

θ = arctan(y/x)

と書くことは、

y = x^2 の例でいえば、

y = √x

と書くようなものです。

909 :ご冗談でしょう?名無しさん:2023/08/27(日) 10:17:11.62 ID:ZruweeoG.net
あるいは、

y = -√x

と書くようなものです。

まずいですよね。

910 :ご冗談でしょう?名無しさん:2023/08/27(日) 10:39:03.85 ID:???.net
>>909
日本語が読めんのか? 
お前は関数の主値が理解できないだけ

911 :ご冗談でしょう?名無しさん:2023/08/27(日) 11:01:51.91 ID:???.net
θ = arctan(y/x)  の θの値が無限個あるのも知らんのか

912 :ご冗談でしょう?名無しさん:2023/08/27(日) 13:24:06.54 ID:???.net
その式x=0で破綻してるよね

913 :ご冗談でしょう?名無しさん:2023/08/27(日) 13:40:46.78 ID:ZruweeoG.net
arctanと書くと主値を表す事が多いですよね。

>>912

その問題もありますね。
θ = atan2(x, y) と書けば何の問題もありませんよね。

914 :ご冗談でしょう?名無しさん:2023/08/27(日) 13:42:47.13 ID:ZruweeoG.net
訂正します:

θ = atan2(y, x) と書けば何の問題もありませんよね。

915 :ご冗談でしょう?名無しさん:2023/08/27(日) 13:57:49.60 ID:???.net
重箱の隅つついて悦にいってるな
後で惨めにならん?

916 :ご冗談でしょう?名無しさん:2023/08/27(日) 14:09:42.97 ID:???.net
これを重箱の隅と考えるようじゃ科学は向いてないな

917 :ご冗談でしょう?名無しさん:2023/08/27(日) 14:50:06.57 ID:???.net
でぼぼぼぼん

918 :ご冗談でしょう?名無しさん:2023/08/27(日) 16:13:55.60 ID:???.net
GIGAZINEに
IAEAが福島第一原発のALPS処理水放出システムをモニタリングできるウェブページを公開中
という記事が載ってた。
ただし
>モニタリングページで使われているデータはIAEAが直接計測しているのではなく、東京電力から送られているものであることに留意する必要があります。
という注意書きが付きで。

919 :ご冗談でしょう?名無しさん:2023/08/27(日) 16:16:35.09 ID:???.net
実数の関数レベルのスレレスではモヤモヤが解消できないだけだ諦めろ

920 :ご冗談でしょう?名無しさん:2023/08/27(日) 16:30:31.15 ID:???.net
>>918
>東京電力から送られている
紛らわしいな
当然だが、東京電力単独でALPS処理水の放射線を測定してれは疑われて反日勢力のカモだ
東京電力から独立した第三者機関が測定データをクロスチェックしている。

921 :ご冗談でしょう?名無しさん:2023/08/27(日) 16:40:30.07 ID:???.net
>>919
登記じいさん、まだスレとレスの区別すらつかないのか

922 :ご冗談でしょう?名無しさん:2023/08/27(日) 16:40:54.66 ID:???.net
マイナンバーカード問題も多重チェックせず一人で手作業させれば間違いが起こるのが当たり前

923 :ご冗談でしょう?名無しさん:2023/08/27(日) 16:52:11.22 ID:???.net
MSXでarctanがatnだったのはなぜだろう

924 :ご冗談でしょう?名無しさん:2023/08/27(日) 16:55:44.64 ID:???.net
他の関数記号と区別ができればいいだけだ
頭悪い奴ほどどうでもいい事にこだわる

925 :ご冗談でしょう?名無しさん:2023/08/27(日) 17:13:08.12 ID:???.net
昔のMPUは演算速度が遅く主メモリも小さい制約からプログラム言語の単語は4文字以下が多い

926 :ご冗談でしょう?名無しさん:2023/08/27(日) 17:24:46.10 ID:ZruweeoG.net
直交座標はx軸とy軸が必要です。
極座標は角度を測る基準となる有向直線と原点が必要です。
なぜ、極座標は補助的な座標系という位置づけなんですか?

927 :ご冗談でしょう?名無しさん:2023/08/27(日) 17:32:21.63 ID:???.net
>>926
頭悪いな、不便だからに決まってるだろ
普通に多用する平行移動の計算が簡単に出来ないだろが

928 :ご冗談でしょう?名無しさん:2023/08/27(日) 17:58:20.63 ID:???.net
>>916
攻撃で返すてーことは
痛いとこ突かれたか?

929 :ご冗談でしょう?名無しさん:2023/08/27(日) 17:59:00.61 ID:???.net
極座標が補助的なんて聞いたことがない

930 :ご冗談でしょう?名無しさん:2023/08/27(日) 18:00:57.74 ID:???.net
直交座標以外の座標は量子化などが上手くいかんかったりする

931 :ご冗談でしょう?名無しさん:2023/08/27(日) 19:54:29.21 ID:???.net
平面の1点は常にただ一つの(x, y)に対応する
しかし(r, θ)はそうならない
だから補助的の使われてる

932 :ご冗談でしょう?名無しさん:2023/08/27(日) 20:11:22.21 ID:???.net
なるほど

933 :ご冗談でしょう?名無しさん:2023/08/27(日) 20:24:13.56 ID:???.net
補助的なんてことはない
扱う系によって便利な座標系が選ばれる

934 :ご冗談でしょう?名無しさん:2023/08/28(月) 00:54:06.20 ID:???.net
頭の固った奴ばっかだな直交座標と極座標しかないと決めつけるのか

お前らが知らんだけで座標変換で直交座標に変換できる曲線座標は無数にあり
ベクトル、微積分も同様に定義できる

935 :ご冗談でしょう?名無しさん:2023/08/28(月) 01:00:41.52 ID:jg7mDup/.net
固った

936 :ご冗談でしょう?名無しさん:2023/08/28(月) 03:09:54.67 ID:???.net
>>933
このひとことで全てすむ話

937 :ご冗談でしょう?名無しさん:2023/08/28(月) 15:02:57.49 ID:???.net
>>934
その変換は全単射ではないことが問題なんだよ
本質的に等価でない

938 :ご冗談でしょう?名無しさん:2023/08/28(月) 15:05:02.88 ID:???.net
でぼぼぼぼ でぼぼぼぼぼぼ でぼぼぼ でぼん

939 :ご冗談でしょう?名無しさん:2023/08/28(月) 16:20:22.93 ID:???.net
斜交座標で共変と反変の違いが現れる

940 :ご冗談でしょう?名無しさん:2023/08/28(月) 16:35:40.00 ID:???.net
>>937
お前のような奴は実数の値しか見えないから諦めろと言ってるだろ

941 :ご冗談でしょう?名無しさん:2023/08/28(月) 16:47:48.01 ID:???.net
直観的に解りたければDungeons&Dragonsの周回の迷路に行け

942 :ご冗談でしょう?名無しさん:2023/08/28(月) 16:48:45.94 ID:???.net
その変換は原点で破綻している
一つの点が無限の点に対応している
これは全単射でないことを示している
斜行座標は一対一対応するので問題ないが

943 :ご冗談でしょう?名無しさん:2023/08/28(月) 16:49:59.22 ID:???.net
極座標のこの歪性はシュバルツシルトの見かけの事象の地平面にも現れる
直交座標で解けばこれは現れない

944 :ご冗談でしょう?名無しさん:2023/08/28(月) 16:51:57.93 ID:???.net
数学じゃ二つの構造間に全単射が存在するかは極めて重要な問題

945 :ご冗談でしょう?名無しさん:2023/08/28(月) 17:09:03.14 ID:???.net
原点は原点に対応させるだけ、なんの問題もない

946 :ご冗談でしょう?名無しさん:2023/08/28(月) 17:17:29.19 ID:???.net
点電荷のクーロン場は原点で定義できないが除外できるから矛盾が起こらない

947 :ご冗談でしょう?名無しさん:2023/08/28(月) 17:24:54.92 ID:???.net
>>934>>940は飛び抜けて頭が悪い

948 :ご冗談でしょう?名無しさん:2023/08/28(月) 17:40:21.77 ID:???.net
>>943
馬鹿

949 :ご冗談でしょう?名無しさん:2023/08/28(月) 17:42:46.66 ID:???.net
極座標は原点で連続性が消えてしまうからね
直交座標はそうはならない

950 :ご冗談でしょう?名無しさん:2023/08/28(月) 17:46:45.02 ID:???.net
>>943
直行座標でも駄目じゃね?
地平面の内部で時間が空間的になってしまうから
時間軸使うような座標系ではうまくいかないのでは?

951 :ご冗談でしょう?名無しさん:2023/08/28(月) 17:48:55.00 ID:???.net
>>949
頭悪い奴だな
数学では座標の原点で点を添加できるのだよ、だから座標の連続性が無くならない

952 :ご冗談でしょう?名無しさん:2023/08/28(月) 17:59:12.25 ID:???.net
>>951
簡単に説明しよう
直交座標の原点以外の各点は極座標の原点以外の点に変換される
直交座標の原点は極座標の原点に対応させる
それで連続性が保たれている

953 :ご冗談でしょう?名無しさん:2023/08/28(月) 18:11:22.87 ID:???.net
>>951
どのような定義をしようと極座標は原点で不連続です
連続な定義があるならしてみなさい

954 :ご冗談でしょう?名無しさん:2023/08/28(月) 18:15:27.40 ID:???.net
wikipediaでも言及されていますね

>特異点は (r, θ) = (0, θ) 即ち、xy座標での原点 (x, y) = (0, 0) である。

955 :ご冗談でしょう?名無しさん:2023/08/28(月) 18:33:49.65 ID:???.net
>>953
馬鹿は連続の定義も知らんのか

直交座標 y=0, ±x->0  ならば極座標 θ=0,θ=π, r->0 となるから
任意の方向からでも同じ、原点同士の点を対応させれば連続になる。

956 :ご冗談でしょう?名無しさん:2023/08/28(月) 18:38:32.20 ID:???.net
>>954
つまり、座標変換の特異点は除去できるということだ

957 :ご冗談でしょう?名無しさん:2023/08/28(月) 19:16:15.90 ID:???.net
>>934
原点での∂/∂θによる微分を定義してください

958 :ご冗談でしょう?名無しさん:2023/08/28(月) 19:26:49.12 ID:???.net
>>955
θ=0,π
それ連続じゃないですね

959 :ご冗談でしょう?名無しさん:2023/08/28(月) 19:31:39.20 ID:jg7mDup/.net
むしろ、原点の特異性を逆に利用して
Δ(r) = -4πδ^3(r)
のような便利な式が使える

960 :ご冗談でしょう?名無しさん:2023/08/28(月) 19:32:12.85 ID:jg7mDup/.net
ミスった
Δ(1/r) = -4πδ^3(r)


961 :ご冗談でしょう?名無しさん:2023/08/28(月) 19:41:43.78 ID:???.net
>>957
アホか
連続に出来ても特異点の性質は変わらん

962 :ご冗談でしょう?名無しさん:2023/08/28(月) 19:43:10.22 ID:jg7mDup/.net
連続に出来るって言ってるのはお前だろw
まじでこいつ日本語あやしすぎるwww

963 :ご冗談でしょう?名無しさん:2023/08/28(月) 19:46:08.33 ID:???.net
>>962
違いが判らん馬鹿

964 :ご冗談でしょう?名無しさん:2023/08/28(月) 19:49:49.30 ID:???.net
スレに湧くのは馬鹿ばかりで数学レベルが低すぎる

965 :ご冗談でしょう?名無しさん:2023/08/28(月) 19:59:15.05 ID:???.net
極座標は原点で連続じゃないですよ

966 :ご冗談でしょう?名無しさん:2023/08/28(月) 20:01:14.23 ID:???.net
原点への極限は方向依存性があるので連続ではないですね

967 :ご冗談でしょう?名無しさん:2023/08/28(月) 20:09:30.63 ID:???.net
馬鹿はまだ分からんのか
極座標の変換は原点で連続でない、点の穴が開いてる
だから、別な定義 直交の原点->極座標の原点 で穴埋めで連続にするだけだ

特異点性質は何も変わらん

968 :ご冗談でしょう?名無しさん:2023/08/28(月) 20:10:29.20 ID:???.net
物理数学においてウリジナル定義は不要

969 :ご冗談でしょう?名無しさん:2023/08/28(月) 20:10:56.14 ID:???.net
その定義で連続性の証明してください
出来ないですよね

970 :ご冗談でしょう?名無しさん:2023/08/28(月) 20:12:34.91 ID:???.net
>>968
馬鹿なおまえは解析数学で普通やってることも知らないだけだ

971 :ご冗談でしょう?名無しさん:2023/08/28(月) 20:15:36.35 ID:???.net
>>969
日本語が読めんのか
>>955 に簡単に証明してるだろが、詳細にやりたけば教科書を読め

972 :ご冗談でしょう?名無しさん:2023/08/28(月) 20:16:03.35 ID:???.net
>>955
それ連続の定義満たしてないですよ(笑)

973 :ご冗談でしょう?名無しさん:2023/08/28(月) 20:23:54.64 ID:???.net
>>972
お前頭わるすぎだろ、少しは考えろ

どの方向からも極限が0だから、 0,0 -> θ,0 で原点を埋めれば連続になる

974 :ご冗談でしょう?名無しさん:2023/08/28(月) 20:25:03.62 ID:???.net
f(r,θ)=θについてx軸上正方向から原点への極限値とy軸上正方向から原点への極限値を述べて下さい

975 :ご冗談でしょう?名無しさん:2023/08/28(月) 20:29:02.26 ID:???.net
>>974
意味不明

976 :ご冗談でしょう?名無しさん:2023/08/28(月) 20:31:59.93 ID:???.net
答えられないんですね

977 :ご冗談でしょう?名無しさん:2023/08/28(月) 20:37:41.49 ID:???.net
馬鹿はキリがないから、おしまい

極座標が連続に出来たので、θ=0(θ=π) 方向の原点を通るr(t)による
原点中心の単振動がやっと記述できるようになるのだよ。
(原点が不連続なら不可)

978 :ご冗談でしょう?名無しさん:2023/08/28(月) 20:42:45.71 ID:???.net
結局答えられてなくて草
連続の定義から勉強しましょうね~

979 :ご冗談でしょう?名無しさん:2023/08/28(月) 20:46:15.59 ID:???.net
まあ多変数関数の微積分を学んだことがなさそうなので仕方ないか

980 :ご冗談でしょう?名無しさん:2023/08/28(月) 20:47:44.31 ID:???.net
>>973
>どの方向からも極限が0

間違えてて草

981 :ご冗談でしょう?名無しさん:2023/08/28(月) 20:52:00.09 ID:???.net
またしょうもない定義の話題か?

982 :ご冗談でしょう?名無しさん:2023/08/28(月) 20:52:51.34 ID:???.net
>>973
普通に間違えてるやんw

983 :ご冗談でしょう?名無しさん:2023/08/28(月) 21:00:44.80 ID:jg7mDup/.net
>>971
お前っていっつも間違った説明して、教科書読めって言って逃げてるよなw

984 :ご冗談でしょう?名無しさん:2023/08/28(月) 21:01:37.89 ID:jg7mDup/.net
相手をバカ呼ばわりする前に、ちゃんと勉強して下さい。

985 :ご冗談でしょう?名無しさん:2023/08/28(月) 21:08:34.74 ID:???.net
ムーニーちゃんしんぷだい

986 :ご冗談でしょう?名無しさん:2023/08/28(月) 21:37:58.34 ID:???.net
馬鹿は治らんな

直交座標では原点で連続で値は(0,0)
r = √(x^2+y^2) 任意の方向から原点(0,0)に収束すれば
x->0,y->0 だから r->0
極座標(θ,r)の原点の値を(θ,0)と定義すればrが連続になる。

θが原点で連続でないのは原点を通過すれば自明

987 :ご冗談でしょう?名無しさん:2023/08/28(月) 21:44:58.84 ID:???.net
θが原点で連続でないことは、直交座標で符号が変るのと同様だから
物理運動では問題にならない。

988 :ご冗談でしょう?名無しさん:2023/08/28(月) 21:50:05.85 ID:???.net
直交座標でxが原点近傍で符号変化するのは連続ですよw

989 :ご冗談でしょう?名無しさん:2023/08/28(月) 21:51:17.28 ID:???.net
>>973
それ連続じゃないですw

990 :ご冗談でしょう?名無しさん:2023/08/28(月) 22:14:18.34 ID:???.net
>>957
極座標と直交座標の変換は
x = r cos θ
y = r sin θ
だから ∂/∂θ の計算は
∂x/∂θ = - r sin θ = - y
∂y/∂θ = r cos θ = x
原点での値は
∂x/∂θ = 0
∂y/∂θ = 0
これのどこが問題なんだ?

991 :ご冗談でしょう?名無しさん:2023/08/28(月) 22:19:00.85 ID:???.net
座標変換と単位変換のあいだに

ゲージ変換

を入れていいものなのだろうか?

992 :ご冗談でしょう?名無しさん:2023/08/28(月) 23:33:11.02 ID:???.net
>>990
その定義θが原点まわりで不連続ですよw

993 :ご冗談でしょう?名無しさん:2023/08/28(月) 23:34:05.94 ID:???.net
連続だ!と主張するくせに何故連続の定義は頑なに無視するんだろう🤔

994 :ご冗談でしょう?名無しさん:2023/08/28(月) 23:51:28.61 ID:???.net
平面での連続は全ての方向からの極限が一致することですね
θは原点でそれを満たさないので不連続ですね

995 :ご冗談でしょう?名無しさん:2023/08/29(火) 04:26:41.54 ID:???.net
ムニちゃーんポポ

996 :ご冗談でしょう?名無しさん:2023/08/29(火) 08:17:52.20 ID:???.net
>>946
微分形式と超関数の理論を一緒にした「カレント」の理論

997 :ご冗談でしょう?名無しさん:2023/08/29(火) 08:19:21.27 ID:???.net
>>944
ゲージスライスは同値類の代表元の選び方

998 :ご冗談でしょう?名無しさん:2023/08/29(火) 08:54:53.50 ID:???.net
>>996
>>997
アホw

999 :ご冗談でしょう?名無しさん:2023/08/29(火) 09:15:40.32 ID:???.net
>>998
数学コンプ?

1000 :ご冗談でしょう?名無しさん:2023/08/29(火) 11:39:22.64 ID:???.net
>>992
すり替えで逃げたか

1001 :ご冗談でしょう?名無しさん:2023/08/29(火) 11:40:58.91 ID:???.net
>>994
まず間違いを認めろよ

1002 :ご冗談でしょう?名無しさん:2023/08/29(火) 12:19:12.58 ID:???.net
>>1001
それはお前だろw
原点で連続性の定義満たすことを示せよw
君いつも教科書読めで逃げるよねw

1003 :2ch.net投稿限界:Over 1000 Thread
2ch.netからのレス数が1000に到達しました。

総レス数 1003
259 KB
掲示板に戻る 全部 前100 次100 最新50
read.cgi ver.24052200